Series 7

Ace your homework & exams now with Quizwiz!

the tender off, unless withdrawn, under Regulation 14E, must remain open for at least _______________ days from the offer is first announced

20 business days

in a long margin account, minimum maintenance is

25% of the LMV

short term gain or loss time frame

365 days or less

a BD must retain records of all of the identification information obtained from the customer for ______ years after the account is closed

5 years

long term debt is money borrowed for a minimum of

5 years -mostly 20-30 years

Sell stop at 52 and when the market is at 53 52.5 51.88 51.5. 51.75. 52.25

51.5 Triggered at 51.88 and then becomes a market order

LEAPS options with more an 9 months to expiration are marginable. Initial requirement is

75%

estate taxes are due...

9 months after death

sales charges on a fixed premium variable life contract may not exceed

9% of the payments to be made over the life of the contract

In order for a REIT to avoid being taxed like a corporation, it must distribute at least

90% of its taxable income

ABC corporation's 4% bond is trading at 98 & 5/8 the price is

986.25

calculate tax equivalent yield. 40% tax bracket 4.2% coupon municipal bond

=4.2%/(100%-40%) =7% investor would need to purchase a corporate or government with a coupon of 7%

Which of the following is a bull spread? A) Long Jul 30 put, short Jul 35 put B) Long Aug 30 call, short Aug 25 call C) Short Aug 40 call, short Aug 40 put D) Long May 40 put, short May 35 put

A A debit call spread is bullish and a credit put spread is bullish. Long Jul 30 put, short Jul 35 put is the only bullish position in the answer choices. Short Aug 40 call, short Aug 40 put is a short straddle, not a spread, and the remaining two positions are bearish: long Aug 30 call, short Aug 25 call and long May 40 put, short May 35 put. Remember, buying the low strike and selling the high strike is always a bull spread, regardless of the spread being puts or calls. LO 10.e BUY LOW SELL HIGH

One of the ways in which U.S. government agency issues differ from those offered directly by the U.S. Treasury is that A) agency issues typically carry higher returns than Treasury issues because of the lack of direct government backing. B) agency issues frequently trade on the NYSE, while Treasuries never do. C) agency issues are taxable on the federal level, while Treasury issues are not. D) agency issues are more likely to be issued in larger amounts.

A Agencies, with very few exceptions (GNMA being one), do not carry the direct backing of the U.S. Treasury. While they are quite safe, that lack of direct backing causes their yields to be somewhat higher. Agencies are never traded on the stock exchanges, and their float is almost always smaller than Treasuries. Both are taxable on the federal level. LO 7.d

In which of the following will a change in interest rates cause the greatest price fluctuation? A) 7% 30-year U.S. Treasury bond B) 7% AAA-rated corporate bond with eight years until maturity C) Series EE bond D) 7% AA-rated one-year municipal note

A Price fluctuations are the greatest in bonds with the longest terms to maturity. The riskier the instrument, the more price volatility. Long-term bonds have greater risk than short-term bonds. LO 4.e

A technology fund manager concerned about a downturn in the value of his portfolio would hedge by A) buying narrow-based index puts. B) selling narrow-based index calls. C) selling broad-based index calls. D) buying broad-based index puts.

A The portfolio consists of sector-specific securities, so broad-based index puts such as the OEX would not be appropriate. Instead, the manager should buy narrow-based index puts (for example, indices on technology and electronics). LO 10.g TECH

Zero-Tranche CMO (Z-Tranche)

A Z-tranche receives no payment until all preceding CMO tranches are retired (the most volatile CMO tranches). *Z TRANCH WOULD NOT BE SUITABLE FOR AN INVESTOR NEEDING FUNDS IN A SPECIFIED AMOUNT OF TIME BECAUSE OF THE UNPREDICTABLE NATURE WHEN PAYMENTS WILL BE RECEIVED

Direct Participation Program (DPP)

A business organized so as to pass all income, gains, losses, and tax benefits to its owners, the investors; the business is usually structured as a limited partnership.

General Obligation Bonds (GOs)

A municipal debt issue backed by the full faith, credit, and taxing power of the issuer for payment of interest and principal.

Moral Obligation Bonds

A state or local-issued, or state or local agency-issued bond. If revenues or tax collections backing the bond are not sufficient to pay the debt service, the state legislature has the authority to appropriate funds to make payments. The potential backing by state revenues tends to make the bond more marketable, but the state's obligation is not established by law; it is a moral obligation only. *Moral obligation bonds are revenue bonds only. For instance, a state may take on the moral obligation to service debt on city-issued general obligation bonds when the city has surpassed its statutory debt limit. This situation occurred in New York City's financial crisis of 1975. Typically, moral obligation bonds are issued in times of financial distress and have increased credit risk.

If a husband makes a gift of $100,000 to his wife, a U.S. citizen, how much of the gift is subject to gift taxes? A) $0 B) $100,000 C) $50,000 D) $90,000

A) $0 Interspousal gifts to citizens of the United States, regardless of amount, are not subject to gift taxes. LO 13.h

When the inside market (best bid and best offer) for XYZ stock was 17.30-17.60, a market maker bought 100 shares from a customer at 16.90. At the time of the trade, the market maker's market was 17.25-17.70. What was the amount of the markdown? A) $0.40 B) $0.75 C) $0.35 D) $0.65

A) $0.40 Markdown is always based on the inside quote. In this case, the inside bid is 17.30 and the difference between that and the 16.90 buying price represents a $0.40 markdown. LO 16.e

An investor places an order to sell short ABC at the open. The opening bid of $15.25 is the same as the prior day's close. At what price may the investor sell short at the open? A) $15.25 B) $15.24 C) The investor is prohibited from selling short at the open D) $15.26

A) $15.25 Short sales can occur any time during the trading day, including the opening and the close. In this case, the sale would take place at the best bid, which is $15.25. LO 16.c

All or None Order (AON)

An order that instructs the firm to execute the entire order or to cancel it. Firm does not have to execute immediately. Can be a day order of GTC

At the open order

An order that is executed at the opening of the market. Partial executions are allowable.

Current Yield Formula

Annual Interest / Current Market Price

Variable Annuity

Annuity that has a varying rate of return based on the mutual funds in which one has invested

ACATS stands for

Automated Customer Account Transfer Service

The term high-yield bond would apply to a bond with a Moody's rating of A) BBB. B) Ba. C) Baa. D) BB.

B

A customer with an income objective who resides in a state with a high personal income tax might find it best to purchase A) Corporate bonds with an investment-grade rating. B) Bonds issued by the U.S. Virgin Islands. C) Mortgage-backed securities issued by the Government National Mortgage Association (GNMA). D) U.S. Treasury STRIPS.

B Bond issued by U. S. Territories, such as the Virgin Islands, are triple tax-exempt. That is, investors do not have to pay federal, state, or local income taxes on the interest. GNMA and corporate debt securities are taxable on all levels. Although the Treasury STRIP is exempt from state income tax, as a zero-coupon bond, it provides no income. LO 7.e

With regard to a variable annuity, all of the following may vary except A) the value of annuity units. B) the number of annuity units. C) the value of accumulation units. D) the number of accumulation units.

B During the accumulation phase, the number of accumulation units will increase as additional money is invested. When the contract is annuitized, the annuitant is credited with a fixed number of annuity units. Once annuitized, the number of annuity units does not vary. The value of accumulation and annuity units varies with the investment performance of the separate account. LO 9.c

It would be most unusual to see which of the following issued at a discount? A) Commercial paper B) Jumbo CD C) Treasury bill D) Banker's acceptance

B Jumbo (negotiable) CDs are one of the few money market instruments issued at face value. Unlike those issued at a discount, they are interest bearing. LO 4.c

Treasury STRIPS and Treasury receipts are quoted based on A) 0.03125 (1/32 of a point in dollars). B) yield to maturity. C) amortization of premiums. D) 0.125 (⅛ of a point in dollars).

B Noninterest-bearing securities, like zeroes, are quoted based on their yield to maturity. They are sold at a discount and mature at par. LO 7.a

Which of the following mortgage-backed securities would provide investors with the most predictable maturity date? A) Ginnie Maes B) PACs C) TACs D) Fannie Maes

B PACs are planned amortization class CMOs and have established maturity dates. Prepayment risk is transferred to the PAC companion, or support, class bonds. LO 12.d

Which of the following mortgage-backed securities would provide investors with the most predictable maturity date? A) Fannie Maes B) Planned amortization classes (PACs) C) Targeted amortization classes (TACs) D) Ginnie Maes

B PACs are planned amortization class collateralized mortgage obligations and have established maturity dates. Prepayment risk is transferred to the PAC companion—or support—class bonds. LO 12.d

A registered representative's customer is speaking of a variable life insurance contract she owns. She makes several statements regarding the contract. Which of the following is not an accurate statement concerning a variable life insurance contract? A) The policy provides a minimum guaranteed death benefit. B) The death benefit can never be more than the guaranteed benefit. C) The portion of the premium invested in the insurance company's general account is used to provide for the minimum guaranteed amount of the death benefit. D) There is no guarantee regarding the investment results of the separate account.

B The minimum guaranteed death benefit is provided by that portion of the payment invested in the insurance company's general account. The remainder of the premium is invested in the separate account. While there is no guarantee on how investments in the separate account will perform, depending on its investment performance, the separate account could provide for a larger death benefit than the minimum guaranteed amount. LO 9.a

An investor purchases 100 OID municipal bonds at 95 with a 10 year maturity. If the bonds are held to maturity, the investors consequences are A. $5000 ordinary income B. $5000 tax free income C. $5000 long term capital gain D. $4000 long term capital gain, $1000 short term capital gain

B accretion on an OID is tax free

Which of the following balance sheet items is not a current liability? A) Long-term debt amount that is due within one year B) Mortgages C) Accrued taxes D) Accounts payable

B) Mortgages Short-term or current liabilities are those entries on a balance sheet that are due in one year or less. Accounts payable, accrued taxes, and that portion of long-term debt due within the year are all current liabilities. Mortgages are generally long-term liabilities, although that portion of a mortgage that is due within the year would be classified on the balance sheet as a current liability. LO 13.c

An investor purchases a bond on its initial public offering. Even though the bond has a maturity value of $1,000 in 10 years, the offering price is only $600. If the bond is held to maturity, A) $400 is reported as ordinary income. B) there is a $400 long-term capital gain. C) there are no tax consequences to report. D) there is a $360 long-term capital gain and $40 in ordinary income.

C A bond issued at a significant discount from its maturity value is known as an original issue discount bond (OID). In the case of a corporate bond, the computation is more complex than can be tested, but there are two things you need to know: A portion of the discount is taxed as ordinary income each year until maturity, even though it is not actually received. This is called phantom income. Each year's taxable amount is reported on Form 1099-OID. Because a portion of the discount has been taxed each year, at maturity there are no tax consequences—no gain, no interest. LO 5.e

A holder of an American depositary receipt (ADR) assumes all of the following risks except A) foreign currency risk. B) political risk. C) liquidity risk. D) market risk.

C An American depositary receipt (ADR) represents an ownership interest in a foreign domiciled company. The shares trade on the New York Stock Exchange or in the OTC market. The risk of lack of liquidity is absorbed by the presence of trading on U.S. exchanges or in the OTC market. The shares are subject to market risk, political risk, and foreign currency risk. LO 3.g

A customer just opened a new account at your firm and gave her lawyer limited power of attorney (POA). Which of the following statements is true? A) Confirmations of trades will be sent to the account owner and the lawyer. B) Confirmations will only be sent to the party given the POA unless otherwise requested. C) The POA ceases upon the death of either party. D) The POA must be renewed on the last business day of April and October.

C Confirmations of trades will always be sent to the account owner. They may only be sent to a third party upon written request of the account owner. A POA need not be renewed and will cease upon the death of either party. It is a good-til-canceled (GTC) order that must be renewed at the end of April and October, not a POA. LO 1.d

Under ERISA, all of the following retirement plans must set standards for vesting, eligibility, and funding except A) profit-sharing plans. B) Keogh plans. C) deferred compensation plans. D) corporate pension plans.

C Deferred compensation plans are not qualified plans and may be discriminatory. Keogh, profit-sharing, and corporate pension plans must meet set standards for vesting, eligibility, and funding under ERISA. LO 1.i

Advantages of owning a real estate DPP program include all of the following except A) depreciation. B) appreciation. C) Depletion. D) cash flow.

C Depletion only applies to natural resources, such as oil or gas. Land does not get "used up" as does oil, gas, or coal, for example. LO 11.f

A type of alternative trading system that trades listed stocks and is required to register with the SEC as a broker-dealer is A) an ETN. B) a dark pool. C) an ECN. D) the fourth market.

C Electronic communication networks (ECNs) are a type of alternative trading system (ATS) that trade listed stocks and other exchange-traded products. Unlike dark pools, another type of ATS, ECNs display order in the consolidated quote stream. As ATSs, ECNs are required to register with the SEC as broker-dealers and are also members of FINRA. Trading in the fourth market (institution to institution) is done largely through ECNs. LO 3.h

A Treasury bond is quoted in The Wall Street Journal as follows: Bid 100:15 Asked 100:17 Bid Chg. -1 Yield 7.9 From this information, you know that the nominal yield is A) 7.90%. B) 7.89%. C) greater than 7.90%. D) less than 7.90%.

C Explanation The Bid and Asked prices show that the Treasury bond is being quoted at a premium (above par), with a yield to maturity of 7.9%. When bonds are trading at a premium, the nominal yield (coupon rate) is greater than the yield to maturity. LO 7.b

A customer long 100 shares of XYZ stock who wishes to reduce risk and generate income should A) buy an XYZ call. B) sell an XYZ put. C) sell an XYZ call. D) buy an XYZ put.

C If the customer sells a call, the risk of owning the stock is reduced by the call's premium. Receipt of the premium satisfies the customer's income objective. LO 10.d

ABC Corp. has outstanding a 10% noncumulative preferred stock. Two years ago, ABC omitted its preferred dividend. Last year, it paid a dividend of $5 per share. In order to pay a dividend to common shareholders, each preferred share must be paid a dividend of A) $5. B) $25. C) $10. D) $15.

C In order for a common dividend to be paid when the issuer has noncumulative preferred stock outstanding, the preferred dividend must be satisfied. In this case, the stated dividend is 10% or $10 (10% × par of $100). LO 3.e

Moody's Investment Grade (MIG) ratings are applied to A) municipal bonds. B) corporate bonds. C) municipal notes. D) money market instruments.

C Moody's Investment Grade ratings are applied to municipal notes, which are short-term municipal debts such as bond anticipation notes (BANs). LO 4.f

The industry term "junk bond" applies to a bond with a Standard and Poor's rating no higher than A) BBB. B) C. C) BB. D) B.

C Once a bond's rating has fallen below the top four grades (AAA, AA, A, and BBB), it is no longer considered investment grade. At that point, BB (or Moody's Ba) or lower, it is considered a high-yield or junk bond. LO 4.f

Which of the following is an issuer of federal agency securities? A) The U.S. Treasury B) The California Urban Development Authority C) The Tennessee Valley Authority D) The Indiana Highway Authority

C The TVA is a federal agency formed as an act of Congress in 1933. As such, the debt securities it issues are considered federal agency securities. Treasury issues are simply that—issues of the Treasury, not an agency. The other two choices are municipal bonds. LO 7.c

An investor purchased 10 GO bonds at a discount of 2 points per bond. The bonds mature in 10 years. After holding the bonds for 5 years, they were sold at par. For tax purposes, the investor has A) a $50 gain. B) no gain and no loss. C) a $100 gain. D) a $100 loss.

C The cost per bond is $980. The accretion amount each year is $20. $20 ÷ 10 years = $2 per year. $2 per year × 5 years = $10 per bond accretion, making the adjusted cost basis $990 per bond. When the bonds are sold at par ($1,000), there is a profit of $10 per bond × 10 bonds, which equals a $100 gain. LO 6.e

Your customer opened a Roth IRA on February 2 and deposited $5,000. In August of the same the year, the value of the account had risen to $6,000, but by December, the value declined to $5,100 and the customer closed the securities positions and withdrew the entire amount. If the customer was age 55 at the time of the withdrawal, what would be the early withdrawal penalty? A) $510 B) $500 C) $10 D) 0

C The customer would have an early withdrawal penalty of $10. A 10% early withdrawal penalty applies to the taxable amount withdrawn for those aged less than 59½.In a Roth IRA, the investment is made with after-tax dollars. Therefore, only the increase above the investment is subject to an early withdrawal penalty. In this case, the earnings of $100 would be taxable and have a 10% penalty equaling $10. LO 1.g

You have a client who invested in the PQR Growth Fund 10 years ago and now, as retirement age approaches, asks you about using the exchange privilege to move into the PQR Balanced Fund. The client should know that A) the old shares are liquidated at NAV and the new shares are purchased at the POP. B) the exchange qualifies for any breakpoint reduction. C) this exchange is considered a taxable event as of the date of the exchange. D) any tax consequences are deferred until the Balanced Fund shares are liquidated.

C The exchange privilege allows for an exchange at net asset value (NAV) between funds that are members of the same "family." The exchange is considered a taxable event. Because the exchange is made at NAV, the concept of breakpoint is irrelevant. LO 8.e

An investor is concerned about safety. When consulting the ratings, which of the following securities would appear to be least likely to default on its obligation to make timely payment of interest and principal? A) A rated mortgage bond B) AAA rated common stock C) AA rated debenture D) BB rated sovereign debt

C When it comes to reducing default risk, "the As have it." That is, the more As in the rating, the lower the default risk. True, the common stock is rated triple A, but stock has no obligation to pay interest and repay principal. Why isn't the mortgage bond a safer bet than the debenture? Aren't secured bonds the safest? These are good questions, but the rating services take that into consideration when giving a rating. In their eyes, the debenture, an unsecured debt, merits a double A rating while the mortgage bond, even with the pledged collateral, can only be awarded a single A rating. Sovereign debt, the debt of a country's government, is usually quite safe, but history has shown us that governments can, and do, default. The BB rating here indicates a certain question as to the safety. LO 4.f

ABC Corporation has outstanding a 7.75% convertible debenture currently trading at 102. The bond is convertible into common stock at $40. ABC stock is trading $45 per share. Which of the following statements is true? A) The bond is at parity with the stock. B) An arbitrage opportunity does not exist in this situation. C) To profit in this situation, the investor should buy the bonds and short the stock. D) To profit in this situation, the investor should buy the stock and short the bonds.

C With a conversion price of $40, the bond is convertible into 25 shares of ABC common stock ($1,000 / $40 = 25 shares). As the common stock is currently trading at $45 per share, the value of the stock as converted would be $1,125 (25 shares × $45 = $1,125), which is greater than the current price of the bond ($1,020). Therefore, the bond and the stock are not at parity. An investor could profit in this situation by shorting the stock and buying an equivalent number of bonds. A bond could be purchased for $1,020 and immediately converted into stock worth $1,125—a risk-free profit opportunity. LO 5.d

If an investor opens a new margin account and buys 100 shares of DEF at 39, with Regulation T at 50%, what is the investor's initial margin requirement? A) $1,950 B) $975 C) $2,000 D) $3,900

C) $2,000 In a new margin account, for first trades between $2,000 and $4,000, the initial margin requirement is $2,000. LO 16.d

If XYZ common stock has a $4 dividend, a yield of 4.2%, a price-to-earnings (P/E) ratio of 12, and it is trading at $96, its approximate earnings per share (EPS) is A) $4.00. B) $50.40. C) $8.00. D) $48.00.

C) $8.00. The stock's P/E ratio is price to EPS. Dividing the stock's price by the P/E will give the EPS ($96 / 12 = $8). LO 13.d

A customer has placed an open order to buy 1,600 shares of GHI at $60. GHI declares a 25% stock dividend. On the ex-date, this order is considered a buy limit order for A) 1,600 shares at $45. B) 2,000 shares at $60. C) 2,000 shares at $48. D) 1,600 shares at $50.

C) 2,000 shares at $48. The order is adjusted on the ex-date. The number of shares increases by the percentage of the stock dividend, and the specified price is reduced to compensate. In this case, the number of shares increased to 2,000 (1,600 + 25%), and the specified price is adjusted to $48 per share. To get the adjusted price, divide the total value of the original market order (1,600 × $60 = $96,000) by the new number of shares ($96,000 ÷ 2,000 = $48). The order's total market value, $96,000, remains the same. LO 16.a

Each of the following statements concerning fill-or-kill (FOK) orders and all-or-none (AON) orders are true except A) an AON order must be filled in its entirety. B) an FOK order must be filled in its entirety. C) an AON order must be canceled if the whole order cannot be executed immediately. D) an FOK order must be canceled if the whole order cannot be executed immediately.

C) an AON order must be canceled if the whole order cannot be executed immediately.

A margin account is restricted by $2,000. which of the following actions may the customer take to bring to Reg T req? A. W/d $2000 of SMA B. Deposit $1000 of cash C. Deposit $4000 of fully paid marginable stock D. Deposit $2000 of fully paid marginable stock

C. Deposit $4000 of fully paid marginable stock

what document is checked by the RR to verify the accuracy of the trade? A. customer confirmation B. BD confirmation C. execution report D. order flow report

C. execution report

*Returns on CMOs

CMOs yield more than treasury securities and normally pay investors interest and principal monthly

Electronic Municipal Market Access (EMMA)

Centralized online tool for locating key information about municipal securities. Presented for retail, non-professional investors. -provides real-time prices

waiver of premium

Continuation of life insurance coverage if the insured becomes totally disabled and is unable to pay the premiums.

*Easiest corporate characteristics to avoid for a partnership

Continuity of life -partnerships avoid this because they have a predetermined date of dissolution

Purchase of 1000 shares of THV for $20. Cash purchase $20 to $15

Customer experiences -25% loss (gain/initial investment) -5k/20k

Purchase of 1000 shares of THV for $20. Margin purchase $20 to $15

Customer experiences -50% loss (gain/initial investment) 5k/10k

Purchase of 1000 shares of THV for $20. Margin purchase Stock goes from $20 to $30

Customer experiences 100% return (gain/initial investment) 10k/10k

Purchase of 1000 shares of THV for $20. Cash purchase $20 to $30

Customer experiences 50% return (gain/initial investment) 10k/20k

A J & J Treasury bond with a 5% coupon due July 1, 2019, is purchased in a cash transaction on February 24. What is the number of days of accrued interest? A) 53 B) 55 C) 63 D) 54

D A bond begins accruing interest on the prior interest payment date (January 1) and accrues up to, but not including, the settlement date (February 24). Did you notice that this was a cash transaction? That means the settlement date is the same day as the trade (February 24). Normally, Treasury securities settle T+1. If this was a regular-way trade, the accrued interest would be 55 days because settelment would have been February 25. Be careful reading the question; it is easy to skip over critical information. Because accrued interest on government bonds is computed actual days, actual year, 31 days for January plus 23 days for February, it equals 54 days. LO 6.e

Although there are general suitability rules that always apply, FINRA's Rule 2330 on variable annuity suitability specifies that, to be considered suitable, there is a reasonable basis to believe that the customer has been informed—in general terms—of various features of A) immediate variable annuities. B) single premium variable annuities. C) deferred annuities of all types. D) deferred variable annuities.

D FINRA's primary suitability concern is with deferred variable annuities. That does not mean there are no requirements for being careful with the others, it is just that most of the violations have involved the deferred VA. LO 9.e

Which investor has the greatest potential risk if the price of QRS goes up? A) Long 10 calls on QRS B) Short 10 puts on QRS C) Long 10 puts on QRS D) Short 10 calls on QRS

D If the market is rising, the greatest potential risk that an investor can take is a short naked call because the potential loss is unlimited. Please note that any short option, put or call, is uncovered unless something in the question indicates the opposite. LO 10.h

Greater Growth Capital (GGC), a FINRA member firm, has just been acquired by Better Retirement Outcomes (BRO), a much larger FINRA member. If GGC would like to effect a bulk transfer of its customer accounts using a negative consent procedure, FINRA rules A) require GGC to obtain affirmative written consent before transferring a customer's account to BRO. B) prohibit GGC from charging a fee to any existing GGC customers who elect to transfer their accounts to BRO, but permit a nominal charge if the customer wishes to transfer to another member firm. C) require that GGC send a notice to each affected customer at least 60 calendar days before it effects the bulk transfer. D) prohibit GGC from charging a fee to any existing GGC customers who decide to transfer their accounts to a different firm.

D In the case of a bulk transfer, such as when a member firm is acquired by another member, some customers may wish to opt out of the transfer. In those cases, it is prohibited for them to be charged any fees for transferring their accounts to another member firm. This is true only if the customer opts out during the allowable time period, which is 30 calendar days, not 60. The FINRA rule permits negative consent as long as the requirements are met. LO 1.e

All of the following securities would be suitable investments for a traditional IRA except A) blue-chip common stocks. B) A rated corporate bonds. C) AAA rated U.S. government agency bonds. D) AAA rated municipal bonds.

D Municipal bonds, which generate tax-free interest income, are unsuitable for retirement plans. One loses the federally tax-free income at distribution. LO 1.g

On April 15, 2016, your client purchased a variable life insurance policy with a death benefit of $450,000. The November 2019 statement showed a cash value of $28,000. If the client wanted to borrow as much as possible, the insurance company would have to allow a loan of at least A) $25,200. B) $14,000. C) $21,000. D) $28,000.

D Once a variable life policy is in force for a minimum of three years (this one is a bit longer than that), there is a requirement to make the loan provision available. At the three-year mark, that minimum becomes 75% of the computed cash value. Seventy-five percent of cash value of $28,000 is $21,000. LO 9.e

A structured instrument known as an asset-backed security would not be backed by A) credit card debt. B) auto loans. C) student loans. D) loans on marginable securities.

D One common theme uniting asset-backed securities is the contractual obligation to make payments. In the case of a margin account, there is no repayment schedule. The margin debt can exist for years with the only payment being that of interest. LO 12.c

For which of the following business structures is the income taxed to the business? A) A general partnership B) An S corporation C) A limited partnership D) A C corporation

D Partnerships, limited or general, and S corporations do not pay income tax. Any income earned by the business flows through to the owners. On the other hand, C corporations are taxable entities and must pay tax on their income before they can distribute dividends to shareholders. LO 1.c

Which of the following is not considered a debt security? A) Debenture B) Promissory note C) Equipment trust certificate D) Prior lien preferred stock

D Stock, whether preferred or common, represents equity (ownership) and is never considered a debt security. The most common example of a promissory note on the exam is commercial paper, a money market instrument. Debentures represent an unsecured debt of the issuer. Equipment trust certificates represent debt secured by specific equipment, typically rolling stock. LO 4.a

Your customer wishes to lock in a long-term yield with minimal risk and is not interested in regular income. Which of the following securities should you recommend? A) Treasury bill B) Corporate A-rated zero-coupon bond C) Treasury bond D) Treasury STRIPS

D The Treasury STRIPS is long-term, no-interim income security and has a locked-in yield because it is purchased at a discount from par. The Treasury bill is short term, the Treasury bond provides semiannual interest, and the corporate zero is riskier than the STRIPS. LO 7.a

Which of the following persons must sign a stock or bond power to effect good delivery of securities sold from an account set up under the Uniform Transfer to Minors Act? A) Minor B) Parent C) Donor D) Custodian

D The custodian must sign securities in a custodial account to effect good delivery. LO 1.b

An aggressive investor buys ABC stock with a beta of 1.7. The S&P 500 has a 10% rate of return for the year, and ABC's return is 12%. What is the alpha for ABC? A) +5% B) +3% C) -3% D) -5%

D) -5% With a beta of 1.7, an investor would expect ABC stock to be 70% more volatile than the general market as measured by the S&P 500. Remember, the beta of the market is 1.0. Therefore, we would expect to see the stock return 17% based on the S&P 500's 10% return. However, the actual return on ABC was only 12%. Alpha measures the difference in the actual return vs. the expected return. The difference between the expected return (sometimes referred to as the required return) of 17% and the actual return of 12% is a negative 5%. That represents an alpha of -5% for ABC stock. LO 13.b

An investor sold 100 shares of RAN common stock short at $50 per share. The RAN is now at $38. The investor is still bearish on the stock but would like to protect that gain. What would you recommend if this were your client? A) Enter a sell stop order at $40 B) Enter a sell order at $40 C) Enter a buy stop order at $55 D) Enter a buy stop order at $40

D) Enter a buy stop order at $40 Buy stop orders are used as a protective tool for short sellers. In this case, if the stock should begin to rise from its current price of $38, once it reaches or exceeds $40, a buy order at the market is entered. The stock purchased is used to cover the short position, and the investor's profit is the $50 sale price minus the cost of the purchase. Buy stops are always placed above the current market. Because the investor is short, the only protective order would be a buy, not a sell (you buy protection).. LO 16.a

A corporation has determined that if it were to go bankrupt, common stockholders would receive $8.47 per share. This calculation is known as A) retained earnings per share. B) settlement value per share. C) net asset value per share. D) book value per share.

D) book value per share. The liquidating value of the company is its book value; the book value per share is what the common stockholders would receive. LO 13.d

On the order book, all of the following orders are reduced on the ex-date for a cash dividend except A) buy limit. B) sell stop limit. C) sell stop. D) buy stop.

D) buy stop. Only orders placed below the market price are reduced for cash dividends on the order book. Buy limits and sell stops are entered below the market price. Buy stops are entered above the market price. LO 16.a

The visible supply has been increasing steadily over the past 30 days. This is an indication that A) fewer new issues will be offered in the next 30 days. B) yields are likely to fall. C) prices are likely to rise. D) yields are likely to rise.

D) yields are likely to rise. When the visible supply increases, it tells us that the number of bond issues coming to market is increasing. Greater supply puts downward pressure on prices. As bond prices fall, yields increase. LO 13.f

T or F: short selling is allowed in a IRA

F

Long-Term Equity Anticipation Securities (LEAPS)

Long-term call and put equity options with expirations of up to 39 months

oils and gas partnership-income degree of risk

Low

Value managers tend to see a _______ P/E ratio or a ____ price-book ratio

Low; low

All short sales must be done in a

Margin account

In long margin accounts, customers borrow

Money

oils and gas partnership-exploratory/wildcatting risk

Most risky

Which security is not considered to be identical in a wash sale?

Option

convertible preferred stock

Preferred stock with an option to exchange it for common stock at a specified rate. -LOWER DIVIDEND RATE

A syndicator in a limited partnership is responsible for the

Preparation of any paperwork necessary for the registration of the partnership

Limited partnerships may be sold through

Private placements or public offerings

ERISA vesting

Process of how employer contributions become property of employee -time limit

Business Development Company (BDC)

Raises money by selling securities to investors. A BDC uses the money it raises to invest in private companies, small developing businesses, and financially troubled companies. -closed ended IC -must make available significant managerial assistance

seniority of claims paid in result of a bankruptcy

Secured-mortgage bond, equipment tust certificates, collateral trust Unsecured-debenture, general creditors Subordinate Preferred stock Common stock -debt security is always over an equity security

Leveraged ETFs

Seek to deliver multiples of the performance of the index or benchmark they track. -volatility is magnified -suitability is an issue

If a open GTC order to sell 100 HFC at 50 stop. If there is a 2:1 split the order becomes sell

Sell 200 HFC at 25 stop

SLoBS over BLiSS

Sell limits and Buy Stops over Buy Limits and Sell Stops -SLoBS above current market place -BLiSS orders placed below the current market place

Class B Shares

Shares of a mutual fund that generally assesses a contingent deferred sales charge if the shares are sold within a certain period. Class B shares generally have higher 12b-1 fees than Class A shares -cdsc declines over time

HYF ^s of '31 is selling at 120, convertible at $40 how many shares would one get if they converted the bond?

$1000/$40=25 shares -conversion price is higher than price of common stock

A customer sells 100 shares of OUF stock at $40 and the stock declines to $25 what is the customer's profit?

$1500.00 The customer buys 100 shares at $25

*Jumbo CDs are insured up to

$250,000 (FDIC limit) BUT ARE NOT SECURED BY ANY BANK ASSET

yield to maturity premium formula

(annual interest - (premium/years to maturity))/average price of the bond

Special Situation Funds

- buy securities of companies that may benefit from a change within companies or the economy

investment advisory and underwriter contracts must include

-all compensation that will be paid -approved at least annually by the BOD or by majority vote if s/o if it is to be renewed after the first 2years -provides that it may be terminated at any time, without penalty by the BOD or by majority vote if the s/o on not more than 60 days written notice to the IA

tax-free (tax-exempt) bond fund and UIT

-produce income exempt from federal income tax -appropriate investment for those in a high marginal tax bracket

trust indenture

-rate covenant-promise to maintain rates sufficient to pay expenses and debt service -maintenance covenant-a promise to maintain the equipment and facility -insurance covenant-a promise to insure any facility built so bondholders can be paid off if the facility is destroyed or becomes inoperable -additional bonds tests-open-ended or closed-ended -sinking fund-money to pay off interest and principal obligations -catastrophe clause-promise to use proceeds to call bonds and repay bondholders if a facility is destroyed -flow of funds-the priority of disbursing the revenues collected -book and records covenant-requires outside audit records & financial reports -call features

CMO Risks

-rate of principal repayment varies -if interest rates fall and homeowner refinancing increases, principal is received sooner than anticipated -if interest rates rise and refinancing declines, the CMO investor may have to hold his investment longer than anticipated (extended maturity risk)

municipal bonds marketability factors

-rating -maturity -yield -call features -refunding -dollar price -coupon -block size

Rule 2111 3 suitability rules

-reasonable basis suitability -customer specific suitability -quantitative suitability

Rule g-8 and g-9, books and recordkeeping requirements

-records that need to be kept on file for their lifetime, 6 or 4 years

a RR who establishes a corporate account must establish

-the business' legal right to open the acct -limitations of owners, S/O, court or entity placed on -who will represent the business in transactions

sources of revenue for revenue bonds

-utilities -housing -transportation -education -health -industrial -sports

IRA compensation

-wages, salaries, tips -alimony -commissions & bonuses -self-employment income -nontaxable combat pay

*policy loans for VA, the insurer is never req. to loan ____________ of cash value

100

There is an open order to buy OFV at 30. If there is a 20% stock dividend the order becomes

100 OFV at 25 Only round lots are allowed on the order books. The extra 20 shares are in the account but cannot be a part of the open order For reverse splits, open orders are canceled

hedge funds maximum investors

100 or less

S corporation member limits

100 s/o

*round lot

100 shares

For a stock trading under $5 per share, a customer must maintain

100% of SMV or $2.50 per share whichever is greater

When buying options, customers must deposit

100% of the premium

for tender offers: the target company within _______ business days of the announcement must provide its s/o with the statement: (3)

10; accepting or rejecting the offer expressing no opinion of the offer that it is unable to take a position on the offer

DBL 10s of '39 what is the semiannual interest payment?

10s=nominal yield is 10% $100/2=$50

dated date

The date on which interest on a new bond issue begins to accrue.

Corporate C dividend exclusion

When corporation C owns stock in another corporation 50% of dividends received from that other corporation are excluded from the corporations income

straddle

a combination of a call and a put, each with the same exercise price and expiration date

Nasdaq

a computerized data system to provide brokers with price quotations for securities traded over the counter

Simplified Employee pension (SEP-IRA)

a tax-deferred retirement plan for small businesses and self-employed people, in which the employer makes contributions directly to employee IRA accounts

$60 to $50/share for 1000 shares LMV: $50,000.00 DR: $30,000.00 EQ: $20,000.00 account is restricted by what? customer wants to sell $10,000.00 of stock update margin chart

account is restricted at $5000.00 since LMV=$25,000.00 and the Equity=$20,000.00 LMV: $40,000.000 DR: $20,0000 EQ: $15,0000 SMA: $0 $5000 to reduce the DR and the other $5000 to the customer

accumulation units

accounting measure that represents an investor's share of ownership in the separate account -accumulation unit has a NAV -A variable annuity contract owner's interest in the separate account prior to annuitization.

authorized stock

amount of shares the corporate charter authorizes

discretionary account

an account that gives your broker the power to make trades for you

In a margin account, customers pay interest on the ________ balance.

borrowed balance (difference between the purchase price and the amount of money put in is a loan)

cash account

brokerage account in which securities are paid for in full

open-end investment company prohibited activities

cannot: -purchase any security on margin -participate on a joint basis in any trading account in securities -sell any security short -acquire more than 3% of the outstanding voting securities of another investment company

income bond/adjustment bonds pay interest only if the

corporation has enough income to meet the interest payment and if the BOD declares a payment -not suitable for income seeking investors

*investment management fee is the

cost of the management of the chosen separate account subaccounts

An investor with $5,000 to spend could purchase approximately how many shares of a mutual fund with a net asset value per share of $13.00, a sales charge of $1.00 and an underwriter's concession of $0.20? A) 352 shares B) 378 shares C) 362 shares D) 357 shares

d Mutual funds sell at NAV plus sales charge. In this case, $13.00 plus $1.00 = $14.00 public offering price (POP). Dividing $5,000 by $14 equals 357 shares. What about the underwriter's concession of $0.20? That is part of the $1.00 sales charge. On each sale, the principal underwriter of the fund earns $0.20 and the selling broker-dealer keeps the other $0.80. LO 8.c

if the issuer has not pledged to pay operating and maintenance expenses first in a revenue bond what gets paid first?

debt service is the priority expense -when a debt service is paid first, the flow of funds is called a gross revenue pledge

in times of deflation for TIPS, interest payments

decrease

pump and dump

denoting the fraudulent practice of encouraging investors to buy shares in a company in order to inflate the price artificially, and then selling one's own shares while the price is high.

for margin accounts, Regulation T states that the customer must

deposit a minimum of 50% of the transaction within 2 additional days of regular business days (S+2 or T+2 for most securities)

Preemptive rights are

designed to prevent dilution of a shareholder's ownership in the company.

if the YTM is greater than the coupon, the bond is trading at a

discount

when a DMM acts as a agent, they

execute all orders brokerage firms leave with them

A stockholder who receives rights may:

exercise to buy stock by sending the rights certificate and a check -let the rights expires & lose their value -sell the rights & profit from their market value

*options: debit spread=__________________=___________

exercise; widen

conversion ratio/rate

expresses the number of shares of stock a bond may be converted into

a dealer failing to check the market value of a security when acquiring it from another dealer or customer would be a violation of

fair pricing responsibilities -even if the dealer makes no or very little profit on the trade

sellers/writers of calls want the market price of the underlying stock to

fall

conflicts of interest with BD as FAs

firm acts both as a member of the underwriting syndicate and FA for the same issue cannot do this -municipal securities dealer may help prepare the final official statement for a new issue -cannot use confidential information to solicit purchases

close end IC capitalization is

fixed

a fixed annuity payout is

fixed throughout the annuitant's life

variable annuity, monthly payments

fluctuate up and down

Corporate Bond Funds

focus on bonds issued by high-quality firms that tend to have a low degree of default risk

limited partnership

form of partnership where one or more partners are not active in the daily running of the business, and whose liability for the partnership's debt is restricted to the amount invested in the business

*refund provisions for variable life insurance

free look for 45 days from execution of the application or 10 days from the time the owner receives the policy whichever is longer -policyholder may terminate and receive a full refund -within 2 years, the s/o can terminate and will receive a refund of the cash value + a % of sales charges -after 2 years, s/o can receive the surrender value of the policy is the cash value (no sales charges)

*income and capital gains earned from investments in any IRA account are not taxes until

funds are w/d, if qualified w/d not taxed at all in the case of a Roth IRA

Prime Brokerage Account

institution that selects 1 member firm to provide custody, trading and other services (prime broker) while other firms (executing broker) will execute most trades placed by the customer.

the premium for the life insurance policy is calculated according to the

insured health, age, and sex as well as the policy's face amount at issue

if a security has a long duration means that

interest rate risk is higher and has a greater market price movement than a shorter duration

The net asset value (NAV) of an international bond fund can be expected to increase if interest rates rise abroad. interest rates fall abroad. the U.S. dollar strengthens. the U.S. dollar weakens.

interest rates fall abroad. the U.S. dollar weakens. If interest rates fall, bond prices will rise, thus increasing the NAV of a bond portfolio. If the U.S. dollar weakens, the value of other currencies will rise. This would also increase the NAV for a portfolio of international bonds. LO 10.g

*PREFERRED STOCK IS SENSITIVE TO

interest rates just like the price of the bond

*an investor purchases shares of a mutual fund, 3 months later the fund has a long term capital gains distribution this would be taxed to the investor as

long-term capital gains

CMOs are backed by

mortgages

mutual fund benefits-choice of objectives

multiple objectives to chose from

once the rate is set for a bond it

never changes

*the earnings in excess of the cost basis are taxed at ______________ when w/d from an annuity

ordinary income

a taxable distribution from ANY retirement plan is taxed at

ordinary income -NEVER AS CAPITAL GAINS

call sellers want to be ______ the money

out of

a control relationship exists when a BD is

owned by or is under common ownership with, or owns an entity that issues securities -no problems with this relationship as long as it is disclosed

Hybrid REIT

owns commercial property and own mortgages on community property -combination of both mortgage and equity REIT

special tax bonds

paid from excise taxes (tobacco and alcohol)

If the bond has a YTM and CY that are equal, the bond is trading at?

par

a May 35 call is trading at ___ when the market price is at 40 and the premium is 5

parity

a May 35 put is trading at ___ when the market price is at 30 and the premium is 5

parity

*MBS are susceptible to this type of risk

reinvestment risk

pretax margin for income statements is calculated by

sales-COGS-operating expenses=net operating profit

Coverdell ESA non-qualified w/d

same as 529 earnings are subject to income taxes and have a 10% federal penalty

marginable

securities that can be used as collateral in a margin account

Keogh plans are for people who are

self employed

growth/value fund

stocks of companies who businesses are growing rapidly -tend to reinvest all or most of their profits R&D rather than pay a dividend -focused on capital gains

call BE

strike price+premium

put BE

strike price-premium

at the money

strike=market price

*any exchange of funds is considered a sale for

tax purposes, any gains or losses are fully reportable at the time of the exchange

REIT taxation to the s/o

taxed only on dividends paid by REIT and on gains upon the disposition of REIT shares

with hedge funds being under 100 investors, what are they avoiding?

they are avoiding having to register with the SEC

straddle maximum gain

unlimited because the potential gain on a long call is unlimited

two types of annuities

-fixed -variable

annuity payout options in order from largest monthly payout to smallest is

-life annuity aka straight life or life only -life annuity with period certain -joint life with last survivor annuity -unit refund option

mutual funds risks

-market risk -interest rate risk -net redemptions -expense risk -tenure risk

pricing of close end IC shares

-supply and demand determines the bid & ask price -typically trade at a premium or discount to the share's NAV

Planned Amortization Class (PAC)

-targeted maturity date -retired first and offer protection from prepayment risk and extension risk

RMDs after the first must be w/d by

12/31

within ____ days of account approval for an options account, the firm must obtain from the customer

15; a signed written agreement

60 day rollover the AO must withhold

20% of withhold amount

AGH closes at 35.00 the following date is the ex-dividend date for $0.31 cash dividend. AGH should open at

34.69

Buying power for day traders

4x the maintenance margin excess

*what is the current yield of a 6% bond trading for 120?

60/1200=5% bond is trading at a premium

*what is the current yield of a 6% bond trading for 80?

60/800=7.5% bond is trading at a discount

*ABC authorized 1,000,000 shares of common stock, issued 800,000 shares and purchased 200,000 for treasury stock. How many shares are outstanding?

600,000 outstanding shares

FINRA Rule 2341: maximum sales charge of POP of open end IC?

8.5%

There are four corporate characteristics, more than two of which must not be in evidence for a direct participation program to avoid corporate taxation. Which of the four is virtually impossible to avoid? A) Centralized management B) Free transferability of interest C) Continuity of life D) Limited liability

A In connection with a DPP rollup, member firms may not solicit votes from limited partners unless the compensation is 2% or less. The 10% limitation is the maximum compensation in the sale of a DPP. The 15% limitation is the maximum percentage of the gross proceeds of a DPP that may be used for the organization and offering expenses. The 5% is likely an attempt to make you think about the FINRA 5% markup policy. That does not apply to DPPs. LO 11.h

High short interest means

A bullish indicator

Sole Proprietorship

A business owned by one person

Which of the following orders may be used to acquire a security at a specific price or better? A buy stop limit A buy limit A sell stop A sell limit

A buy stop limit A buy limit Only buy orders can acquire stock. Only buy limit orders can acquire stock at a specific price or better. LO 16.a

Qualified Legal Opinion

A conditional affirmation of the legal issuance of a municipal bond issue.

annuity

A contract between an individual and an insurance company where the individual makes a series of payments that are invested by the company and repaid to the individual at a later date, generally during retirement.

foreign currency option

A contract that provides the right to buy or sell a given amount of currency at a fixed exchange rate on or before the maturity date

duration

A measure of the interest rate sensitivity of a bond.

Beta

A measure of the volatility, or systematic risk, of a security or a portfolio in comparison to the market as a whole.

power of attorney

A written document, which is usually notarized, authorizing another to act as one's agent;

Tender offers must be open for how many business days from the first publication of the offer? A) 20 B) 5 C) 10 D) 15

A) 20 If the terms are changed, the offer must be open for at least 10 business days from the date of change but never for less the original 20 business days. LO 14.b

Which of the following do not have a fixed maturity rate? A. CMO B. GO bond C. STRIPS D. Subordinate debentures

A. CMOs CMOs are mortgage backed securities because mortgages are often paid ahead of the scheduled maturity the next exact date of CMOs is uncertain

Current Yield Formula

Annual dividend per common share/ market value per common share

A UK company exports sweaters to the U.S. and will be paid in U.S. dollars upon delivery. To hedge foreign-exchange risk using listed currency options, the UK company should A) sell British pound calls. B) buy British pound calls. C) buy British pound puts. D) sell British pound puts.

B Normally, exporters buy puts on foreign currency to hedge. There are no listed currency options available on the U.S. dollar, so the British company should buy calls on its own currency. LO 10.g

An investor who buys a stock and wishes to limit the potential downside risk should A) enter a sell limit order. B) buy a put. C) buy a call. D) enter a buy stop order.

B The purchase of a put limits the downside risk to the difference between the stock price and the put's strike price. LO 10.d

An investor is always purchasing newly issued shares of common stock when investing in A. closed end IC B. open end IC C. UIT D. a holding company

B open ended companies only issue common stock

Under the USA PATRIOT Act of 2001, member firms must maintain records of reports of currency transactions involving more than $10,000 for A) 3 years. B) 5 years. C) 6 years. D) 1 year.

B) 5 years. The USA PATRIOT Act of 2001 requires that all currency transactions involving more than $10,000 be reported on a Form 112 Currency Transaction Report and that these forms be maintained for 5 years. LO 15.f

Credit call spread is bullish or bearish?

Bear Spread

A downward trendline is

Bearish

Long stock is

Bearish a

But limit orders are placed

Below the current market

Collateral Trust Bonds

Bonds backed by financial assets that are readily liquid

Technical- support level

Bottom of the trading range

One of your clients has appointed his daughter as the trusted contact person per FINRA Rule 2165. She contacts you to explain that her father's cognitive abilities are declining. Because of that, before it gets too late, she wants to know what can be done to give her control over the account. It is likely that the best suggestion would be to have her father sign A) the discretionary power authorization. B) a limited power of attorney. C) a durable power of attorney. D) a full power of attorney.

C A durable power of attorney is used when the account owner has diminished physical or mental capacity. It is durable because it survives the client becoming legally incompetent (but does not survive death). LO 1.d

A method of analyzing limited partnerships by identifying the sources of revenues and expenses is known as A) liquidity analysis. B) capital analysis. C) cash flow analysis. D) technical analysis.

C Cash flow analysis compares income (revenues) to expenses. LO 11.g

Which of the following would not be found within the protective covenants for a municipal revenue bond issue? A) Flow of funds B) Catastrophe clause C) The issue's rating D) Call features

C There are different sources for bond ratings, but they would not be found within the revenue issue's protective covenants. The municipality agrees to abide by the covenants, and a trustee appointed in the bond indenture supervises the issuer's compliance with them. Some common covenants include rate or fee (promise to maintain user fees high enough to pay expense and debt service) maintenance, insurance, additional bonds test, sinking fund, catastrophe, flow of funds, books and records, and call or put features. LO 6.b

Your client's investment portfolio is 50% growth stocks, 10% foreign stocks, and 40% blue-chip stocks. If the client is interested in further diversification, which mutual fund would best meet that goal? A) Global equity fund B) Aggressive growth fund C) Bond fund D) Emerging market fund

C) Bond fund All of the current holdings are equities. To further diversify the current portfolio, the bond fund would be the best choice of those given to meet this objective. LO 14.a

Which of the following order types is permitted in Nasdaq markets but not in NYSE equity markets? A) Limit B) Immediate-or-cancel (IOC) C) Fill-or-kill (FOK) D) Market

C) Fill-or-kill (FOK) FOK and all-or-none orders may no longer be entered in the NYSE equity market but are still accepted in both the bond market and Nasdaq. LO 16.a

Market timing is normally associated with which of the following portfolio management styles? A) Modern portfolio theory B) Passive management C) Tactical asset allocation D) Strategic asset allocation

C) Tactical asset allocation Tactical asset allocation, which attempts to capitalize on short-term market swings, is a market timing strategy. LO 14.a

All of the following will be included on a confirmation for noncallable municipal bonds purchased on a yield basis except A) yield to maturity. B) dollar price. C) taxable equivalent yield. D) par value.

C) taxable equivalent yield. As investors' tax brackets vary, taxable equivalent yield is never required to be shown. LO 15.a

which is most often organized as a limited partnership? a. face amount company b. ETF c. hedge fund d. unit investment trust

C. hedge fund

Information about which of the following is available on EMMA but not RTRS? A. auction rate securities B. general obligation bonds (GO bonds) C. municipal fund securities D. revenue bonds

C. municipal fund securities (section 529 plans are included in EMMA but not RTRS

*Inheritance taxes

Clients cost basis for determining if there is taxable capital gains is the fmv of the date of death

Which of the following must be registered as investment companies under the Investment Company Act of 1940? Closed-end investment companies Separate accounts of insurance companies offering variable products Variable annuity contracts Variable life insurance policies

Closed-end investment companies Separate accounts of insurance companies offering variable products

Roth Conversion

Converting a traditional IRA to a Roth IRA

EPS other formula

Current market price of common stock / p/e ratio

Purchase of 1000 shares of THV for $20. Cash purchase

Customer pays $2000

Which of the following would establish a covered put? A) Long stock at 40, short put at 35 B) Short stock at 40, long put at 45 C) Long stock at 40, long put at 45 D) Short stock at 40, short put at 35

D A covered put is created when a short stock is combined with a short put. Covered puts are established when the investor is neutral or slightly bearish; therefore, the strike price of the put is less than the cost of the stock sold short. The reason the put writer is covered (protected) is that if the stock's price should decline below 35 and the holder of the option exercises, the stock purchased by the writer is used to cover (replace) the borrowed stock for the short sale at 40. LO 10.d

A customer opens a margin account with a broker-dealer and signs a loan consent agreement. The loan consent agreement allows the firm to A) hypothecate securities in the account. B) commingle the customer's securities with securities owned by the firm. C) lend the customer money. D) loan out the customer's margin securities.

D A signed loan consent agreement permits a firm to loan out a customer's margin securities; this is considered another way to finance a customer's debit balance. LO 2.g

Which of the following is true regarding a 5-for-4 stock split? A) Retained earnings will be increased. B) The par value will be unchanged. C) The net worth of the company will be reduced. D) Each shareholder's proportionate equity will be unchanged.

D Because each shareholder will receive additional stock, the proportional equity will remain the same. LO 3.b

XYZ Corporation has a market price of $45 per share and earnings per share (EPS) of $3 when XYZ announces a 3-for-1 split. After the split, the price-to-earnings (P/E) ratio of XYZ will be A) 45. B) 5. C) 3. D) 15.

D Before the split, the company had a P/E ratio of 15 ($45 per share / $3). After the split, the price per share and the EPS drop in the same proportion, leaving the P/E ratio unchanged (new price = $15, new EPS = $1). LO 13.d

Under what circumstances will a dilution of equity occur? A) Stock split B) Issue of mortgage bonds to replace debentures C) Stock dividend D) The conversion of convertible bonds into common stocks

D Dilution of equity occurs when stockholders experience a reduction in their percentage ownership of the company. If bonds are converted, more common shares are issued, and the shareholder's equity is diluted. A stock dividend or stock split does not change a stockholder's percentage of ownership. Refunding debts has no effect on stockholders. LO 5.c

Which of the following investments is most likely to have extension risk? A) A mortgage bond B) A hedge fund C) A zero-coupon bond D) A CMO

D Extension risk is the uncertainty that a debt will be paid off ahead of schedule. This happens most frequently with mortgage-backed securities, such as CMOs. A CMO's yield and maturity are estimates based on historical data or projections of mortgage prepayments from the Public Securities Association (PSA). When that estimate is incorrect and the mortgage prepayments decline (such as in a period of rising interest rates), yields turn out to be lower than projected. Bonds do not have their maturity dates extended, so there is no extension risk. LO 12.d

An investor purchases a zero coupon bond at a price of 64. The bond matures in nine years. Five years later, the investor sells the bond at a price of 80. This would result in A) no gain and no loss. B) a long-term capital gain of $160. C) a long-term capital loss of $200. D) a long-term capital loss of $40.

D This question deals with accretion of the discount. The discount here is $360 (the difference between the $640 paid and the $1,000 maturity value). With nine years until maturity, the annual accretion is $360 divided by nine, or $40 per year. After five years, the bond's basis has increased by $200 ($40 times 5 years) to $840. The sale at $800 represents a long-term loss of $40. 840-800=40 LO 5.e

If a customer buys 1 OEX Feb 350 call at 5, then sells 1 OEX Feb 335 call at 16 when the underlying index is at 344, the breakeven point is A) $340. B) $342. C) $339. D) $346.

D To determine a call spread, add the net premium to the lower strike price to find the breakeven point. The net premium is the difference between the premium paid (5) and the premium received (16), or 11 (335 + 11 = a breakeven point of 346). LO 10.h

A summary statement of all interest and dividends credited to a customer's account must be sent to the primary accountholder each year in A) April. B) July. C) December. D) January.

D) January. Member firms must provide an IRS Form 1099 to the primary account holder of all the interest and dividends credited to the account in January. This form is used in the customer's tax return preparation. LO 15.b

A portfolio manager using index options is trying to reduce which of the following types of risks? A) Selection B) Financial C) Purchasing power D) Systematic

D) Systematic Systematic risk refers to the impact the overall market has on an equity portfolio's value. Index options help insure portfolios against systematic risk. The purchase of index puts to protect a portfolio is called portfolio insurance. LO 14.a

The market-wide circuit breaker (MWCB) rule uses which of the following as the pricing reference point to measure a market decline? A) The S&P 100 index recalculated monthly B) The Wilshire index recalculated daily C) The DJIA recalculated quarterly D) The S&P 500 index recalculated daily

D) The S&P 500 index recalculated daily The MWCB rule uses the S&P 500 recalculated daily as the pricing reference point to measure market declines for the purpose of triggering market circuit breakers to halt trading. LO 16.e

A fundamental analyst researching a stock is concerned with all of the following except A) the stock's market price as a multiple of the company's earnings. B) the capitalization ratio. C) management efficiency. D) the volume of shares traded.

D) the volume of shares traded. A fundamental analyst is concerned with the economic climate, the inflation rate, how an industry is performing, a company's historical earnings trends, how it is capitalized, and its product lines, management, and financial statement ratios, such as the P/E ratio. A technical analyst is concerned with trading volumes or market trends and prices. LO 13.e

Certain orders on DMM order books are reduced when a stock goes

Ex-dividend

The first date on which the owner does NOT QUALIFY for the current dividend is the

Ex-dividend date

Immediate or Cancel (IOC)

Execute any portion available immediately Cancel any balance remaining

T or F: Treasury receipts are backed by the full faith & credit of the gov.

F

Who can buy and sell property for the partnership: GP OR LP

Gp

The more risk a BD assumes the ________ the justification for higher markups

Greater

Transactions of relatively small dollar amounts generated warrant ________ % markups than large dollar transactions

Higher

trustee-to-trustee rollover

IRA custodian to a IRA custodian -AO never takes ownership of the funds -unlimited amount of transfers -avoids 20% withholding

*when will securities not have the last sale information readily available?

If they are identified as OTC non-Nasdaq since they are thinly traded securities

Unit refund option

If value remains in the account after the death of the annuitant, it is payable in a lump sum to the annuitants beneficiary. This option may be added as a rider to one of the others. -only option that guarantees all the money in the contract to be distributed -smallest check

oils and gas partnership-income advantages

Immediate tax flow

oils and gas partnership-income tax features

Income sheltering from depletion allowances

whole life insurance

Insurance that is kept in force for the participant when they turn 100 and pays a benefit upon the person's death, whenever that may be. -not a security

Auction Rate Securities (ARS)

Issued by municipalities, nonprofit hospitals, utilities, housing finance agencies, and universities, auction rate securities are long-term variable rate bonds tied to short-term interest rates.

lump sum w/d from annuities are taken by

LIFO (earnings are removed before contributions)

random withdrawals from annuity contracts are taxes under

LIFO (earnings are removed before contributions) as ordinary income -with nonqualified annuities, it is only the earnings that are taxed

market value of the securities falls from $50,000 to $36,000 adjust the margin chart and find regulation T and margin maintenance

LMV=$36,000 DR=$30000 EQ=$6000 Regulation t=18000 (.5x36000) Minimum maintenance req=9000 (.25x36000) This account is subject to an account maintenance call

*Can SMA be used to meet a maintenance margin call?

No

NAV=$10 sales charge % is 4.8% calculate POP

POP=$10.50 $10/(100%-4.8%)=$10.50

All orders entered below the market are ———- on the ex-dividend date

Reduces

From first to last, which of the following sequences reflects the priority of payments made when a limited partnership is liquidated? General partners Limited partners General creditors Secured creditors

Secured creditors General creditors Limited partners General partners Creditors are paid first in a liquidation, with priority given to the secured lenders; general partners are the last to be paid. LO 11.d

adjustable-rate preferred

Some preferred stocks are issued with adjustable, or variable, dividend rates. Such dividends are usually tied to the rates of other interest rate benchmarks, such as Treasury bill and money market rates, and can be adjusted usually quarterly -low interest rate risk

Consolidated stock price

Stock price is in a narrow range

T or F: margin accounts aren't allowed in a IRA

T

*T or F: a 401k is not considered a payroll deduction plan

T -considered a salary reduction plan NOT a payroll

Regular Way Settlement

T +2, settlement date plus 2 business days (corporate securities, municipals, and gov. agencies) T + 1 for Treasury securities Same day as money market securities

FCO settlement

T+1

Option Settlement

T+1

interest rate options settlement-

T+1 -IN CASH

*settlement date of an option that is exercised

T+2 T+1 if expires or bought or sold

direct rollover

Take the company's 401(k) plan and flip it over into your own IRA account

intrinsic value of an option

The amount the option is in the money -OPTIONS NEVER HAVE A NEGATIVE INTRINSIC VALUE

Annual Gift Tax Exclusion

The annual amount that a donor can give to each donee that is excluded from taxable gifts $15K

Good Delivery

The delivery of securities in good deliverable form is expected on the settlement date

Spread

The difference between the bid and ask

Alpha

The extent to which an asset's or portfolio's return exceeds or falls short of it's expected return

legal opinion

The written opinion of bond counsel that attests to the fact that the municipal issue was legally authorized and issued, and to the tax status of the interest

does a UTMA or UGMA expand the types of property you can gift to a minor?

UTMA

bearish breakout

a decline through the support level

Depreciation

a lessening in value over time for assets

for variable annuities what is required prior to or concurrent with the sale?

a prospectus

only a change in the AO name can only be authorized by

a qualified, registered principal designated by the member

grey market

a security not currently traded on any of the OTC quoting marketplace or any other US quotation mediums

*all contributions to annuities are made with

after-tax dollars

joint account

an account held by two or more people

Order Audit Trail System (OATS)

an automated computer system created to record information relating to orders, quotes, and other trade information from all equities that are traded on Nasdaq

OTC market (over the counter)

an electronic marketplace for stocks and bonds who are not listed trade -aka pink sheets

day trader

an individual who buys and then later sells stocks and other securities in the same day to take advantage of intraday price movements

Dividend payout ratio

annual dividend per share / earnings per share

*you would only recommend municipal bonds to an investor if they

are in a higher tax bracket

secured debt security

assets are put up as collateral for the loan

Net Asset Value (NAV)

assets minus liabilities expressed on a per-share basis

what level of trading is appropriate for a fee based account?

at least a moderate level of trading

BABs are issued

at par

a bond always matures

at par

for employee stock purchase plans, when does the company buy the stock?

at the end of a 6mo. period (discounted stock price)

what does ACATS do?

automates and standardizes the procedure for the transfer

*debt coverage ratio

available revenues/debt service requirements -the higher the ratio the better

A customer has a long margin account with a market value of $30,000 and a debit balance of $20,000. His short margin account has a $7,000 market value and a $10,000 credit balance. The FRB margin requirement is 50%. What is the minimum equity requirement for the short position? a. 0 b. $2,100 c. $9,000 d. $11,000

b. $2100 The SRO minimum maintenance requirement for a short position is 30% of the market value. The market value is $7,000, and 30% of $7,000 equals $2,100.

guaranteed bonds

backed by a company other than the issuer -the value of the guarentee is only as good as the strength of the company making that guarentee

appraised value of DPP or REIT

based on valuations of the assets and liabilities of the DPP or REIT performed at least annually -derived from methodology

the buyer of the option wants the contract to

be exercised

Short stock is

bearish

ERISA beneficiary

beneficiaries must be named to receive payment at AO's death

a market maker in DGVV common stock is quoting 52.15 -- 52-27, 4X6 this means

bid=52.15 ask=52.27 dealer is willing to buy 400 shares and sell 600 shares

limited tax GOs

bond secured by a specific tax

current refunding-municipal bond

bonds will be redeemed within 90 days or less from the date of issuance of refunding bonds -increased the quality of the bond, thus increasing marketability

*to determine whether a spread is bullish or bearish, if you are buying at lower strike price you are

bullish BLSH buy low, sell high

which risk is high if an investor only holds stock in one issuer or in lower rated bonds?

business risk

when a DMM acts as a principal, they

buy & sell in their own accounts to make markets in assigned stock -MAY NOT buy stock for their own account at a price that would compete with the current market

if an investor thinks the value of the currency is going to rise, they should

buy calls or sell puts on the currency

Bear Spread

buy high, sell low

Bull Spread

buy low, sell high

mutual fund benefits-convenience

buying, liquidating and doing exchanges can be accomplished by going to a fund's website, mail or by calling

how is a debt security acquired?

by buying a issuer's bond

investment advisory and underwriter contacts must be approved

by majority vote of the shareholders

sell a call and it expires will result in a

capital gain

sell a put and it expires will result

capital gain

Excessive buying and selling of securities to generate commissions when it does not meet the customer's investment goals is called

churning

two types of liabilities

current and long term

Fixed-Time Withdrawal Plan

customers liquidate their holdings over a fixed period

In a direct participation program (DPP) limited partnership, the general partner has A) limited liability and a passive role. B) limited liability and an active role. C) unlimited liability and a passive role. D) unlimited liability and an active role.

d In a DPP limited partnership, the general partner is the active partner managing the business who assumes unlimited liability. Limited partners who take an active role jeopardize their limited liability status. LO 11.d

for a variable annuity, the rate of return is

dependent on the separate account performance

mutual fund risk-net redemption

during declining markets, there is an excess of s/o redemption over new share purchases known as net redemption -fund with a high net redemption is going to deliver s/o poor performance

statutory voting

each share held is assigned one vote in the election of each member of the board of directors.

who contributes to a contributory plan?

employee and employer contributes page 37

the seller of the option wants the contract to

expire

Negotiable Certificates of Deposit (CDs)/jumbo CDs

fixed-maturity interest-bearing deposits with face values of $100,000 or more that can be resold in the secondary market with no prepayment penalty

general partnership income or losses are

flown through to the investors -does avoid double taxation

mutual funds may be bought in

full or fractional shares

tax deferred

income that is not subject to taxes immediately but that will later be subject to taxes.

Municipal Notes

maturity under 5 years or less

GNMA denomination

minimum: $1000 and multiples of $1

debt capital

money borrowed from a business or investor that must be repaid over time with interest

annuitized payouts are typically made out ____________ and are taxed according to an ________________

monthly; exclusion ratio

purchasing a bond at a discount will always result in getting back par which means

more (a profit) than the original investment

do zero coupon bonds pay any interest payments?

nah baby nah

which risk can be diversified away? systematic or nonsystematic

nonsystematic

* is the exercise of an option a taxable event?

nope

fixed annuities are they securities?

nope

*investments made in a money market fund insurance

not insured or guaranteed by the FDIC or any other government agency -it is possible to lose money in a MMF

*Are dividends from a REIT qualified or non-qualified and how are they taxed?

not qualified, taxed as ordinary income

Current liabilities include

notes payable accounts payable accrued wages payable Current long term debt Accrued taxes

*the tax-equivalent yield for a municipal bond issued by an entity within a state with a state income tax will have a higher-tax equivalent yield to a resident of that state because

of the double tax exemption

*closed ended funds computes NAV

on a weekly basis

interest on a corporate bond is taxable

on all levels

interest on issues of the federal government issues is taxed

on the federal level

Index Options expire ______.

on the third Friday of the expiration month

*if the LI is not completed, the sales charge amount that applies is based

on the total amount invested -share appreciation and income paid by the fund do not count toward completion of the letter

mutual fund benefits-minimal initial investment

once you are a s/o, most funds permit additional investments of $100 or even less

Pattern Day Trader

one who executes four or more day trades in a five business day period

pattern day trader

one who executes four or more day trades in a five business day period -minimum requirement: $25k in the day of trade

Special Assessment Bonds

only assess property owners who benefit from the bond issue -sidewalk, sewers, etc.

Interest Rate Options

options based on the price or yield of U.S. Treasury securities strike prices reflect the yield

special document that is provided to the AO when opening an options account

options disclosure document

put sellers want to be ______ the money

out of

*T Notes Pricing

percentage of par -1/32% -book entry

*if a bond had a 5% coupon, but the current rate was 4% the bond is selling at a discount, par, or a premium?

premium

short call maximum gain

premium

trading at parity

premium equals the intrinsic value of the contract

CEF risk-pricing risk

price of close end shares is determined by supply and demand -s/o might purchase at NAV

Forward Pricing

pricing method used by mutual funds based on the next price to be computed

a hedge fund is offered by

private placement memorandum

Nasdaq level 3

provides subscribers with all of the services of levels 1 and 2 and allows registered market makers to input and update their quotes on any securities in which they make a market

fixed annuity has this risk

purchasing power risk

no load fund is permitted to charge

redemption fees

Tax swap

replacement of a bond that has a capital loss for a similar security; used to offset a gain generated in another part of an investor's portfolio Used to control tax liability, extend maturity or update investment objectives Has to have either a different maturity, couponing issuer

tax treatments for LEAPS, LEAP writers must

report short-term capital gains at expiration EVEN IF THEY HAVE HELD THE CONTRACT FOR OVER 12 MO

A DMM is someone who

represents a member firm of the NYSE

Revenue bonds are backed by ___________________ generated by ________________________.

revenues; municipal facility

before opening a margin account, the BD must provide the client with a

risk disclosure, needs to be provided annually thereafter

accelerated depreciation

schedule that spreads depreciation over fewer years to generate larger tax reductions at the beginning

"I want to sell my FHH stock if it falls to $30, but I don't want less than $29.95 for my shares." how would this be placed?

sell 1000 shares FHH 30 stop 29.95 once FHH falls at or below 30, the order becomes a limit order to sell at 29.95 or better.

Short Straddle

short call and short put

Brokered CDs

sold by broker/dealers directly to customers broker/dealer buys a master CD from a bank then subdivides the CD into smaller pieces for resale to customers -commissions or fees to buy brokered CDs are generally higher than bank-issued ones call risk -usually not redeemable before maturity -FDIC insurance may not apply

employee stock purchase plan

some corporation allow employees to put certain money amount of paycheck to buy company stock and at some point match that amount with shares of stock

fiduciary

someone who has the legal authority to act on another's behalf

Roth 401(k) RMD

starts at 72

REIT prices are determined by

supply and demand -do compute NAV but it is an approximation

*which asset allocation would be appropriate for a fee based structure?

tactical asset allocation

owners in a joint account are called

tenants

Issued stock refers to the number of shares:

that have been sold to investors -Outstanding plus treasury shares.

close-end investment company registers its offering with

the SEC

capital in excess of par

the amount of contributed capital less the par value of the stock

what happens to an investor who sells a bond before the semiannual interest payment date?

the buyer pays the seller the amount of interest that has accrued since the last payment. Then on the next payment date, the new owner received the full 6 months of interest

taxation of premiums at expiration for option contracts

the buyer reports it as a capital loss while the seller reports it as capital gains=premium

Equity (EQ)

the customer's net worth in the margin account; it represents the portion of the securities the customer fully owns

if a party in TIC dies, the decedent's interest in the account goes to

the decendent's estate

Credit Call Spread maximum loss

the difference between strike prices - net premium

accretion

the difference between the discounted purchase price and the full face value at maturity is the return the investor receives

repurchase agreement profit

the difference between the sale price and the repurchase price is the interest earned by the investor

Internal Rate of Return (IRR)

the discount rate that makes the future value of an investment equal to its present value

Collateralized mortgage obligation (CMO) tranche A has been created to have the most predictable near-term principal pay off. A tranche set up in this way will have the highest reinvestment risk. the least reinvestment risk. a higher yield. a lower yield.

the least reinvestment risk. a lower yield. A CMO created to have the most predictable near-term principal pay off will have lower reinvestment risk and lower yield than other CMOs. LO 12.d

municipal bond marketability-call features

the longer the call protection, the more marketable the bond

the lower the current ratio,

the lower the company's liquidity

margin

the minimum amount of equity a customer

Assumed Interest Rate (AIR)

the minimum rate of return necessary to provide the level death benefit

for a variable annuity, payments are invested in

the separate account

Roth 401(k) income limit

there is non unlike the Roth IRA but income limits still apply to Roth IRA

money market indenture

there is none

all qualified corporate plans must be established under a

trust agreement

Small-Cap Fund

under $2B

*an owner of REITs hold

undivided interest in a pool of real estate investments

long call maximum gain

unlimited

open end IC capitalization is

unlimited

Backing Away

violation in which a market maker fails to honor a published firm quote to buy or sell a security at a stated price

ETF risk-tracking risk

virtually impossible to have performance equal to the index -all expenses equal, you most likely want to recommend the ETF with the highest tracking reliability

if an investor buys a long straddle they think the stock price

will be volatile

if an investor buys a short straddle they think the stock price

will not change or will change very little

after the order ticket is prepared, it is sent to the

wire room or trading desk where the order is routed to the proper market for execution

high-yield bonds (junk bonds)

with ratings lower than Baa, bonds that have a substantial probability of default -higher risk, but higher profit

private placement debt

-money loaned on a private basis -is not registered with any regulatory body -high risk

To make a delivery, the broker dealer

Lends the security to its customer

T of F: dollar bonds are term bonds

T

municipal bond marketability-maturity

the shorter the maturity, the more marketable

counteroffer or counterbids are common when

the spread between the market maker's bid and ask is fairly wide

all investment companies must file annual financial reports with

the SEC

open-end investment company registers its offering with

the SEC

amount that is short of their RMD is taxed

50%

T of F: dollar bonds are serial bonds

F

customer balance sheet

-cash -securities -home -car -collectibles -IRA -life insurance -mortgage -credit card debt

3 documents for margin accounts

-credit agreement -hypothecation agreement -loan consent

*SMA has a buying power of

2:1

Not held order

An order that gives the floor broker discretion as to time and price

Exchange securities are bought and sold on an

Auction market

Market value at maintenance formula

DR / .75

all growth in the value of an accumulation unit is

DEFERRED UNTIL W/D

Exchange markets are also sometimes called

Double auction markets

Most common sharing arrangement for oil and gas

Functional allocation

*nominal quotes can be given for informational purposes and can be printed only

If labeled as such

*what is the one foreign currency option that does not have a contract size of 10,000?

Japanese Yen (1 million)

*ALL DIVIDENDS ARE PAID

QUARTERLY

Overnight Repo

Repurchase agreement for one day

which plan has eligibility requirements?

Roth IRA

safest money market security

US treasury bill

*the less active a security, the ______the spread

Wider

gifts in a UGMA/UTMA are

irrevocable

mutual fund benefits-liquidity

the investment company act of 1940 requires than an open-end IC stand ready to redeem shares at the next computed NAV per share. -payments must be made w/in 7 days of the redemption request -can be a redemption charge -liquidity is assured

when interest rate fall, why call?

the issuer could take advantage of lower cost of borrowing by issuing new bonds at the lower interest rates prevailing and use the proceeds to call in the bonds with higher coupons

Big Board

the large display board at the New York Stock Exchange that reports on stocks traded on the exchange

Risks of limit orders

the market may never go as low as the buy limit price OR as high as the sell limit price SOMETIMES, even if the stock trades at the limit price it will not execute due to STOCK AHEAD stock ahead=limit orders the DMM's book for the same price are arranged according to when they were received

debt limit

the maximum amount of gross or net debt that is legally permitted for a municipality

market breadth

the number of issues closing up or down on a specific day -when declines outnumber by a large amount the market is bearish -bull market: advances substantially declines

short interest

the number of shares sold short

in a defined contribution plan, who assumes the investment risk?

the plan participant

in a defines benefit plan, who assumes the investment risk?

the plan sponsor

asset allocation

the process of spreading your assets among several different types of investments to lessen risk

Reverse Repo

the purchase of securities by one party with an agreement to sell them -purchaser initiates the deal

Arbitrage

the purchase of securities in one market for immediate resale in another to profit from a price discrepancy

cash settlement

the same day

*closed-end IC trade in the

the secondary market which is why the price is based on supply & demand -the market price is independent of the fund's NAV

buying securities on margin

the securities firm lends the necessary funds at the time of purchase with the securities in the portfolio serving as collateral for the loan

If a customer buys 1 OEX Feb 350 call at 5 and sells 1 OEX Feb 335 call at 16 when the underlying index is at 344, she will profit if the spread narrows. the spread widens. the underlying index does not change. the underlying index rises in value.

the spread narrows. the underlying index does not change. Explanation This is a credit spread because the investor received more premium than was paid. Sellers profit if both contracts expire or the spread narrows. The breakeven point is 346 (335 + 11, the net premium), and because the spread is bearish, the customer profits if the index is below 346. LO 10.e

If XYZ is trading at 39, and a customer sells 1 XYZ Jun 40 put and buys 1 XYZ Jun 35 put, he will profit if the spread widens. the spread narrows. the contracts expire. the contracts are exercised.

the spread narrows. the contracts expire. When a spread's premiums are not available, the more valuable option is found by examining the strike price. A put with a higher strike price has a higher premium because a put represents the right to sell. Because the investor is selling the more valuable put (the one with the higher strike price), this is a credit spread, and profit occurs if the options expire worthless (in this case, the customer keeps the net credit) or the spread narrows between the premiums. LO 10.f

total debt

the sum of all bonds issued by the municipality

UIT shares (units) are redeemed by

the trust

indenture

the written agreement between the corporation and the lender detailing the terms of the debt issue

zero coupon bond duration

their duration is always equal to the length of maturity coupon

negative response letter informs the recipient of the letter that

there is an impeding action and requires the recipient to respond or act w/in a specific time frame -no response=action will take place

time decay of an option

time value will diminish as an option's expiration date comes near -last day before expiration, no more time value & premium=intrinsic value

net worth

total assets minus total liabilities

revenues on the income statement is the

total sales during the period

Shareholders' Equity

total value of assets - total value of liabilities

Close end funds pricing

trade at some distance from their NAV -independent of their NAV

Stop order

trade is not to be executed unless stock hits a price limit then becomes a market order

ETF pricing

trade very close to NAV

standard ETF trading

trades like a stock on an exchange or NASDAQ -can take advantage of price changes due to market flucuation

while the ROP or branch office manager reviews and does a final approval of the new option account application, this can occur

trading can occur

which securities do not have default risk?

treasury securities

buy stop-limit at 52 entered when the market is 51 when would the price be executed? 51.88 51.99 52.13 52.13 51.88

trigger at 52.13 execute at 51.88

buy stop at 52 entered when the market is 51 when would the price be executed? 51.88 51.99 52.13 52.13 51.88

trigger is at 52.13 executed at 52.13

A corporate profit-sharing plan must be set up under A) a beneficial ownership. B) an administrator. C) a trust. D) a conservatorship.

trust

when a company's current s/o doesn't exercise their preemptive rights in an additional offering, a corporation has an

underwriter standing by to purchase whatever shares remain unsold as the result of rights expiring

short call maximum loss

unlimited

*naked call writers face

unlimited potential risk of loss

*selling stock short has

unlimited potential risk of loss

debentures

unsecured bonds backed only by the credit worthiness of the bond issuer -written promises of the corporation to pay the principal at its due date and interest on a regular basis

GOs require

voter approval

convertible debt securities

when a company states that lenders (the investors) may exchange (convert) the debt into shares of the company's common stock

UIT risk-interest rate risk

when interest rates are rising, the fixed position creates a difficult position for investors since in a bond MF, the cont. inflow of new $$$ allow the fund managers to purchase the new higher yield in a bond, cant be done in a UIT

UIT benefit-liquidity

while not traded in the secondary market, the trustees are obligated to redeem units tendered by the investors at the NAV

reinvestment of divideds must be offered back into the fund

without sales charges (at NAV)

if a partner dies in a partnership account, the member needs

written authority from the remaining partners before executing any further orders (usually an amended partnership agreement)

do IRA's need a ben?

yea

an amended partnership is needed every _____ if changes are made

year

can a employer take a income tax deduction for contributions made in SEP-IRAs

yes

can employers skip contributions in a profit sharing plan?

yes, popular because of this

Roth 401(k) plan

you make contributions with after-tax dollars, and qualified distributions at retirement are received income-tax free -same rules as a Roth IRA

options that are out or at the money have an intrinsic value of

zero

the long term expected value of ETN is

zero

*which instruments are not subject to reinvestment risk?

zero coupon bond

what type of bond avoid reinvestment risk?

zero coupon bonds, nothing to reinvest

US Treasury Bills

•typically do not have regular coupon payments •have maturities of 4, 8, 13, 26, 52 weeks •have very little default risk. •have very little interest rate risk

*if an investor receives a lump-sum withdrawal from an annuity before 59.5, taxes will be

the earnings portion is taxed as ordinary income and is subject to an additional 10% tax penalty unless death or disability

REIT are bought on

the exchange or in the OTC market

Conversion Price

the face value ($1000) of a convertible bond divided by the number of shares received if the bond is converted -conversion price is higher than price of common stock

interest earned on GNMA, FHLMC, and FNMA certificated is taxable at

the federal, state, and local levels

margin documents need to be signed before

the first trade is made

*STRIPS are backed by

the full faith & credit of the US gov. -Treasury receipts are not

for a fixed annuity, payments are invested in

the general account

if the investor dies during the accumulation period the bene will receive

the greater of the current value of the account or the amount invested

the higher the current ratio

the greater the company's liquidity

joint accounts suitability is based on

the group not any one individual

municipal bond marketability-coupon

the higher the coupon, the more marketable the bond

municipal bond marketability-rating

the higher the rating, the more marketable the bond

In a sole proprietorship, the income or losses is

the individuals -they can lose everything they own

*Mortality risk covers the risk that

the insured may live for a period shorter or longer than assumed

*policy loans for variable life insurance

the insured may only borrow a % of cash value -minimum % is 75% after the policy has been in force for THREE YEARS

Voluntary Accumulation Plan

A mutual fund account into which the investor commits to depositing amounts on a regular basis in addition to the initial sum invested.

cash requirement: 100 shares at $15 FINRA & Reg T

$1500 total FINRA: $1500 Reg T: $750 NEEDED: $1500

FINRA minimum rule: 100 shares at $30

$2000

FINRA minimum rule: 100 shares at $50

$2000

What is the minimum initial dollar requirement in a short margin account?

$2000

if a customer was shorting margin accounts what is the minimum requirement needed? If the customer was shorting 100 shares at $18?

$2000 even if total price is $1800

cash requirement: 100 shares at $50 FINRA & Reg T

$5000 total FINRA: $2000 Reg T: $2500 NEEDED $2500

Uniform Practice Code (UPC)

-Guides for how FINRA members transact business with other members Regulates: -Settlement Dates -Ex-dividend dates -Rules of good delivery -Confirmations -Don't know (DK) procedures

Breakpoint Sale

-Prohibited -attempting to make higher commissions by selling shares just below the point at which a sales charge is reduced

Benefits of REITs

-Properties are selected by professionals -reasonable income &/or capital appreciation -Liquidity -negative correlation to the general stock market because real estate princes and the stock market frequently moves in opposite directions

An organization is classified as a partnership for federal tax purposes if it has two or more members and none of the following

-REIT -insurance company -corporation or incorporated -an organization classified as a trust or any other special treatment under the IRC

Dissolving a Limited Partnership settlement order

-Secured lenders -other creditors -LPs -GPs

Nonqualified deferred compensation plan

-This is an agreement between a company and an employee in which the employee agrees to defer receipt of current income in favor of payout at retirement - it is assumed that the employee will be at a lower tax bracket at retirement -(BOD members with no other affiliation are not eligible for retirement planning)

types of money market instruments

-US treasury bills -commercial paper -banker's acceptances -negotiable certificates of deposits (CD) -repurchase agreements

Treasury Receipts

-Zero coupon bond created by brokerages. -B/D buy treasury securities and place them in trust at a bank and sell separate "receipts" against the principal. *Not backed by the U.S. Government

mutual fund benefits-combination privilege

-a investor seeking a reduced sales charge may be allowed to combine separate investments in 2 or more funds within the same family of funds to reach a breakpoint

Farm Credit System (FCS)

-a national network of lending institutions that provides agricultural financing and credit -privately owned, government-sponsored enterprise that raises loanable funds through the sale of Farm Credit Securities to investors -NOT GUARENTEED BY THE FEDERAL GOV. -issues discount notes, floating rate bond, and fixed rate bonds proceeds go to famers with real estate loans, rural home mortgage loans, and crop insurance

nonfinancial investment considerations

-age -marital status -employment -dependents & their ages -tax status -education & healthcare needs -attitudes

Aggressive Growth Funds

-aka performance funds -invest in small cap ($2B and under)

book value per share

-almost identical to NAV calculation -liquidation value of the enterprise

illiquid securities

-annuities -real estate -DPP -hedge funds

lifetime records

-articles of incorporation or partnership agreement -minutes of board meetings -records of stock certificates

*3A's for discretionary trading

-asset -amount -action

5 uses of ETFs

-asset allocations -following industry trends -balancing a portfolio -speculative trading -hedging

Nasdaq level 2

-available to approved subscribers only -provides the current quote and quote size available from each market maker in a security in the system

6 year records

-blotters -customer ledgers -customer WRITTEN complaints -general ledger -customer account records -principal designation record-handy for 2 years

Coverdell ESA age limit

-contributions must be made before bene turns 18 unless special needs -funds must be used before bene is 30 years old

*two items that do NOT include on the order ticket

-customer's name or address -current market price of the security

SMA increases when

-dividend -sale of stock -non required cash deposits -loan value

C Corporation Disadvantage

-double taxation

Fixed-dollar withdrawal plan

-fund liquidates enough shares each period to send that sum

UIT benefits

-income -liquidity -rolling over proceeds

accretion for a discount bond

-increases cost basis -increases reported interest income

the two components of duration:

-interest rate -maturity date

3 risks with GNMA MBS

-interest rate risk -prepayment risk, the risk that underlying mortgages will be paid odd earlier than anticipated (occurs when interest rates fall) -extended maturity risk, the risk that the underlying mortgages will remain outstanding longer than anticipated (occurs when interest rates rises)

Immediate Annuity

-is purchased with a lump sum -the payout of benefits usually commences within 60 days

common stock may be classified as (4)

-issued -authorized -treasury -outstanding

New Housing Authority Bonds (NHAs)

-issued by local housing authorities to develop and improve low-income housing. -backed by the full faith and credit of the US government -also called Section 8 bonds -AAA rated -most secure of all municipal bonds

alternative investment considerations

-lack of regulation -low transparency -low liquidity -high fees -lack of historical data

4 components of capital structure

-long-term debt -capital stock -capital in excess of par -retained earnings

benefit of investing in a funds of hedge fund

-lower initial investment -

disadvantages of convertible securities to investors

-lower interest paid on convertibles (not suitable for those who seek income) -lower priority than other debt securities -if the security is callable, possibility the issuer will call it and force the investor to convert or not: if converted no more interest payments

UIT risks

-market risk -interest rate risk

Working Capital

-measure of the firm's liquidity -the difference between current assets and current liabilities -increases in WC are caused by profits and sales of securities -decreases in WC caused by declaring a dividend, paying off long term debt and net loss

In the case of real estate program, expenses creating the losses are

-mortgage interest expenses -depreciation allowances for the wearing out of the building -expenses for improvement to the property

securities that are exempt from Regulation T:

-municipal securities -Treasury bills, bonds, and notes -government agency issues

a member must obtain the following before opening an account:

-name -address -SSN/Tax ID -DOB

SECURE act

-no age limit to contributions with earned income -RMDs begin at 72 -2 additions exceptions to avoid 10% federal penalty: baby or adoption in 1st year up to $5k -inherited retirement accts require distributions to be completed w/in 10 years after AO passing -529 rule: $10k for student loan payment -ERISA: part time worker (500 hours for 3yrs) is now eligable for 401k

maximum sales charge for annuities

-no maximum as long as it is reasonable

S corporation

-offers limited liability -profits & losses go directly through the shareowners

Alternative Orders (OCO)

-one cancels the other -if one of the orders is executed, the other is immediately canceled

Oil and gas arrangements

-overriding royalty interest -reversionary working interest -net operating profit interest -disproportionate sharing -carried interest -functional allocations

Strategic Asset Allocation

-passive strategy -refers to the proportion of various types of investments that should compose a long-term investment portfolio ex: a 30 year old should be 30% in bonds/cash 70% in stock

mutual fund risk-tenure risk

-past performance is no assurance of future results -portfolio manager's contracts are renewed every year so it is possible for the PM to leave

Hedge Fund Lock-Up Provisions

-period where the investor cannot withdrawal investment dollars -lock up provision is dependent on what the investment strategy of the fund is AND how long the PM anticipates it will take to implement the strategy and then see results of that implementation

*section 1035 exchange abuses where the RR emphasizes the tax-free nature of the exchange without pointing out possible disadvantages include of

-possible surrender charges on the old policy -a new surrender charge period on the new policy -possible loss of higher death benefit that existed on the old policy

Why use a SELL stop order?

-protect against a loss in a long stock position -protect a gain from long stock position -establish a short position when a breakout occurs below the line of support

tax sheltered annuities offered through 403b plans are offered to employees of

-public educational institutions -tax-exempt organizations (501 (c)(3) organizations) -religious organizations

*what type of common stock has interest rate risk?

-public utility stocks because of their dividend policy & because they are usually highly leveraged -dividend policy: as interest rates rise , stock prices decrease -leverage: as interest rates go up so does the utility's borrowing cost which could result in a reduction of the dividend

Cannot be purchased on margin and cannot be used as collateral:

-puts and call options -Rights -Non-NASDAQ OTC issues not approved by the Federal Reserve -insurance contracts

*Components of Income Statement

-revenues -pretax income -COGS

CMO suitability

-some varieties of CMOs like PAC companion tranches may be unsuitable for small or unsophisticated investors because of their complexity and risks -customer must sign suitability statement before buying

IRA investments

-stock -bond -REIT -mutual funds -gold & silver -unit investment trusts -limited partnerships -US government securities -annuities -platinum bullion -palladium

estate tax exclusion

-the estate of a deceased person is allowed to exclude some of that person's estate from taxation

flow of funds

-the issuer pledges to pay expenses in a specific order -net revenue pledge is used meaning that operating and maintenance expenses are paid first

donor taxes

-when a person dies, tax is due on the estate -tax is payable by the estate, not by heirs who inherit the estate -if a person gives a gift, tax is due on the gift -*gift tax is payable by the donor, not the recipient -*estate and gift taxes are progressive taxes -the valuation of the estate is the date of death -the valuation of a gift is the date it is given

An investor is long 300 shares of VHJ and is short 300 shares of VHJ. A tender offer is announced, how many shares may be tendered by the investor?

0 300-300=0

maturity of money market

1 year or less

A person is long a security if he:

1. Has title to it 2. Has an options to purchase an option and has exercised that option 3. Owns a security convertible into or exchangeable for the security and has tendered such security for conversion or exchange 4. Has purchased the security or has entered into an unconditional contract to purchase the security but hasn't received it yet

*the markup policy is named for two reasons:

1. It applied to markups, markdowns, and commissions meaning it's applicable to principal and agency transactions 2. 5% is not the limit. A transaction charge more than 5% might be fine if it's reasonably based on the circumstances of the trade

Flow of Funds in Net Revenue Pledge

1. Operations & Maintenance 2. Debt Service Account (int/prin maturing in current year) 3. Debt Service Reserve Fund (Holds 1yr debt service) 4. Reserve Maintenance Fund (Supplement general maintenance fund) 5. Renewal and Replacement Fund 6. Surplus Fund

When more than one broker enters the same bid or offer, the specialist (DMM) awards the trade in the following order

1. Priority 2. Precedence 3. Parity

the two main roles of a NYSE DMM is:

1. facilitate trading in certain stocks (display their best bid and ask prices) 2. minimize price disparities that may occur at the open of daily trading by buying or selling stock from their own inventory only when a need for such intervention exists

before a member can open a account for day trading the member must (2)

1. provide the customer with a risk disclosure 2. approve the account for day trading or receive from the customer written statement that the customer DOES NOT intend to engage in day trading

each basis point is

1/100 of 1%

*foreign currency option sizes

10,000

each option contract represents ____ shares

100

hold period for a specified adult who may be financially exploited is

15 business days

BD must report CTRs (wires & currency) within

15 days of receiving it

Level 2 Halt

15 min halt if 13% decline in S&p from 9:30-3:25

Level 1 Halt

15 min halt if 7% decline in S&P from 9:30am-3:25pm

*for variable annuities or variable life how many licenses are needed for sale?

2 -securities & insurance

Long Straddle Breakeven

2 BE points -strike price +/= premiums

Level 3 Halt

20% decline in S&P 500 At any time -- trading shall halt and not resume for the rest of the day

any margin requirement can be met with marginable stock worth

200% of the amount of the call

If the terms of the tender offer are changed, the revised offer must remain open for at least ______ business days from the commencement and ____ business days from the date the terms are changed

20; 10

how many times are dividends taxed?

2X -interest payments are taxed 1X

if a change was made on the order ticket due to a mistake, all the facts surrounding the trade must be retained for

3 years

*ABC authorized 3,000,000 shares of common stock. It issued 70% of those shares and purchased 600,000 for its treasury stock. How many shares are outstanding?

3,000,00 x .7=2,100,000 2,100,000-600,000=1,500,000 shares

if a customer makes any changes to their account record the member firm must send the new, updated account record to them within

30 days

FCO expiration

3rd Friday of the expiration month

when do option contracts expire?

3rd friday of the month at 11:59pm

if no extension is received by the morning of the ______ business day, the firm is forced to __________

3rd; must sell out the securities purchased and freeze the account for 90 days. If the customer wants to purchase funds in the frozen account, customer must make a full payment before order entry.

Broad-based index options trade until _______ p.m. ET.

4:15 ET

*if a trade is made for 550 shares of common stock how many round lots were there?

5 round lots 1 odd lot (50)

*Treasury bills maturities

52 weeks or less -52 weeks are auctioned monthly

how long is the standard maturity of an options contract?

9 mo.

net investment income formula

=DIE =dividends+intersest-expenses of the fund -operating expenses arent included but attorney & accounting fees, management fees, bank charges are

In a DPP, liability for the debts of the business falls on A. GP B. LP C. S/o D. Agents selling the program

A

One of your customers owns five JLO 5s of 2042. The debentures have a conversion price of $15. When the market price of the convertible is 80, the parity price of the stock is A) $12.00. B) $5.33. C) $15.00. D) $18.00.

A A debenture with a conversion price of $15 is convertible into 66.66 shares ($1,000 ÷ $15). It is always the par value that is used, not the market price. To determine the parity price of the stock, divide the current market price of $800 by 66.66 and the answer rounds off to $12. Some students find it easier to recognize that the bond is 20% below its par value. To be equal (and that is what parity means), the stock must be 20% below the $15 conversion price (or 80% of it). Reducing $15 by 20% is a $3 reduction to $12 or taking 80% of $15 equals $12. LO 5.d

Due to a distribution of stock, the contract size in the JGH Oct 50 call options is 108. A customer purchasing one of these contracts for a premium of 2½ would expect to pay A) $270. B) $258. C) $330. D) $250.

A With a contract size of 108 shares (likely from an 8% stock dividend) and a premium of $2.50 per share, the total cost is $270. LO 10.j

the bond counsel's services in providing the legal opinion associated with a new issue of municipal bonds is the responsibility of A. the issuer B. the underwriter C. the investor D. MSRB

A. the issuer retains the bond attorneys or the bond counsel for a new municipal bond issue. The underwriters may also hire bond counsel but they don't provide the legal opinion

when a specified adult opens an account the member firm must

ATTEMPT to have a trusted contact info

corporate resolution

Action passed by a corporation authorizing the opening of the account and the officers designated to enter orders

Individual Account

An account that has only one holder

Depreciation write-offs for oil and gas

Applies to the cost of recovery of expenditures for equipment and real estate (land cannot be depreciated)

Gift taxes are due at

April 15

tradition IRA distributions must begin

April 1st in the year after the AO turns 72

Balance Sheet Equation

Assets = Liabilities + Owner's Equity Assets - Liabilities = Owner's Equity

A customer purchases five 6.25% U.S. Treasury notes at 98.24. How much will the customer receive on each interest payment date? A) $312.50 B) $156.25 C) $153.50 D) $154.30

B Explanation Although minimum purchase denominations can be less, always use par value ($1,000) for these calculations. A 6.25% bond pays $62.50 annually (6.25% × $1,000 = $62.50). Therefore, a customer purchasing five bonds receives $312.50 each year. Because Treasury notes pay semiannually, each interest payment equals $156.25. LO 7.b

Investing in undeveloped land satisfies which of the following primary objectives? A) Investment tax credits B) Appreciation C) Deductible interest expense D) Tax-deferred income

B The primary reason to invest in raw land is the appreciation in value. LO 11.e

BACK END LOADS

B SHARES

*which types of orders are automatically reduced for cash dividends?

Buy Limits and Sell Stops (BLiSS)

*Limited POA

Buy stock and sell stock -cannot w/d funds

Which of the following quotes represents a municipal dollar bond quote? A) 8.20 - 8.00 B) 0.085 C) 85½ D) $850 - $870

C Dollar bond quotes are based on a percentage of face amount (Par $1,000). Therefore, a quote of 85½ is 85.5% of $1,000, or $855. LO 6.a

One of your clients with a margin account is concerned about recent declines in the market price of securities in her portfolio. The current market value of her holdings is $2,100,000, and the debit balance is $900,000. She would like to know how low the value can go before she receives a margin maintenance call. You would reply A) $1,050,000. B) $1,575,000. C) $1,200,000. D) $1,170,000.

C) $1,200,000. The computation for this question is the debit balance ($900,000) divided by 0.75 and that is equal to $1,200,000. At that point, the equity in the account is $300,000, which is equal to 25% of the long market value. LO 16.d

A technical analyst would be most interested in A) current ratio. B) corporate earnings. C) support levels. D) price-to-earnings (P/E) ratio.

C) support levels. Technical analysts care about price and volume trends in the marketplace, such as support levels. A corporation's earnings, P/E ratio, and current ratio would be of most interest to a fundamental analyst who reviews a company's financial statements in more detail. LO 13.e

Alternative Minimum Tax (AMT)

Created by Congress to make it more difficult for wealthy individuals to avoid paying taxes through the use of various deductions.

Price Earnings Ratio

Current market price per share/earnings per share

An investor purchased a new issue corporate zero-coupon bond for $600. The bond has a maturity of 20 years. Six years later, the investor sells the bond for $700. For tax purposes, this would result in A) a capital gain of $20. B) a capital gain of $100. C) a capital loss of $280. D) a capital loss of $20.

D The $400 discount is accreted over the 20 years to maturity. That is an annual accretion of $20. After 6 years, that is $120, making the tax basis of the bond $720. Because the sale at $700 is $20 less than the basis, the investor has a long-term capital loss. LO 5.e

If a customer buys 1 OXY Oct 50 call at 3, and the holder exercises the option when the stock is trading at 60, what is the cost basis of the 100 shares? A) $6,000 B) $5,000 C) $6,300 D) $5,300

D The cost basis of acquiring the shares is 100 multiplied by the strike price of 50, which equals $5,000, plus the cost of the call, which is $300. Total cost or cost basis is $5,300. LO 10.i

One of the distinguishing differences between variable annuities and mutual funds is that variable annuities A) generally have lower surrender charges. B) are considered securities. C) offer a guaranteed rate of return. D) have a separate account.

D The variable part of a variable annuity is the separate account. That is what makes it a security product similar to a mutual fund. The only guarantee with a variable annuity is that, once annuitized, payments will continue for life (or the designated period certain). The amount of those payments will vary based on the performance of the separate account. One of the negative features of variable annuities is that they typically have surrender charges that can last a decade or longer. LO 9.a

which is not included in the definition of coterminous debt? A. country B. city C. school district D. state

D property taxes are not assessed by states

Time Spread / Calendar Spread / Horizontal Spread

Different expiration dates but the same strike price

If securities are sold in a restricted account, which of the following are affected? LMV, DR, EQ, SMA?

LMV, DR, & SMA -EQ not affected

*EQ=

LMV-DR

customer purchases 1000 shares of YFG at 60 on margin and borrows the max. 50% from the BD. margin chart:

LMV: $60,000.00 DR: $30,000.00 EQ: $30,000.00

oils and gas partnership-developmental advantages

Less discovery risk than exploratory

open end investment companies can issue

ONLY COMMON STOCK

application of the 5% rule of rule 2121 applies to

OTC and exchange markets when customers buy and sell

Member firms must treat proceeds transactions as

One transaction for markup and markdown purposes

pricing of open-end IC

POP=NAV per share+sales charge

Fixed assets include of on the balance sheet

PPE Usually depreciated over time

Participating Preferred Stock

Preferred stock that shares with common stockholders any dividends paid in excess of the percent stated on preferred stock. -EX: FHC 6% preferred participating to 50%

*PACs have the least amount of

Prepayment and extension risk

TAC are protected against _________ risk but not protected from ______________ risk

Prepayment; extension

If a limited partnership sells privately, investors must receive

Private placement memorandum for disclosure Involves a small group of investors with large contributions

In a public offering of a limited partnership, partnerships are sold with a

Prospectus to a large number of investors with a small contribution

CR=$105,000. SMV=$60,000 EQ=$45,000 Calculate Reg T & margin minimum Also calculate SMA & EE

Reg T=$60000x.5=$30000 Margin minimum=$60000x0.3=$18000 Sma & EE=15000

Which of the following is applicable to the Nasdaq PHLX? Regional exchange operated by Nasdaq Offers trading in equity securities and options contracts Completely electronic exchange with no physical trading floor Regional exchange operated by FINRA for the execution of over-the-counter stocks only

Regional exchange operated by Nasdaq Offers trading in equity securities and options contracts The Nasdaq PHLX is a regional exchange operated by Nasdaq where equity securities and options contracts are traded both electronically and on the floor. LO 10.j

double-barreled bonds

Revenue bonds that have characteristics of GO bonds. Interest and principal are paid from a specified facility's earnings, but bonds are also backed by taxing power of the state or municipality.

Inverted saucer trendline

Reversal of an inward trend

Bullish breakdown

Rise through the resistance level Signals the beginning of a new upward or downward trend

*Unless stated otherwise, firm quotes are good for _____________ only

Round lot only

In short margin accounts customers borrow

Securities

Wash Sale

Selling a security at a loss for tax purposes and, within 30 days before or after, purchasing the same or a substantially identical security. The IRS disallows the claimed loss. Total period is 61 days Only applies to realized losses not gains

municipal anticipation notes

Short-term securities that generate funds for a municipality that expects other revenues soon. -issued at a discount

*T or F: contributions to a defines benefit plan are not affected by the participant's sex

T

*T or F: municipal bonds are exempt from FINRA's 5% policy

T

T or F: dollar bonds have a sinking provision

T

A legal opinion issued for a municipal bond covers which of the following? Feasibility of public works projects Creditworthiness of the issuing municipality Tax status of the municipal debt Constitutionality and legality of the municipal debt

Tax status of the municipal debt Constitutionality and legality of the municipal debt Tax status of the municipal debt Constitutionality and legality of the municipal debt

In the case of stocks that are traded on the NYSE, the quotes come from

The DMM

annuity unit

The accounting measure used to determine the amount of each payment to an annuitant during the payout period

Employee Retirement Income Security Act (ERISA)

The act that stipulates federal standards for private pension plans. -prevents abuse of pension plans

The certificate of limited partnership contains which of the following? The market-out clause The amount of time the partnership expects to exist Each investor's net worth All conditions of dissolution

The amount of time the partnership expects to exist All conditions of dissolution The certificate contains, among other information, the limited partnership's name and business, the amount of time the partnership intends to exist, and the conditions of dissolution. It does not contain each partner's net worth, nor is there a market-out clause like those generally associated with underwriting agreements for new issues. LO 11.c

To open up a margin account, it's mandatory that the customer signs

The credit agreement and hypothecation agreement

C Corporation

The most common type of corporation, which is a legal business entity that offers limited liability to all of its owners, who are called stockholders

Married Put

The purchase of a put option and the underlying stock on the same day. Special tax rules apply if the put expires -no capital loss if expires worthless

preemptive rights

The right of a shareholder in a corporation to have the first opportunity to purchase a new issue of that corporation's stock in proportion to the amount of stock already owned by the shareholder before the general public

reinvestment risk

The risk that a decline in interest rates will lead to a decline in income from a bond portfolio

liquidity risk

The risk that an asset cannot be sold on short notice without incurring a loss -aka market risk -nonsystematic risk

Regulatory Risk

The risk that changes in regulations may negatively affect the operations of a company. -ex: rulings in the environmental protection agency or FDA -nonsystematic risk

Sell limits are used when the investor believes the stock price is currently

Undervalued They want to get out of the stock of they rise to a certain price

Automatic Exercise

Unless other instructions have been given, contracts that are in-the-money by a specified amount at expiration will automatically be exercised if in the money by $0.01. The amount, determined by OCC, applies to both customer and institutional accounts.

GP Liability

Unlimited liability

Ratio call writing exposes an options investor to which of these? Limited loss Unlimited loss Limited gain Unlimited gain

Unlimited loss Limited gain Ratio call writers assume unlimited loss potential in a rising market and limited gain potential in a falling market. Maximum gain—while limited—occurs if the stock is trading at the strike price of the short calls at expiration. LO 10.d

*When is the $2000 minimum waived when you for example buy $15 100 shares?

When it's the first transaction

Excess contributions to an IRA are subject to

a 6% penalty tax if not removed by April 15

profit sharing plan

a benefit whereby employees may share in the profits of the business

straight preferred stock

a preferred stock whose missed dividends do not go into arrears, a.k.a. "non-cumulative preferred."

Collateralized Debt Obligation (CDO)

a security comprised of a bundle of collateralized mortgages or other debt from multiple sources like bonds or non-mortgage loans

Income statement

a summary of a firm's revenues and expenses during a specified accounting period -judges the efficiency and profitability of a company's operation

Day Order

a trading order that expires at the end of the trading day during which it was made

Management Investment Companies

actively manages a securities portfolio to achieve a stated investment objective -either opened or closed-end -mutual funds=open-end -ETF=closed

*calculating accrued interest for US treasuries use _______ days

actual days

Stock dividends

additional shares of a company's own stock given to stockholders -does not change value

retained earnings

aka earned surplus/accumulated earnings are profits that have not been paid out in dividends

Eurobond

bond denominated in a currency other than that of the country in which it is sold -ex: a swiss company issued bonds denominated in British pounds

bond and money market securities negotiability

bonds and money markets are readily transferrable -enables investors to sell their security before maturity date

units

bonds or preferred stocks in addition to a warrant offer -makes securities more attractive

Equipment Trust Certificates

bonds secured with factory and equipment as collateral -will pay a down payment & will finance the rest -corporation will pay off a portion of the loan on an annual basis -after they pay off the loan it will receive the title to its equipment from the trustee

municipal bond marketability-dollar price

bonds with a lower dollar price, the more marketable

combinations

composed of a call & put with different strike prices, expiration months or both -cheaper than long straddles if both options are out of $$

credit agreement

contract that stipulates repayment terms for credit cards

nondeductible contribution

contribution made to a qualified plan or IRA that is made with after tax $, funds grow tax deferred

qualified

contributions are made with pre-tax dollars and earnings in the account are tax-deferred until funds are w/d

current ratio

current assets - current liabilities -the higher the ratio, the more liquid the company

quick assets include of

current assets-inventory

parity price

current stock price/ conversion ratio

*FINRA warns investors that most leverage and inverse ETFs reset

daily -not suitable for buy & hold investors

interest rates increase, preferred stock prices

decreases

decreases in working capital is caused by

decreases in WC caused by declaring a dividend, paying off long term debt and net loss

if payment is late meeting a margin call, the BD may apply to _______________________ for an extension

designated examining authority (DEA)

Portfolio income includes

dividends, interest, net capital gains derived from the sale of securities -always taxes in the year in which it is earned

Open ended investment company number of shares

does not specify the number of share

*What is a periodic payment immediate annuity?

doesn't exist

TSA's are funded by

elective employee deferrals

*JTWROS the ownership is

equal

churning

excessive buying and selling of securities to generate commissions

sales agreement-mutual funds

firm must have a selling agreement with the IC to sell a particular investment company security

Wrap fee account

firms that provide a group of advisory services in addition to brokerage services and charge a single fee (usually a % of assets)

Hypothecation Agreement

gives permission to the broker/dealer to pledge customer margin securities as collateral

to open up a joint account what must be signed?

going account agreement that requires all the owners signatures

the higher the strike price on a put the ____________the premium

higher

how can a bond have a stronger bond rating?

if the loan has collateral it will be stronger

substantially equal periodic payment exception

if you receive IRA payments at least annually based on your life expectancy the w/d avoids 10% penalty

Nominal/subject Quotation

indicates a dealer's estimate of a security's market value

Auction Rate Securities (ARS) maturity is

long term 20-30 year

Risk of money market funds

low risk/high quality -high liquidity -yields are low

when the equity in the account falls below the minimum maintenance requirement, the customer receives a

margin maintenance call

if a employee retires from a deferred compensation plan they

may forfeit the benefits

ex-legal designation allows a bond to

meet good delivery requirements without attached legal opinion

The Securities Act of 1933 deals with

new issues

*Regulation T imposes margin requirements for

new securities transaction and for w/d of cash or other collateral -does not otherwise establish any req. relating to the amt. of margin that must be maintained in a customer's account after it has bought (or sold short) 1 or more securities

*does ERISA regulate federal or state government or non-qualified plans plans?

no

are custodial accounts joint accounts?

no

if the dividend is nonqualified, it will be taxed at

ordinary income -higher than qualified

interest paid by corporations on their debt securities will be treated as

ordinary income -interest is taxable on a state and local level as well

all sales commission and expenses for an open-end fund are embedded in the

pop

TSA/403b plans are made by _____ contributions

pretax

collar is an option strategy generally used to

protect an unrealized gain on a long stock position

Tax Credit BABs

provide the bondholder with a federal income tax credit equal to 35% of the interest paid on the bond in each tax year -can be carried forward if not used

Advantages of margin accounts for customers

purchase more securities with a lower initial cash outlay leverage the investment by borrowing a portion of the purchase price

raw land DPP objective & risk

purchases undeveloped land for its appreciation potential -highest risk

*if the insured dies with a loan outstanding, the death benefit is

reduced by the amount of the loan

advance refunding-municipal bond

refinancing an existing municipal bond issue before its maturity or call date by issuing money from the sale of a new bond issue -the proceeds of the new bond issue are used to purchase special gov. securities -increased the quality of the bond, thus increasing marketability

rule 2121

required that the investing public receives fair treatment and is charged reasonable rates for brokerage services' -CHARGES OF OVER 5% CAN BE REASONABLE****

A sell limit order is executed when a stock is rising. falling. at or below the limit price. at or above the limit price.

rising. at or above the limit price. A sell limit order is placed above the prevailing market price. Therefore, it may be executed if the market is rising. If executed, limit orders will be filled at the limit price or better, which in the case of a sell limit is the limit price or higher. LO 16.a

Roth Conversion avoids 10% early distribution tax if

rolled over within 60 days under 59.5

money that is needed within 3-5 years should be invested for

safety & liquidity

FINRA rule-selling dividends for mutual funds

selling dividends is prohibited practice of inducing customers to buy mutual funds by implying the upcoming distribution will benefit them since NAV will drop by the dividend amount

shareholder of an IC must be sent financial information

semiannually

Commercial Paper (CP)/promissory note

short-term unsecured debt issued by large corporations to raise working capital -maturities: 1-270 days -issued at a discount from face value

duration: the higher the coupon rate, the ________the duration

shorter

when a customer whose securities is carried by a member wants to transfer the account to another member, the customer

sign an account transfer form called a Transfer Initiation Form (TIF)

to open a prime brokerage account for a customer, a the prime broker must

sign an agreement with the customer, spelling out the contract terms and names the executing brokers

*why are REITs liquid?

since they trade on the exchange and OTC

*cumulative voting benefits the

small investor SMALL CUM

a credit spread is profitable if it can be closed out at a

smaller credit than the initial debit

if call is exercised, ______will be delivered within _____ day(s)

stock; 2

stop-limit order becomes a _____ order once triggered

stop order

the current market price of a bond is determined by

supply and demand -price will flucuate

MSRB describes marketability as

the ease or difficulty with which securities can be sold in the market

sovereign debt securities safety depends on

the economy of the issuing country

market risk

the potential decrease in the value of a stock in a stock market -systematic risk -one way to protect against market risk is to have some negatively correlated securities in the portfolio

Business Risk

the potential for loss or failure caused by poor management decisions -at the best, earnings lower -at the worst, the company goes bankrupt

Fundamental analysis

the study of a company's accounting statements and future prospects to determine its value

call protection

the time in which the bond cannot be called- call is deferred

municipal bond marketability-block size

the typical block for municipal bond trade is $100,000. -smaller trades are not as marketable

class c charges

typically 1%-1 year CDSC 0.75% 12b-1 fee 0.25% shareholder service fee 0.75% and 0.25% never go away -quite expensive to own if investing for over 4 years

investors who speculate will

use margin, sell short, & trade penny stocks

DPPs were formerly known as tax shelters because investors

used losses to reduce or shelter ordinary income know they are classified as income and loss from investments as passive income

paying agent of a bond is

usually a trust department of a bank -transmit payments of interest and principal to the investors

*does ERISA regulate private sector plans?

yes

the 5% markup policy

-adopted to ensure that the investing public receives fair treatment and pays reasonable rates for brokerage services in both exchange and OTC markets -considered a guideline ONLY and NOT a firm rule for markup and markdowns Order can be filled: >if the B/D is a market maker in the security, it will (as principal) buy from or sell to the customer, charging a markup or markdown >if the firm is not a market maker in the security, it can fill the order as agent, without taking a position in the security, and charge a commission for its execution services >an order can be filled as a riskless and simultaneous transaction -*applies to markup/downs and commissions -*does NOT apply to mutual funds, variable annuity contracts, or securities sold in public offerings, municipals or anything with a prospectus -*the price to the customer must be reasonably related to the current market -*the more risk, the higher markup is justified

Periodic Payment Deferred Annuity

-allows investments over time; payments of benefits for this type of annuity are always deferred until a later date selected by the annuitant

inverse (reverse) funds

-bearish attitude -attempts to deliver returns opposite of the benchmark the index they are tracking -ex: benchmark is down 2%, funds goes up 2% -not suitable for buy & hold investors

Reporting rules for TRACE

-both sides of the transactions must report -trades must be reported as soon as practicable and no later than 15 minutes are execution -execution date, time of trade, quantity, price, yield and if price reflects a commission charges are all reportable and displayed

new constructions DPP objective & risk

-build new property for potential appreciation -risk: less risky than new land, more risky than existing property

3 types of shareholder equity

-capital stock at par -capital excess of par -retained earnings

Fee-Based Accounts

-charge a single fee (either fixed or percentage of assets) instead of commission based charged for brokerage services. -Fee based accounts are appropriate only for investors who engage in at least a moderate level of trading activity -Accounts with low level of trading activity may be better off with commission based charges

discretionary account rules (5)

-checked regularly -no churning -record of all transactions -officer/partner must approve each order in writing -each order must be identified when entered

information on the order ticket includes:

-customer account number -RR ID # -if the order id unsolicited or solicited -subject to discretionary authority -description of the security -number of shares -action (buy or short) -option (buy, write, covered or uncovered) -order restrictions and price qualifications -type of account -the time the order was received, time of entry, and price at which is was executed

when issued confirmation statement must include

-description of the security -the purchase price -trade date

historic rehabilitation objective & risk

-develop historic sites for commercial use -risk: similar risk as new construction

Government-assisted housing property objective & risk

-develop low-income and retirement housing risk: relatively low risk

DPP or REIT statements include of

-disclosures -net investment -appraised value

*Treasury bill pricing

-discount to par -zero coupon securities -bid & ask price -buyers bid -sellers ask -bid is always higher than asking price -book entry

Nasdaq level 1

-displays the inside market only, the highest bids and the lowest asks for securities included in the system -available to registered representatives thru various public vendors

benefits of including investment company securities in client portfolios

-diversification -professional management

Interval Funds

-do not trade in the secondary market -at certain intervals, investors are allowed to sell a portion of their shares back to the fund at NAV -suitable for long time horizon

MSRB Rule G-17

-each broker dealer and municipal advisor shall deal fairly with all persons and shall not engage with deceptive, dishonest or unfair practices

when assessing the quality of a revenue bond the investor should consider the following factors (5)

-economic justification -competing facilities -sources of revenues -call provisions -flow of funds

2 categories of municipal bonds

-general obligation bonds -revenue bonds

equity income funds

-long history of dividends, current income -mutual funds that focus on firms that pay a high level of dividends

person for breakpoints includes of

-married couples -individuals -parents and their minor children (adult children nope) -corporations -INVESTMENT CLUBS ARENT INCLUDED

bona fide

-must reflect the dealer's best judgement & have a reasonable relationship to the fair market value for that security -may reflect the firm's inventory and expectations of market direction

penny stock history

-new companies -no track record or little to no operating history

common stock transaferability

-no need to get the issuer's permissions to sell

DPP compensation restrictions

-non cash compensation: no gifts over $100 -organization and offering expenses exceed 15% of the gross proceeds FINRA says it is unreasonable

dark pools of liquidity

-not open to the public -generally large volume transactions that occur on crossing networks or ATS systems that match and buy sell orders electronically -usually are institutional traders -does not effect market price quotes -investors can place orders anonymously

3 components of cash flow

-operating activities -investing activities -financing activities

if a 3.78 basis fell to 3.65% we would say the yield has dropped by how many basis points?

13 basis points

Straddle & Combination breakevens

2 breakeven points -2 premiums paid -both premiums +/- strike price

a stop order takes _______ trades to execute

2 trades... 1. trigger-the trigger transaction at or through the stop price activates the trade 2. execution-the stop order becomes a market order and is executes at the next price

hedge fund management fee rule

2-20 rule -2% management fee -20% of any profits are taken by the fund -managers want to have a higher return so they can get more money

minimum requirement for pattern day traders

25k

under Regulation S-P if a BD sends a customer an initial privacy notice that contains an opt-out provision, the firm may not disclose non-public info about the person for ___ days from the mailing

30 days pg. 17

catch-up contribution for IRA

50 years old and over $1k over their annual contribution limit

What's the Reg T in a short margin account?

50%

An investor looking for an open-end investment company with an objective of providing capital appreciation for its shareholders would most likely choose A) a growth fund. B) a bond fund. C) an income fund. D) a venture capital fund.

A Growth funds have the goal of producing capital appreciation (growth); that is why they are named as such. This is a case where you "don't look a gift horse in the mouth." Don't venture capital funds also seek growth? Yes, they do, but they are not open-end investment companies. LO 8.g

When creating a diversified municipal bond portfolio, all of the following should be considered except A) the denomination of the bonds included in the portfolio. B) the geographic location of the issuer. C) the source of funds backing the bonds. D) the credit rating.

A The denomination of the bonds in a portfolio is not relevant to diversification. LO 6.d

If an investor in the 27% federal marginal income tax bracket invests in municipal general obligation public purpose bonds nominally yielding 4.5%, what is the tax-equivalent yield? A) 0.0616 B) 0.1667 C) 0.0572 D) 0.0329

A The formula for computing tax-equivalent yield is nominal yield divided by (1 − federal marginal income tax rate), so 0.045 / (1 − 0.27) = 6.16%. LO 6.f

A stock is trading consistently between $20 and $24. The investor with a long position is neutral on the stock. The goal is to generate income. Which of the following recommendations is most appropriate? A) Sell a call B) Buy a call C) Sell a put D) Buy a put

A The investor should sell a call on the stock and collect the premium (income). The investor is long the stock, so it would be better if the price goes up rather than down. Therefore, the sale of a call is better than the sale of a put, and those are the only real choices when the investor wants income through options. LO 10.d

An investor opens the following position: Buy 1 COD Jan 40 put at 6.50 Write 1 COD Jan 30 put at 2.10 His maximum loss is A) $440. B) $2,100. C) $560. D) $2,600.

A The maximum loss on a debit spread is the net debit. LO 10.e

All of the following terms are associated with general obligation (GO) bonds except A) protective covenants. B) voter referendum. C) coterminous debt. D) limited tax bond.

A The protective covenants are found in the trust indenture of a revenue bond. Among the more common protective covenants are the rate covenant and the maintenance covenant. The former is a promise to maintain rates sufficient to pay expenses and debt service. The latter is a promise to maintain the equipment and facility/facilities. Coterminous debt, or overlapping debt, exists when a single property is taxed by more than one taxing authority (e.g., when property is taxed by both a school district and a county). Certain GOs may have limitations imposed on increasing any of the taxes that back them and are called limited tax bonds. GO bonds require voter approval. LO 6.b

An options investor wishing to follow a market-neutral strategy would be most likely to find which of the following most appropriate? A) time spread B) long straddle C) debit put spread D) long broad index call

A Time spreads, also called calendar or horizontal spreads, consist of two options of the same type with the same strike price, but different expiration months. The strategy expects the market to stay relatively level. The profit arises from the time decay of the later expiration date. A long straddle is profitable only if there is market movement. The same is true with the long call - the market price must go up. A debit put spread is a bearish strategy, so this strategy requires the market price to decline. LO 10.e

A customer in the 28% tax bracket wants to buy a municipal GO bond with a 7.5% yield that matures in 6 years. The tax-equivalent yield of this bond is A) 0.104. B) 0.026. C) 0.075. D) 0.060.

A To calculate the taxable return, use the tax-free equivalent yield formula: municipal bond yield ÷ (1 − investor's tax bracket). Using this formula, 0.075 ÷ (1 - 0.28) = 0.104, or 10.4%. This means the investor, who is in the 28% tax bracket, must earn 10.4% in taxable interest to equal the 7.5% tax-free municipal interest yield. LO 6.f

The owner of an IRA, age 45, has contributed $10,000 into the account and the IRA is now worth $20,000. The owner is going to convert the entire $20,000 into a Roth IRA. What are the tax consequences of this conversion? A) The $20,000 is taxable as ordinary income in the year of the conversion. B) $10,000 will be taxable as ordinary income, and $10,000 will be taxed as a capital gain. C) The $20,000 is taxable as ordinary income, but there is a $2,000 tax penalty for early withdrawal. D) $10,000 will be taxable as ordinary income, and $10,000 will be taxed as a capital gain; in addition, there will be a $2,000 tax penalty for early withdrawal.

A When converting from a traditional IRA to a Roth IRA, the distribution is all taxed as ordinary income in the year of the conversion. There is no 10% tax penalty if the conversion is done prior to age 59½. LO 1.g

Federal National Mortgage Association (FNMA)

A publicly held corporation that purchases conventional mortgages and mortgages from government agencies, including the Federal Housing Administration, Department of Veterans Affairs, and Farmers Home Administration. -Fannie Mae.

Stop Limit Order

A stop order that becomes a limit order after the specified stop price has been reached or passed

Based on yesterday's closing price of $60 per share, Blech Sheet Metal, Inc., has a current P/E ratio of 12:1. If the current quarterly dividend payment is $0.50 per share, the dividend payout ratio is A) 40%. B) 16.66%. C) 10%. D) 3.33%.

A) 40%. With a price-to-earnings ratio of 12:1, the earnings per share (EPS) is $5.00 ($60 divided by 12). Four quarterly dividends of $0.50 is an annual dividend of $2.00 per share. If the company is paying $2 per share from the $5 per share earnings, that is a dividend payout ratio of 40%. LO 13.d

which department in the brokerage firm handles all customer transactions using credit? A. Margin B. Credit C. Purchases and sales D. Reorganizatoion

A. Margin

Which of the following may not lead to an industrial development bond being called? A) Funds are available in the surplus account to call the bond. B) The municipality is approaching a statutory debt limit. C) The facility is destroyed by a storm. D) Interest rates are falling.

B An issuer of industrial development revenue bonds is likely to call bonds to reduce interest costs when interest rates are falling, discontinue interest payments if the facility is destroyed by a natural disaster, or reduce debt if funds are available in a surplus account. Industrial development revenue bonds are not affected by the issuer's statutory debt limits, as they affect the issuance of general obligation bonds only. LO 6.b

John is the annuitant in a variable plan, and Sue is the beneficiary. Upon John's death during the accumulation period, Sue takes a lump-sum payment. What is her total tax liability? A) The entire amount is taxed as ordinary income because it is not life insurance. B) It is the proceeds minus John's cost basis taxed as ordinary income at Sue's tax rate. C) It is the ordinary income on the proceeds over the cost basis plus 10% of the net gain (if any) if Sue is younger than 59½ years old. D) None because it is the proceeds from a life insurance company

B Annuity death benefits are generally paid in a lump sum. The beneficiary is taxed at ordinary income rates during the year the lump sum is received. The amount taxed is the amount of the lump-sum payment minus the deceased's cost basis in the investment. LO 9.d

Individuals with diversified stock holdings in their portfolios write covered calls to A) lock in profits. B) increase their rate of return on the stocks held in their portfolio. C) benefit from share price increases. D) increase the number of shares they own.

B Covered call writing is frequently used by persons who own the underlying stock to increase rate of return. If the options expire unexercised, the writer keeps the premium, which provides additional portfolio income. LO 10.d

An investor interested in monthly interest income should invest in A) corporate bonds. B) Government National Mortgage Associations (GNMAs). C) Treasury bonds. D) utility company stock.

B GNMAs pay monthly interest and principal, Treasury bonds pay semiannual interest, utility stocks pay quarterly dividends, and corporate bonds pay semiannual interest. LO 7.c

A customer establishes the following positions: Buy 100 ABC for 63 Write 1 ABC Jan 70 call for 1 What is the customer's maximum gain? A) 600 B) 800 C) 700 D) Unlimited

B Maximum gain on the covered call position occurs when the stock's market value rises. The short call is exercised when the stock is above 70, so the stock bought for 63 will be sold for 70—a profit of $7 per share. In addition, the customer receives the premium of $1, so the total profit is $800 ($700 + $100). LO 10.h

If an indenture has a closed-end provision, this means A) the bonds must be called before maturity. B) additional issues will have junior liens. C) additional issues have no lien on the revenue stream. D) a sinking or surplus fund must be established.

B These additional issues are also known as junior lien bonds. Under a closed-end indenture, additional bonds issued against the same stream of revenues have a junior (subordinate) claim to those already outstanding unless the funds are required to complete construction of the facility. LO 6.b

An order to sell at 38.65 stop limit is entered before the opening. The subsequent trades are 38.85, 38.50, and 38.35. The order A) was executed at 38.85. B) has not yet been executed. C) was executed at 38.65. D) was executed at 38.50.

B) has not yet been executed. A stop limit order is a stop order that becomes a limit order once the stop price has been triggered. When the limit price is the same as the stop price on a stop limit order, the order may be executed only at or better than the limit price. In this case, the order has not yet been executed because no transaction has occurred at or above 38.65 because the stop was triggered at 38.50. LO 16.a

A customer buys a 20-year, 7% bond on a 7.35 basis. The bond is callable in six years at 103, in eight years at 102, in 10 years at 101, and at par beginning in the 12th year. The customer's confirmation will show yield to A) the 12-year put. B) the 20-year maturity. C) the 10-year call. D) the 6-year call.

B) the 20-year maturity. The confirmation of a bond trade must disclose the lower of yield to call or yield to maturity. This bond is being bought at a discount because the basis is higher than the coupon. Because the yield to maturity on a discount bond is lower than the yield to call, this is the yield that will be shown on the confirmation. LO 15.a

Which statement regarding US gov. agency is true? A. they are all direct obligations of the US gov. B. they generally have higher yields than yields of Treasury securities C. the FNMA interest is taxed only at the federal level D. securities issued by GNMA trade on the NYSE floor

B.

Which of the following types of risks cannot be diversified away? A. Interest rate risk B. Systematic risk C. Business risk D. Liquidity risk

B. Systematic risk cannot be diversified away

A BD would have to complete a form 112 when a customer A. deposits $10k in cash B. deposits $10,000.01 in cash C. wires $3k D. child deposits a personal check for $10k into their TIC account

B. deposits $10,000.01 in cash wires not reported on form 112

investment grade debt

BBB or Baa and higher -top 4 -generally only available for purchase by institutions and by fiduciaries

A customer shorts 200 shares of XYZ and buys 2 XYZ at the money calls as a hedge. These transactions can be done in A) a cash account. B) either a margin or a cash account. C) a margin account. D) a cash account, only if the calls are at the money.

C) a margin account. When shorting stock under any circumstances, a margin agreement is required to borrow the stock. This transaction can be done only in a margin account. LO 16.d

cumulative preferred stock

Preferred stock on which undeclared dividends accumulate until paid; common stockholders cannot receive dividends until cumulative dividends are paid.

investors use this to reduce cost on long puts

debit put spread

Third Market

exchange-listed stock traded OTC

*exchanges are a ______________ event

taxable

price relationship between convertible's value and underlying common stock value

when equity prices are declining, convertible's market price declines

variable securities are they a security?

yes

*variable life insurance contract exchange

you have the right to exchange a variable life insurance contract for a form of permanent insurance issued by the company with comparable benefits -PERIOD IS 2 YEARS OR MORE -NO MEDICAL UNDERWRITING/EVIDENCE OF INSURABILITY IS REQUIRED FOR THE EXCHANGE -AGE OF THE INSURED AS THE ORIGINAL DATE IS THE AGE USED FOR PREMIUM CALCULATIONS FOR THE NEW POLICY

2 major methods of accounting for material costs

-LIFO -FIFO

Mid-Cap Fund

$2-$10B

*minimum maintenance for margin is

25% of LMV

AMT stands for

Alternative Minimum Tax

An upward trendline is

Bullish

*are EE and SMA equal?

Not necessarily

municipal bonds pay interest that is free from

federal taxation

long call maximum loss

premium

the most popular DPP

real estate DPP

REITs are organized as

trust

POP is $10.50 and NAV=$10 what is the sales charge? then calculate sales charge percentage

$10.50-$10=$0.50 $0.50/$10.50=4.8%

Treasury Denominations

$100 to $5 million

Treasury bill denominations

$100 to $5 million

find the current yield: market value $800, 10% nominal interest

$100/$800=12.5%

Coverdell ESA contribution limit

$2k annually for 1 bene across all doners

maximum capital losses that can written off for tax purposes:

$3000 can be carried over the years if losses are exceeded by over $3,000 in one year

A customer purchases 100 SIU at 62 and at the same time buys SIU 60 put at 3. What's the margin deposit?

$3400 $3100+$300

A customer 100 OGS at 62 and simultaneously writes OGS 65 call in his cash account. What is the required deposit?

$5900 $6200-$300

1 mill equals

$0.001

To establish best bid, a buying BD must initiate a bid at least

$0.01 higher than the current best bid

Credit Put breakeven

higher strike price - net premium

put spread breakeven

higher strike price- net credit or net debit -ALWAYS IN BETWEEN THE LOWER AND HIGH STRIKE PRICES

short term fluctuations

hours or days

dollar cost averaging

investing the same amount of money on a regular basis regardless of market conditions

broad based index

reflects movement of the entire market and includes the S&P 100 &500, and the major market index

Price spread (vertical spread)

spread with different strike prices but the same expiration date

Yield to Call (YTC)

the rate of return earned on a bond when it is called before its maturity date

capital structure

the relative amounts of debt and equity that compose a company's capitalization

interest rate risk

the risk of capital losses to which investors are exposed because of changing interest rates -having a diversified bond portfolio won't help since all prices will fluctuate

A customer sells short 1000 shares at $2 per share the margin deposit would be

$2500

discretionary power is given to a broker by

-client gives written authority -account has been accepted by the brokerage firm in writing by the firm

who has a higher dividend rate-straight or callable preferred stock?

callable

when an investor sells a Treasury security at a price higher than the purchase price it is considered a

capital gain

General Obligation Bonds (GOs) are issued for

capital improvements that benefit the entire community -these projects DO NOT generate revenues, so principal and interest must be paid by taxes collected by the municipal issuer -full faith and credit issues

buy a call and it expires results in a

capital loss

callable preferred stock/redeemable

issuing corporation can "call" or buy back issued shares at predetermined pric -used when interest rates fall -WHEN INTEREST RATES FALL, CALL

the safety of a particular municipal issue is based on the

issuing municipality's financial stability

common stock residual claims to assets

it company comes liquidated, each s/o has the residual right to claim corporate assets after all debt and other claims have been satisfied -last to get paid

"I want to sell my FHH stock if it falls to $30, but I don't want less than $29.95 for my shares." sell 1000 shares FHH 30 stop 29.95 quotes are the following: 30.01 29.97 29.94 29.92 when will the trade sell?

it doesn't sell, triggered at 29.97 but it does not sell since the stock does not go above 29.95

Subscription Right Certificate

certificate representing a short term privilege to buy additional shares of a corporation -usually 30-45 days

warrants

certificates entitling the holder to acquire shares of stock at a certain price within a stated period -LONG TERM -purchase price is always higher than current market price on the date of the issued warrant

close-end investment companies can issue

common stock, preferred stock & bonds

in measuring investment performance be wary of

comparing apples to oranges -find the correct evaluation standards for investing is important

Equipment Leasing Programs

created when DPPs purchase equipment leased to other businesses

Head-and-shoulders top

indications of a bearish reversal of an uptrend

Head-and-shoulders bottom

indications of a bullish reversal of a downward trend

*the step up provision does not apply when

inheriting an annuity

life insurance

insurance paid to named beneficiaries when the insured person dies

Long market value (LMV)

the current market value of the stock position the investor purchased

in a custodial account who inputs the trades?

the custodian

options are called derivative securities because

their value is derived from the value of the underlying instrument (index, foreign currency, or stock)

employers tax basis (cost) for contributions into a qualified plan is

zero

Fourth Market

market for exchange-listed securities in which investors trade directly with other investors, usually through a computer network -unassisted by BD -placed through electronic communication networks (ECN) which are open 24 hours a day

money market

market in which money is lent for periods of less than a year in the form of securities and loans

put in the money: market price ____ strike

market price <<<<strike price

put out of the money: market price _____strike price

market price >>>>> strike price

put in the money: market price _____strike price

market price<<<<strike price

call in the money: market price ____ strike price

market price>>>>strike

mutual fund risk-market risk

market prices fluctuate -equity funds have market risk

1 spouse makes $45k and the other spouse makes $2k a year how much can they contribute to their IRA?

maximum of $12k

Net Total Debt

net direct debt + overlapping debt

does a member firm have to hold a client's mail for over 3 months?

nope

ERISA Participation

-21 and over -one year of service minimum

investment objectives

-growth -income -stability/capital preservation

Stock Fund

a mutual fund that invests primarily in common stock -growth is a primary or secondary objective

liquid investments

-ETF -stock -mutual funds -REIT

ETF fees

-minimum -brokerage commissions on each trade (in & out)

amortization for a premium

-reduces cost basis -reduces reported interest income

limit on rollovers

1 per 12 mo.

growth and income fund

a mutual fund whose investment goal is to earn returns from both dividends and capital gains

general partnership consists of

2 or more individuals

mail can be held up to

3 months

copies of customer confirmations must be retained for

3 years

*For FINRA customer complaints must be held for

4 years

tranches

a pool of mortgages structured into maturity classes

any client of a member firm wishing to invest in CMO must first complete

a special suitability form

If there is an open order to buy 500 HFF at 30. If there is a 20% stock dividend, the order becomes

Buy 600 HFF at 25

increases in working capital is caused by

increases in WC are caused by profits and sales of securities

call spread breakeven

lower strike price + net credit -ALWAYS IN BETWEEN THE LOWER AND HIGH STRIKE PRICES

Credit Call Spread Breakeven

lower strike price + net premium

*equity linked notes are equity securities or debt securities?

DEBT SECURITIES -dont be fooled

Long Straddle

Long call, long put

*CMOs are traded on the

OTC

Diagonal Spread

Options differ in both time and price

A VIX contract is based on the expected movements of

S&P's 500

Who oversees the selling and promotion of the partnership?

Syndicator

ETFs can be purchased on

margin and can be sold short

bond

a certificate of indebtedness

level loads

c shares -trail commissions

10 year bond with a 10% coupon rate for 95 if held to maturity, calculate YTM.

(annual interest + (discount/years to maturity))/average price of the bond =(100+(50/10))/975 =105/975 =10.77%

Regulation S-P (Privacy Notices)

-Enacted by the SEC to protect customer information -Deals with nonpublic information like SSN numbers, transaction history -Firms must provide a privacy notice describing its privacy policies whenever a new account it opened and after that. -Reasonable opt-out methods must be provided to customers so information is not sent out to third party vendors. -Lastly firms must secure private information (locking laptops and computers, and locking document vaults)

cannot be purchased on margin but CAN be used as collateral after 30 days

-mutual funds -new issues

CEF risks

-pricing risk -leverage risk

2 methods applied to the analysis of DPPs to measure economic soundness

1. Cash flow analysis-compares income to expenses 2. Internal rate of return-determines the present value of estimated future revenues and sales proceeds to allow comparison to other programs

Two lengths of limit orders

1. Day orders 2. Good till canceled

All of the following option contracts are in-the-money when XYZ is 54 except A) short XYZ 50 put. B) long XYZ 50 call. C) long XYZ 60 put. D) short XYZ 45 call.

A Call options are in-the-money whenever the market price is greater than the strike price. Put options are in-the-money whenever the market price is lower than the strike price. Try to remember: call up and put down. LO 10.c

each customer who opens up a new account must within _______ days of opening the account be furnished with a copy of the account record

30

*calculating accrued interest for corporate, agencies, and municipalities use ______days

30 days in each month

Which of the following statements about a Coverdell Education Savings Account (ESA) is not true? A) Contributions are tax deductible, subject to a modified adjusted gross income phase out. B) Contributions can be made to this type of plan and a Section 529 plan in the same year for the same beneficiary. C) The funds grow income tax deferred and, if used for elementary, secondary, or college educational expenses, the earnings are tax free. D) Contributions of $2,000 per child per year are allowed.

A contributions aren't tax deductible

Low short interest means

A bearish indicator

trade confirmation

a printed document that confirms a trade, its settlement date, and the amount of money due from or owed to the customer

MSRB Rule G-22 control relationships

a control relationship exists if the dealer controls is controlled by or is under common control with that security's issuer -dealer must disclose the control relationship to the customer before affecting transactions

Face Amount Certificate Company

A face-amount certificate company is an investment company which offers an investment certificate as defined by the Investment Company Act of 1940. In general, these companies issue fixed income debt securities that obligate the issuer to pay a fixed sum at a future date. They are generally sold on an installment basis.[1] A face-amount certificate (FAC) is a contract between an investor and an issuer in which the issuer guarantees payment of a stated (face amount) sum to the investor at some set date in the future. In return for this future payment, the investor agrees to pay the issuer a set amount of money either as a lump sum or in periodic installments. If the investor pays for the certificate in a lump sum, the investment is known as a fully paid face amount certificate. Alongside with FAC, other company types that fall under the scope of the Investment Company Act of 1940 are Unit Investment Trusts and Management Companies

balance sheet

A financial statement that reports assets, liabilities, and owner's equity on a specific date.

Joint tenants with right of survivorship (JTWROS)

A form of joint ownership of an account whereby a deceased tenant's equal fractional interest in the account passes to the surviving tenant(s). It is used almost exclusively by husbands and wives.

*Tenants in Common (TIC)

A form of joint ownership of an account whereby a deceased tenant's fractional interest in the account is retained by his estate. -ownership can be unequal

Industrial Development Revenue Bonds

A municipal development authority issues these to construct facilities or purchase equipment, which is then leased to a corporation. The municipality uses the money from lease payments to pay the principal and interest on the bonds. the ultimate responsibility for the payment of principal and interest rests with the corporation leasing the facility; therefore, the bonds carry the corporation's debt rating. Technically, industrial revenue bonds are issued for a corporation's benefit. Under the Tax Reform Act of 1986, the interest on these nonpublic purpose bonds may be taxable because the act reserves tax exemption for public purposes. Because these bonds are used for a nonpublic purpose, the interest income may be subject to the alternative minimum tax (AMT) discussed later in this course.

broker's broker

A municipal securities firm acting as a broker for other firms who are not active in the municipal securities market in order to obtain pricing superior to what the firm could obtain itself.

Anti-Reciprocal Rule

A rule created by the Financial Industry Regulatory Authority (FINRA) to protect individual investors from conflicts of interest that may arise when brokerage firms and mutual funds collaborate.

Subject quote

A securities quotation that does not represent an actual offer to buy or sell but is tentative, subject to reconfirmation by the broker/dealer.

Letter of Intent (LOI)

A signed agreement allowing an investor to buy mutual fund shares at a lower overall sales charge based on the total dollar amount of the intended investment. A letter of intent is valid only if the investor completes the terms of the agreement within 13 months of signing the agreement. A letter of intent may be backdated 90 days.

Balance sheet provides

A snapshot of a company's financial position at a specific point in time -identifies assets and liabilities

Forward Stock Split

increases the number of shares and reduces the price without affecting the total market value of shares outstanding FORWARD UP 2:1 3:1

*a relatively wide spread indicates

A thin trading market for the security

Advance/Decline Line

A tool used in technical analysis to measure the breadth of the market. The analyst takes difference between the number of stocks that increased in value each day less the number that have decreased.

ETF risks

index risk tracking risk

Mark to Market

Adjusting the value of a security to reflect current market values. MARGIN ACCOUNTS are marked to the market to ensure compliance with maintenance requirements.

mortgage REIT

a real estate investment trust that invests in mortgage loans that finance the development of properties -they make real estate loans (mortgages). Their earnings come from the interest payments on those loans

ERISA- Communication

All corporate plans must be in writing at inception employees must be given annual updates

exchange market

All of the exchanges on which listed securities are traded. -NYSE -OTC -CBOE -Nasdaq

The key tax benefit of DPP is that they

Allow the economic consequences of a business to flow through to investors. Any income or loss to the investor is considered passive because the investor does not take an active role in the management of business No dividends are paid

If a investor has a diversified portfolio, non systematic risk is reduced to

Almost zero

Market order

a request to buy or sell a stock at the current market price

capitalization

long term debt and equity securities

Narrow-based index

An index that is designed to reflect the movement of a market segment, such as a group of stocks in one industry or a specific type of investment. Examples include the Technology Index and the Gold/ Silver Index. Related item(s): broad-based index; index.

Joint Life with Last Survivor Annuity

An insurance product that, when annuitized, makes payments to the annuitant, the annuitant and his/her spouse, or the annuitant and another beneficial party until both the annuitant and his/her spouse have passed away. -second smallest check

Unit Investment Trust (UIT)

An investment company that sells redeemable shares in a professionally selected portfolio of securities REMAINS FIXED, NOT ACTIVELY MANAGED. It is organized under a trust indenture, not a corporate charter. -sold by prospectus

Credit Spread

An option spread position in which the premium of the option sold is greater than the premium of the option purchased

Coverdell ESA contribution date

April 15 of the following year

Municipal Finance Professional (MFP)

Associated person of a broker dealer who is primarily engaged in municipal securities activities

EPS after dilution

Assumes that all convertible securities like warrants and convertible bonds and preferred stock have been converted in common stock

Market interest rates have been rising, which means the price of bonds traded in the secondary market has A) increased. B) decreased. C) not changed because bond prices are not affected by interest rates. D) not changed because only new bond prices are impacted by changes in interest rates, not the price of bonds already trading in the secondary market.

B

an investor owning 400 shares of FTC notices that there is a 3:2 stock split. How many additional shares will they get? a. 100 b. 200 c. 400 d. 600

B 400x3=1200 1200/2=600 600-400=200

A company is about to pay a dividend of $0.70. On the ex-dividend date, an open order to sell at 46 would

Be adjusted to 45.30 stop

Debit put spread is bullish or bearish?

Bear Spread

coterminous debt (GOs)

Bonds issued by different municipal authorities that tap the same taxpayer wallets

mortgage bonds

Bonds that are secured by real property. -if corporation develops financial problems as is unable to pay the interest on the bonds those real assets pledged as collateral are generally sold to pay off mortgage bondholder

*who may correct a bona fide error?

a supervisory person with the BD but not a RR

*Failure to maintain or adequately staff an OTC order room or other department assigned to execute customers' orders can/cannot be justification for executing away from the best available market?

CANNOT

During a halt, a customer can

Cancel an order that was placed before the halt

3 documents that are required for a limited partnership to exist

Certificate of limited partnership Partnership agreement Subscription agreement

Mortality Expense

Charge collected by the insurance companies as a form of protecting them against mortality risk *

Buy order: if the firm is acting as the broker they

Charge the customer the same price they paid plus commissions

*Which two corporate characteristics are most likely to be avoided by a DPP?

Continuity of life and freely transferable interest

Which of the following statements regarding Section 529 education savings plans are true? Contributions are considered gifts under federal law. Contributions are tax deductible under federal law. Earnings generated are taxable each year. Earnings generated are tax deferred.

Contributions are considered gifts under federal law. Earnings generated are tax deferred. Under federal law, contributions made into Section 529 plans are considered gifts and are not deductible at the federal level. Furthermore, earnings generated each year are tax deferred and, on withdrawal, are tax free at the federal level—if used for qualified education expenses. LO 6.g

Which of the following statements regarding qualified retirement plans are true? Contributions are made with pretax dollars. Contributions are made with after-tax dollars. Distributions are 100% taxable. Distributions are taxable only to the extent of earnings.

Contributions are made with pretax dollars. Distributions are 100% taxable.

2 forms of depletion for oil and gas

Cost & percentage

What happens to SMA when market value declines?

DOES NOT DECREASE

Settlement Date

Date by which payment for the purchase of securities must be made; date by which delivery of securities sold must be made -ownership changes

Commissions and markup rates should ________ as a stock price increases

Decrease

*the longer a bond's maturity, duration is

longer

EQ equation for short margin

EQ=CR-SMV

long margin account equity formula

EQ=LMV-DR

EQ formula when a client has combined accounts

EQ=LMV-DR-SMV+CR

when leveraged and inverse index funds are traded on an exchange they are known as

ETFs

defined contribution plan

Employees and employer pay a specific amount into the plan for each participant. -tax deductible

If a investor fails to choose which shares to sell, the IRS will sell the shares at

FIFO

MSRB rules are enforced by

FINRA with SEC oversight

Trade Reporting and Compliance Engine (TRACE)

FINRA-approved trade reporting system for corporate Bonds trading in the OTC secondary market.

401(k) plan

a tax-deferred retirement plan offered to employees by their employer -pre tax dollars

Business Deductions for DPPs

Expenses of the partnership such as salaries, interest payments, and management fees results in deductions in the current year to the income of the business -principal payments are NOT DEDUCTIBLE EXPENSES INTEREST IS BUT NOT PRINCIPAL

if a F&A municipal bond is traded regular way on March 5 the number of accrued interest is

February 30 days march 5 trade 6 days days of accrued interest: 36 days -all most have 30 days

Who regulates margin transactions?

Federal Reserve Board

oils and gas partnership-developmental disadvantages

Few new wells produce

oils and gas partnership-exploratory/wildcatting disadvantages

Few new wells produce

American Depository Receipts (ADRs)

Foreign shares held by a custodian, usually a U.S. bank, in the issuer's home market and traded in dollars on the U.S. exchange

mutual fund benefits-convenient tax information

Form 1099 is sent by the fund company explaining the taxability of distributions

NAV per share

Fund's NAV / Number of shares outstanding

Target Date Funds

Funds that automatically adjust the retirement account mix based on the employee's age. -Also known as life-cycle funds. -help manage investment risk

fill or kill order

a trading order which if not filled immediately expires

Who can make decisions that legally bind the partnership: LP OR GP

GP

a debit spread is profitable if it can be closed out at a

Greater credit than the initial debit -the spread between the sale price of the long and the purchase price covering the short must be greater than the initial debit

*treasury bond pricing

percentage of par -1/32% -book entry

oils and gas partnership-exploratory/wildcatting tax features

High intangible drilling costs for immediate tax sheltering

oils and gas partnership-exploratory/wildcatting advantages

High rewards for discovery of new reserves

high Beta means

High volatility

Growth managers tend to see a _______ P/E ratio or a ____ price-book ratio

High; high

Growth companies usually have a _______ P/E ratio than cyclical companies

Higher

Earnings before taxes: a highly leveraged company will have __________ interest expenses and a ________ EBT

Higher; lower

The current bid is the price at which the

Highest price the Market maker is willing to pay and buy

deductible contributions

tIRA or 401k plan contributions which can be tax deductible

Index options settle

IN CASH EQUAL TO THE INTRINSIC VALUE t+1

If you hold you ETN as a long-term investment, it is likely that you will

lose all or a substantial portion of your investment

time value of an option

Premium - Intrinsic Value PIT

unlike 529 contributions Coverdell contributions are not

tax deductible

The subscription agreement for a limited partnership includes of

Investors net worth Investors annual income Statement attesting that the investor understands the risk involved POA appointing the GP as the agent of the partnership

When a BD sells to a client on a net basis, it means there

Is no commission charge -this occurs when the member acts as a principal in the trade

SMA or buying power

Line of credit that a customer can borrow from or to use to purchase securities

If a customer is long 500 shares and short 200 shares the net position is

Long 300 shares

Cyclical companies usually have a _______ P/E ratio than growth companies

Lower

The current ask is the price at which the

Lowest price the Market maker is willing to accept

*customer complaint's are required to stay on file for FINRA and MSRB for

MSRB-6 FINRA-4

Average Cost Basis

Method of figuring cost basis on securities for purposes of reporting capital gains and/or losses. The investor averages the cost for all purchases made in the stock, as opposed to identifying particular shares to the IRS when selling. Average basis / total number of all shares

straight line depreciation

Method that allocates an equal portion of the depreciable cost of plant asset (cost minus salvage) to each accounting period in its useful life.

In CDOs, the higher the risk, the

More the CDO pays

collateralized mortgage obligation (CMO)

Mortgage-backed bond that separates mortgage pools into different maturity classes, called tranches. This is accomplished by applying income (payments and prepayments of principal and interest) from mortgages in the pool in the order that the CMOs pay out. Tranches pay different rates of interest and can mature in a few months, or as long as 20 years. Issued by the Federal Home Loan Mortgage Corporation (Freddie Mac) and private issuers, CMOs are usually backed by government-guaranteed or other top-grade mortgages and have AAA ratings.

accretion on a original issue discount bond is

tax free

A customer buys 10 FBV LEAPS AT $4.50 each LEAPS expiration is 24 months. What must the customer deposit?

Must deposit $3375 (0.75x4500)

*can a firm charge a markup/markdown and a commission on the same trade?

Nope one or the other Never both

Bankers' Acceptances (BAs)

Primary and unconditional liabilities of the bank, used primarily as short-term good-faith financing to fund goods as they are being shipped from one country to another. -maturity: 1-270 days

*markups/markdowns are charged when the market maker is acting as a

Principal

what level of trading is appropriate for a commission-based account?

low level of trading

*how common is it to short municipal bonds?

Not common since the market is too thin

a listed ABG May 35 call option is issued by

OCC

terms of the options contracts are standardized by the

OCC

oils and gas partnership-income disadvantages

Oil prices, wells stop producing

Voter approval may be required for new bond issues for construction of which of the following? Airports Turnpikes State prisons Public high schools

State prisons Public high schools Voter approval may be required for new issues of GO bonds. State prisons and public high schools are among the facilities for the public good that are built and supported by GO issues. User fees (like tolls) support revenue bond issues for the construction of facilities such as airports and turnpikes. LO 6.b

he lower strike price on a put the ____________the premium

lower

Securitization

Pooling loans into standardized securities backed by those loans, which can then be traded like any other security. -allows the risk of investing underlying assets to be diversified

economic viability

Potential for returns from cash distributions and capital gains

Market interest rates have risen steadily over the past several months. The market price of which two of the following shares would probably reflect the biggest impact of this change? Growth stock Money market mutual fund Preferred stock Public utility stock

Preferred stock Public utility stock Stocks that are interest rate sensitive will reflect the impact of a change to market interest rates more than others. Preferred stock, with its fixed dividend, and utility stocks, with their high degree of debt leverage, are considered interest rate sensitive. The yield of the money market fund will change, but the price is fixed at $1 per share. LO 14.a

Put/Call Ratio

Put volume divided by the call volume; The higher the ratio, the more negative the sentiment (bearish); Sentiment indicator

one the new account form for an option account is completed, a

ROP and branch office manager must review & approve the new account in writing at or before the initial trade WITHIN 10 DAYS

reasonable-basis suitability

RR has reasonable basis to believe a recommendation is suitable FOR SOME INVESTORS -recognizes risk & reward

*interpositioning can be justified if it

Results in a better execution for the customer Meaning lower price for the inside offer/ask OR higher price than the inside bid

Saucer trendline

Reversal of a downtrend

Over-the-counter (OTC) trading practices in corporate securities are supervised by the Securities Investors Protection Corporation (SIPC). the Securities and Exchange Commission (SEC). the Federal Open Market Committee (FOMC). the Financial Industry Regulatory Authority (FINRA).

SEC & FINRA

ETF registers with

SEC under the Investment Company Act of 1940 either as a unit investment trust or an an open-end management company

foreign currency option settlement

SETTLED IN CASH

What happens to SMA when market value increases ?

SMA increases

fixed annuity guarentees

a fixed rate of return

Limited Liability Company (LLC)

a form of business ownership that offers both limited liability to its owners and flexible tax treatment -not personally responsible for debts of the LLC

Real Time Transaction Reporting System (RTRS)

System used to report municipal securities transactions. Operated by the MSRB. -maintains audit trail -municipal fund securities are exempt

Portfolio's Total Risk can be divided into 2 Parts ...

Systematic and nonsystematic risk

Regulation FD

The federal regulation governing corporate officials who are required to make public (fair disclosure) any information disclosed to analysts or investors

Principal-Only CMOs (POs)

The flow of income for underlying mortgages is divided into principal and interest streams and directed to principal only CMOs

US Government Funds

The fund invests in securities issued by US government. Miniscule default risk. Not considered a direct obligation of the government. Fund not guaranteed by US Gov't. -seek current income & maximum safety -agency security funds are not considered quite as safe from default risk as us gov, bonds

changes in investment policy for an investment company requires

a majority vote of the outstanding voting ex: changing the nature of its business

nominal yield/coupon rate

The interest rate stated on the face of a bond that represents the percentage of interest the issuer pays on the bond's face value (also called coupon rate and stated yield)

negative beta means

The investment will move in the opposite direction than the overall market Ex: if the market is declining then the assets should increase in value thereby increasing the expected performance of the portfolio

The Bond Buyer

The newspaper of the municipal industry containing news and announcements primarily related to new issues. The Bond Buyer publishes a variety of statistics including the 30 day Visible Supply, Placement Ration, acceptances, and ratio indexes

accumulation phase

The pay-in period of an annuity during which the contract owner pays premiums. Synonymous with accumulation period. -contract terms at this time is flexible -can terminate the contract at any time during the accumulation stage

Ratio indexes

The percentage of the total dollar value of new issues sold, versus the total dollar value of new issues offered for sale the prior week.

Reverse Churning

The practice of placing clients who trade infrequently in fee-based accounts -disadvantages the client -low trading is better for commissioned accounts

Market Arbitrage

The simultaneous purchase and sale of the same commodity or security on different exchanges to profit from price disparity between the two markets.

largest borrower and biggest credit risk is the

federal government

Net Direct Debt

The total debt issued by a municipality, less any self-supporting (revenue) debt -includes GOs and short term notes issued in anticipation of taxes or for interim financing

Capital Asset Pricing Model (CAPM)

a model that relates the required rate of return on a security to its systematic risk as measured by beta -higher the risk, higher the returns

the higher strike price on a call the ____________the premium

lower

Oil and gas taxation-tangible drilling cost

Those costs incurred that have salvage value -these costs are not immediately deductible -deductible over several years

A customer buys 1 SHB 60 call at 2.25 and writes 1 SHB 50 call at 8.50. What is the required deposit?

This is a bear call spread at a net credit of 6.25. The maximum loss is the difference between the 10 point spread of the strike price and the 6.25 credit. This 3.75 and makes the required deposit $375

Sell stop limit at 52 while the market is 53 52.5 51.88 51.5. 51.75. 52.25

Trigger at 51.88 Execute at 52.25

YDU is at 40.63 the customer puts a sell YDU at 39.87 stop limit 40.63. 40.75. 40.75. 40.05. 39.83. 39.8. 39.93

Triggered at 39.83 Executed at 39.93

T or F: market risk cannot be diversified away

True

*the interest paid by margin customers on money borrowed is a _________ rate based on the __________

Variable; broker call rate

The thinner the market for the security, the more ________the stock and the ________ the market risk to anyone dealing the stock

Volatile; greater

no load fund

a mutual fund that has no commission fee -can charge up to 0.25% in combined 12b-1 fee and shareholder services fee

When must a Regulation T call be met? Immediately Within 4 business days in a margin account Within 4 business days in a cash account Within 10 business days if both a cash and a margin account are maintained

Within 4 business days in a margin account Within 4 business days in a cash account Under Regulation T, the initial margin requirement on purchases in a margin account, and the full payment on purchases in a cash account, must be made within four business days of trade date. LO 16.d

A bond's basis is its

YTM

Bid yield is also known as the

YTM

a bond's basis is its

YTM

investment company

a business that specializes in pooling funds from individual investors and investing them

3 friends want to start a business. It is anticipated that it will be years before seeing a profit. Which type of business should they open if the want to limit their tax liabilities & receive favorable tax treatment? A. C corp B. S corp C. sole proprietorship D. general partnership

b. S corp allows flow through of profits or losses 12

unsecured debt securities

backed by reputation, credit record, and the financial stability of the corporation

Certificates of Participation

backed by revenue from lease-purchase agreements with the facility they are financing

secured debt securities

backed by various kinds of assets of the issuing corporation

New housing authority bonds/public housing authority bonds are the only municipal bonds that are

backed in full by the US government -also known as section 8 bonds

long straddle maximum loss

both premiums paid

established customer exception for penny stocks

member may solicit w/o having to prepare a suitability statement if the client has: a. effected a non penny stock transaction or made a deposit of funds or securities in an account more than 1 year before penny stock trade OR b. made 3 unsolicited penny stock purchases on 3 separate days involving 3 separate issues -suitability is not required if a client turns in a nonsolicited order to buy a penny stock

Fannie Mae denominations

minimum: $1000 with $1 increments after -interest paid semiannually

Freddie Mac denominations

minimum: $2000 with $1000 increments

ETF benefits

-taxation -expense ratio -portfolio specificity -exchange -traded

Excess equity (EE)

The value of money or securities in a margin account that is in excess of the reg T requirement.

To establish best offer/ask, a buying BD must initiate a bid at least

$0.01 lower than the current best offer

soft dollars

The value of research services brokerage houses provide "free of charge" in exchange for the investment manager's business.

1 point of a bond equals $

$10

a customer buys a 5% municipal bond at 90 maturing in 10 years. calculate accretion. what is the capital gain/loss if the sell for 97 5 years later?

$10 each year 5 years later=$95 sell at $97-95=$20 capital gain per bond

the bank secrecy act requires BD to report on form 112 any currency deposited or received over

$10,000+ =$10,000 cash DOES NOT NEED TO REPORT IT

A customer has the following margin account balance: Market value: $50,000 Balance: (DR) $26,000 SMA: $0 Regulation T: 50% If a customer sells securities in the amount of $20,000 in this account, what is the maximum amount she can withdraw from the account after the settlement date?

$10,000.00 When stock is sold in a restricted margin account, 50% of the proceeds can be withdrawn. This account is restricted because the equity of $24,000 (LMV of $50,000 − DR of $26,000) is less than Regulation T ($25,000 = 50% of the LMV) but more than minimum maintenance ($12,500 = 25% of the LMV). LO 16.d

no registered investment company is permitted to make a public offering of securities unless it has a net worth of

$100,000 or more

par value

$1000

A customer has. Long margin account with a market value of $12000 and a debit balance of $10000. The customer would receive a maintenance call for

$1000 Regulation t=lmv*0.5=6000 Minimum maintenance requirements=lmv*0.25=3000 3000-2000=1000

Farm Credit System (FCS) denominations

$1000 minimum with $1000 increments

*principal payments are made in $_________ increments for CMOs

$1000.00

Sallie Mae denomination

$10000 minimum

an investor purchasing a 7 year Treasury note at a price of 101.20 would be paying

$1016.25

Large-Cap Fund

$10B+

A customer has a long margin account with the following securities in the account. (Assume a 50% FRB initial margin requirement.) Stock Market Price Market Value Debit Balance 100 A Co. $30 $3,000 $8,400 100 B Co. $25 $2,500 200 C Co. $15 $3,000 100 D Co. $35 $3,500 ----------- $12,000 How far could the market value of the securities in the account decline, before a maintenance call will be sent?

$11,200 Explanation: To determine how far the securities worth $12,000 in the account can decline before the customer receives a maintenance call, multiply the debit balance of $8,400 by 4/3. $8,400 x 4 = $33,600 divided by 3 = $11,200. Another method that can be used is to take 1/3 of the debit balance, which is $2,800, and add it to the debit balance of $8,400. The result would be 4/3rds of the debit balance and would equal $11,200. ($8,400 + $2,800 = $11,200.)

Regulation T requirement: 100 shares at $30

$1500

Regulation T requirement: 100 shares at $50

$2500

The minimum equity requirement for pattern day traders is

$25000

debt to equity ratio

Total Debt/Total Equity

the bank secrecy act requires BD to report any wire transfers that are

$3000 or more

cash requirement: 100 shares at $30 FINRA & Reg T

$3000 total FINRA: $2000 Reg T: 1500 NEEDED $2000

A customer purchases 100 HUS AT 62 & simultaneously writes HUS 65 call at 3 in her margin account. What is the margin deposit?

$3100 6200x.50=$3100 The margin deposit is $2800 since $300 from the premium is already in the account

A couple's home has a assessed valuation of $40,000 and a market value of $100,000 what would be the tax if a rate of 5 mills is used?

$40,000*0.05=$2,000

a taxpayer's home is currently valued at $400,000. The tax is based on 50% assessment of market value. If the annual tax rate is 7 mills what is the taxpayer's property liability?

$400,000*.50=$200,000 $200,00*0.07=$1,400

A customer sells short 1000 shares of stock at $4 per share the margin deposits would be

$4000

A municipal dollar bond is quoted at 98¼ to 98¾. The municipal dealer's spread is equal to $5.00 $50.00 5 basis points 50 basis points

$5 50 bps The spread is half a point. In each point, which is worth $10, there are 100 basis points. Therefore, half a point is worth $5 and represents 50 basis points. LO 16.b

A customer has a short margin account. There is 1 stock currently trading at $10 per share. The minimum maintenance requirement for this account is

$5 per share

mutual fund benefits

-choice of objectives -convenience -liquidity -minimum initial investment -automated investing & w/d -convenient tax information -combination privileged -exchanges within a family of funds

A customer buys RVY JAN 60 PUT AT $8.50 and writes RVY JAN 50 PUT AT $2.25. What must the customer deposit?

$625 net debit=max loss

maximum combined contribution for Roth and traditional IRA

$6k $7k if over 50

Traditional IRA tax deduction limit

$6k per individual; $12k per couple OR 100% taxable compensation for 2019

customer buys an 8% municipal bond with 8 YTM at a price of $108. calculate amortization. if held to maturity, what is the capital cost or gain?

$80 must be amortized over 8 years=$10 or 1 point -1 year =$107 =1=$106 -if held to maturity, there is no capital loss since the cost basis has been reduced to par

yield to maturity discount formula

(annual interest + (discount/years to maturity))/average price of the bond

10% coupon bond at 105 with 10 years remaining to maturity. If held to maturity the YTM is

(annual interest - (premium/years to maturity))/average price of the bond =(100-(50/10))/1025 =95/1025 =9.3%

cum right to find the theoretical value of the right

(market price-subscription price)/(# of rights to purchase 1 share + 1)

*cum rights, ABC price per shares is $41 the subscription price per share is $30. Ten rights are needed to purchase 1 share of stock. Find the theoretical value of the right

(market price-subscription price)/(# of rights to purchase 1 share + 1) =(41-30)/(10+1) =11/11 =1

the ex right formula to find the theoretical value after the ex date

(market price-subscription price)/(# of rights to purchase 1 share)

book value per share formula

(tangible assets-liabilities-par value of preferred) / (shares of common stock outstanding) = book value per shares

Roth IRA earnings may be w/d tax free 5 years following the initial deposit if

-AO is 59.5+ -money is used for a first time home purchase -AO has died or become disabled

may be purchased on margin and used as collateral

-Exchange-listed stocks, bonds -Nasdaq stocks -Non-Nasdaq OTC issues approved by the FRB -Warrants

congress authorizes the following agencies of the federal government to issue debt securities

-Farm Credit Administration -Government National Mortgage Association (GNMA or Ginnie Mae) -Federal Home Loan Mortgage Corporation (FHLMC or Freddie Mac) -Federal National Mortgage Association (FNMA or Fannie Mae) -Student Loan Marketing Association (SLMA or Sallie Mae)

Volatility Market Index (VIX)

-Fear Index -measure of the implied volatility of the S&P 500 Index options traded on the CBOE -designed to reflect investor expectations of market volatility over the next 30 days

T-Bonds (Treasury Bonds)

-Long-term securities that pay interest every 6 months -Issued and mature in more than 10 years -T-bonds are priced the same as T-Notes (1/32 of a percentage)

Regulation SHO

-Mandates a locate requirement for securities to be sold short prior to the short sale -not doing this is not permitted which is naked selling

funds of hedge funds

-Registered mutual funds available to all investors that invest primarily in unregistered hedge funds -They can target and diversify among several hedge funds and in this way, give non-accredited investors access to hedge funds

Taxes on ADRs

-The foreign income tax may be taken as a credit against any US income taxes owed by the investor -withholding tax on dividends is taken at the source

three year records

-U4 & U5 -order tickets -long and short securities differences -failed to receive and failed to deliver records -fingerprint cards -FOCUS reports -virtually all other reports -confirmation of trades security and cash loan records

2 ways to cover a put

-a cash covered put -taking a short position in the stock underlying the short put

close end fund benefits-exchange traded

-ability to buy shares on margin and sell shares short -intraday trading -liquidity -proceeds from a sale of CEF are available 2 business days after the trade

*one a firm becomes aware of a death of the AO

-account is frozen -active trades are canceled -account is marked as deceased -POA is revoked -await instructions for the executor of the estate

2 methods of depreciation

-accumulated depreciation (reduces the value of fixed assets on the balance sheet) -annual depreciation (decreases taxable income on the income statement)

CMO liquidity

-active secondary market for CMOs -certain tranches of a CMO may be riskier than other -some CMOs carry the risk that repayment of principal may take longer than anticipated

Transfer on Death (TOD)

-allows the account owner, upon death, to transfer all or partial proceeds to a named beneficiary. -account avoids probate because the estate is bypassed -the assets in the account do not avoid estate tax, if applicable

75-5-10 diversified management company rule

-at least 75% of total asset must be invested in cash & securities issued by companies other than the investment company itself or its affiliates -the 75% must be invested: --no more than 5% of the fund's total assets are invested in the securities of any one issuer --no more than 10% of the outstanding voting securities of any one issuer is owned by the 75% -no conditions on the remaining 25%

Agency Funds

-bonds issued or guaranteed by the us federal gov. agencies & bonds issued by gov.-sponsored enterprises

*the five primary unsystematic risks are

-business risk -financial risk -political risk -regulatory risk -liquidity risk

LPs basis consists of

-cash contributions to the partnership -no cash property contributions to the partnership -recourse debt of the partnership -nonrecourse debt for real estate partnerships only

advantages of convertible securities to the investor

-convertible debenture pays a fixed rate and is redeemable for its face value at maturity -convertible bondholders have priority over common stockholders in the event of a corporate liquidation -convertible debenture's market price tends to be more stable during market declines than the common stock -since the convertibles can be exchanged for common stock, their market price tends to move upward if the stock price moves up: owners have more upside risk and less downside risk -conversion of senior security into common stock isn't considered a purchase and a sale for tax purposes

to open corporate accounts the firm must obtain a copy of

-corporate charter -corporate resolution

3 types of assets on the balance sheet

-current assets -fixed assets -other assets

specific suitability requirements

-customer has been informed of the various features of deferred variable annuities like risks & charges -the customer would benefit from certain features of deferred variable annuities like tax deferred growth, annuitization, or death or living benefit

traditional IRAs w/d before 59.5 that avoids 10% federal penalty

-death -disability -first time home buyer=$10k -qualified higher education expenses for immediate family members -certain medical expenses above AGI limit

Oil and gas taxation depletion allowances

-depletion is only allowed for natural resource program -timber and mining

2 ways to meet a margin maintenance call

-deposit cash -fully paid marginable securities

fixed percentage or fixed share withdrawal plan

-either a fixed number of shares or a fixed percentage of the account is liquidated each period

close end benefits

-exchange traded -pricing -leverage

terms of the offering for preemptive rights

-exercise price -number of shares -final date of exercising the rights

3 types of oil and gas programs

-exploratory -developmental -income

Investment Company Act of 1940 defines these 3 companies as investment companies

-face amount certificate -unit investment trust -management investment companies

existing property DPP objective & risk

-generate an income stream from existing structures -risk: relatively low risk

Loan Consent Form

-gives permission to the firm to loan customer margin securities to other customers or broker/dealers, usually for short sales

Government National Mortgage Association (GNMA or Ginnie Mae)

-government owned corporation that supports the Dept. of Housing & Urban Development -ONLY AGENCY THAT IS BACKED BY THE FULL FAITH AND CREDIT OF THE FEDERAL GOVERNMENT

Principal Protected Funds

-guaranteed of principal adjusted for funds dividends and distributions, backed by an insurance policy -lock-up period-if you sell shares in the fund before the end of the guarantee period-you lose the guarantee on those shares -hold a mixture of bonds & stocks -typically front-end loaded

strategies employed by hedge funds are:

-highly leveraged portfolios -use of short positions -utilizing derivative products such as options and futures -currency speculation -commodity speculation -investing in politically unstable international markets

Bid Wanted, Offer Wanted

-if a customer of a municipal firm is looking for a specific bond, the dealer will actively solicit offers to sell from the marketplace -if the customer has bonds to sell, the dealer actively solicits bids from the marketplace for those bonds

capital stock at par

-includes preferred and common stock listed at par value -par value is the total dollar value assigned to stock certificate when a corporation's owners first contributed capital

primary source of municipal income-GO

-income and sales taxes are major sources of state income -real property taxes are the principal income sources of countries, and school districts are the largest source of city income 0city income include fines, license fees, assessments, sales taxes, hotel taxes, city income taxes, utility taxes, and city personal property taxes

mutual fund risk-interest rate risk

-income oriented funds & bond funds may be subjected to IR risk -when IR rises, bond prices fall -bond fund does not have a maturity date -income of a MMF will fluctuate

3 ways to move a IRA

-indirect rollover/60 day rollover -direct rollover -trustee to trustee transfer

Exceptions to Community Property

-inheritances -gifts -property owned before marriage

index ETF is different from an index mutual fund in 3 ways

-intraday trading, ETFs are priced continuously throughout the day -margin eligibility, Index ETF shares can be purchased on margin subject to the same terms that apply to common stock -short selling-index ETF can be sold short at any time during trading hours

international funds

-invest only in the securities of foreign companies -long term capital appreciation is their primary objective

DPP investor advantages

-investment managed by others -flow through of income and expenses -limited liability (the most they can lose is their investment)

mutual fund benefits-automated investing and w/d

-investors can start voluntary accumulation plans where amounts are invested at regular intervals -or automatic w/d plans

Student Loan Marketing Association (Sallie Mae)

-issues short term and long term securities -the proceeds from the securities sales are used to provide student loans for higher education -interest paid on these securities is taxable at the federal level and is exempt from taxation in most states

the custodian in a UTMA/UGMA can (3)

-liquidate, trade or sell securities -buy or sell assets -exercise rights or warrants

DPP investors face these disadvantages

-liquidity risk -legislative risk -leverage risk -risk of audit -depreciation recapture is

Risks of REITs

-manager has no direct control over the portfolio and relies on professional management to make all purchase and sale decisions. -problematic loans within mortgage REITs can cause decrease in income flow and diminish capital returns. -greater price volatility than direct ownership of real estate because they are influenced by stock market conditions -if the REIT IS NOT PUBLICLY TRADED, liquidity is very limited

*the three systematic risks are

-market risk -interest rate risk -inflation/purchasing power risk

variable rate demand obligations

-maturity of 20-30 years -interest rate resets periodically -sometimes considered money market instrument -ability of the investor to demand the issuer repurchase the bonds at par

OCC exercise limits

-maximum number of contacts that can be exercised on the same side of the market within a specified period -no more than 250,000 contracts can be on the same side of the market within 5 business days (long calls with short puts & long puts with short calls)

variable life insurance suitability

-must be a life insurance need -the applicant must be comfy with the separate account and the fact that the cash value isn't guaranteed -the applicant must understand the variable death benefit feature

Rules for the DMM:

-must maintain a fair and orderly market -must stand ready to buy and sell for their own account, if necessary to maintain a fair and orderly market -transact business for her own account in such a way as to maintain price continuity and minimize temporary price disparities attributable to supply and demand -must avoid transacting business for their own account at the opening or reopening of trading in a stock if this would upset the public balance of supply and demand -must file the reports and keep the books and records the NYSE requires -may trade for her own account in between the current bid and ask quotes in its books

opening accts for other members' employees must

-notify their employer and the executing member in writing of their association -employers written consent

specialized funds

-offer high appreciation but also pose high risk -speculative -25% of their assets in their specialities

*in a net revenue pledge, what is paid first and from where?

-operations and maintenance costs are paid first -debt service is paid from net revenues

SEP IRA qualifications

-over 21 years old -has worked at the workplace for 3 of the last 5 years -has at least $600 in compensation

types of Ginnie Mae issues

-private lending approved by GNMA originate eligible loans and pools them into securities (known as pass-through securities) and sells the GNMA mortgage-backed securities to investors -the principal represented by GNMA certificate constantly decreases as the mortgages are paid down -guarantees timely payment of interest and principal

Interest-Only CMOs (IOs)

-product of POs -sells at a discount -used to hedge against interest rate risk

Federal Home Loan Mortgage Corporation (FHLMC)

-promotes the development of a nationwide secondary market in mortgages by buying residential mortgages from financial institutions and packaging them into MBS for sale to investors -sells mortgage participating certificates and guaranteed mortgage certificates

general wealth of the community-GO

-property values -retail sales per capita -local bank deposits -diversity of industry in its tax base -population growth or decline

Why use BUY stop orders?

-protect against loss in a short stock -protect a gain from a short stock position -establish a long position when a breakout occurs above the line of resistance

Direct Payment BABs

-provide no credit to the bondholder but instead provide the municipal issuer with payments from the US Treasury equal to 35% of the interest paid by the issuer

local government investment pools (LGIPs)

-provides other gov. entities like cities, school districts, etc with short term investment vehicle to invest funds -formed as a trust where municipalities can purchase shares or units in LGIP investment portfolio -not req. to register with the SEC -no prospectus

Tactical Asset Allocation

-refers to the short-term portfolio adjustments that adjust the portfolio mix between asset classes in consideration of current market conditions -active strategy -those who time the market ***-fee based would be appropriate for

New Account Form Requirements

-required for each new account opened: 1. Full Name 2. Date of Birth 3. Address 4. signature of partner, officer or manager saying it has been accepted 5. name of associated persons **1-4 are required under USA PATRIOT ACT of 2001** 5. Occupation 6. Citizenship 7. SSN

Safe Harbour

-research reports -financial newsletters -quantitative analytical software -seminars o conferences with appropriate content -effecting and clearing securities trades

Savings Incentive Match Plan for Employees (SIMPLE)

-retirement plans for businesses with under 100 employees who have earned over $5k in the preceding calendar year

2 major types of asset classes

-stock -bond -cash

major fund categories

-stock funds invest in stocks -bond funds invest in bonds -balanced funds invest in a combination of stocks and bonds -money market funds invest in very short term investments

Alternative Investments

-structured product -can be made to cater to the needs of a specific investor -suitability is determined on a case-by-case basis

Achieving a Better Life Experience (ABLE) Act

-tax advantaged savings accounts for individuals and their families with disabilities -bene is the AO -income earned is not taxed -disability occurs before 26 -only 1 account allowed -after tax dollars & is not tax deductible! -limit to contributions

suitability of 1035 exchanges for VA's

-tax free if done properly -surrender charges -possible reduction in death benefit -different expenses with the old and new contract -the benefits included in the new contract may not be needed by the purchaser

2 ways day traders can avoid having their account frozen regarding marginable securities:

-they can have sufficient funds in the account before making the trade -"swap" checks on the settlement date, pay the BD before receiving funds

Restricted Account Rules for margin accounts

-to purchase additional securities, put up 50% -to w/d securities from the account, the customer must deposit cash equal to 50% of the value of the securities to be w/d -if securities are sold in a restricted account, at least half of the proceeds must be retained in the account to reduce the debit balance. this is called RETENTION REQUIREMENT

Customer Income Statement

-total gross income -monthly expenses -spendable income -job security

penny stock liquidity

-trade infrequently -could be difficult to sell them once you own them (may be no buyer at the price you wish to sell at) -thinly traded security

the listing and trading of government and municipal securities

-traded OTC -not listed on Nasdaq

Technical market analysis

-trading volume -support and resistance levels -advances/declines

Cautions with investing with CDOs

-very complex and hard to understand - allows the originators to avoid having to collect on them when they become due because they are now owned by someone else -RR don't always understand the product in their entirely and can be fined or suspended

disadvantages of convertible securities to the issuer

-when bonds are converted, shareholders' equity is diluted meaning more shares are outstanding and each share now represents a smaller fraction in the company -shift in the control of the company -reducing corporate debt through conversion means a loss of leverage -the resulting deductible interset costs raise the corporation's taxable income

discretion is the authority to decide

-which security -# of shares or units -whether to buy or sell

senior exploitation

-wrongful or unauthorized taking, withholding, appropriation or use of specified adult's funds -any act or omission taken by a person to obtain control through deception, intimidation or undue influence over the specified adult's money, asset or property OR convert these

RMD required beginning date for qualified corporate plans

-year after AO turns 72 -year after AO retires from workplace

Separate Trading of Registered Interest and Principal of Securities (STRIPS)

-zero-coupon bond -major banks and BDs separate the interest and principal components -minimal reinvestment risk

Your customer, a small-business owner, likes investments that are short term, relatively safe from credit risk, and liquid. He's heard that higher rates of return can be realized from auction rate securities than the rates he is currently getting on the Treasury bills in his portfolio. He asks you to explain them to him. Which of the following would you note as being reasons why they are not suitable for him? 1. Auction rate securities are intended as long-term investments. 2. Interest or dividend rates are reset at established intervals based on a Dutch auction. 3. If the auction fails, holders of ARSs may not have immediate access to their funds. 4. The interest or dividend rate is set as the lowest rate to match supply and demand at the time of the auction.

1 & 3 Auction rate securities (ARSs) are long-term variable rate bonds with maturities of 20 to 30 years tied to short-term interest rates. As long-term instruments, they are not suitable for an investor favoring short-term investments. Additionally, interest rates are reset using a Dutch auction method at predetermined intervals, typically 7, 28, or 35 days. A failed auction can occur due to lack of demand; in this case, no bids are received to reset the rate. This risk would not align the investment objectives of safety and liquidity. LO 6.b

All of the following are common to both DPPs and REITs except A) pass-through of losses. B) pass-through of income. C) centralized management. D) capital gains distributions.

A Both DPPs and REITs are professionally managed pools that pass through income and capital gains distributions to participants. REITs, unlike DPPs, do not pass through losses. LO 11.f

Which of the following statements regarding the suitability of municipal bonds are true? 1. The tax-free interest payments make them more suitable for those in higher tax brackets. 2. The tax-free interest payments make them more suitable for those in lower tax brackets. 3. The tax-free interest is why municipal bonds are not considered suitable investments to be included in one's retirement account, such as an IRA. 4. The tax-free interest is one reason why municipal bonds are considered suitable investments to be included in one's retirement account, such as an IRA.

1 & 3 Tax-free interest payments are more suitable for those for whom the tax advantage has the most impact. That would be those in higher tax brackets, who would pay more taxes on the interest received if the interest payments were taxable. Additionally, the tax-free interest is why municipal bonds are not suitable for retirement accounts. This is because the earnings in retirement accounts are already tax deferred, and the impact of receiving tax-free interest is lost or diminished. LO 6.f

To comply with the regulations regarding customer identification programs, the minimum identifying information that must be obtained from each customer before opening an account includes which of the following? 1. Their name 2. Oral assurance that the customer is of legal age 3. A street address, unless the primary mailing address is a post office box located in the state of residence 4. A taxpayer identification number

1 & 4 Oral assurance that the customer is of legal age is not sufficient; the actual date of birth must be obtained. A post office box is never acceptable without a physical address. In addition, the identity of the person opening the account must be verified through documentation such as an unexpired drivers license or passport. LO 1.d

Which of the following statements regarding prepayment of CMOs are ordinarily true? 1. If interest rates fall, prepayments increase. 2. If interest rates rise, prepayments increase. 3. If interest rates fall, prepayments decrease. 4. If interest rates rise, prepayments decrease.

1 & 4 When interest rates fall, homeowners often refinance their homes to take advantage of lower interest rates, resulting in the existing mortgages being paid off early. Also, homeowners tend to sell their homes to upgrade to larger homes when mortgage interest rates (and monthly payments) are low. When interest rates rise, homeowners do not usually refinance, and housing turnover is reduced. LO 12.d

once the TIF is sent to ACATS and it is forwarded and received by the carrying firm, the old firm has ______ day(s) to validate the securities listed on the TIF or take exception to the transfer instructions

1 business day

a firm quote to buy a specific bond is good for

1 hour and a 5 minute recall

Maximum 12b-1 fee

1% -0.75% for distribution and promotion -0.25% shareholder services fee

long term gain or loss

366 days or more 12 months and 1 day or more

if a cash dividend is qualified, the tax rate is generally a maximum of

15%

special options account agreement must be signed and return with in ____ days after account has been

15; approved

if the signed agreement is not returned within ____ days of account approval, the

15; firm will have do process closing purchases or sales.

If two customers are tenants in common in a joint account, which of the following statements are true? 1. If one of the tenants dies, the survivor will automatically assume full ownership. 2. They need not make equal investments in the account. 3. They need not have equal interests in the property in the account. 4. If one of the tenants dies, the account need not be frozen.

2 & 3 Under tenants in common, the tenants may make unequal investments in the account and may own a disproportionate interest in the property in the account. If one of the tenants dies, their assets are passed to their estate, not to the surviving joint tenant. The account must be frozen until this is carried out. LO 1.b

The minimum margin requirement for pattern day traders is

25% Same as regular customers

In discussing a direct participation program with your customer, she notes investment characteristics that are important to her and some that are not. For a DPP to be considered suitable for the customer, rank the following items in order of those that should be most important to those that should be least important. 1. Tax write-offs 2. Liquidity and marketability 3. Potential for economic gain 4. SEC approval

3 1 2 4 In the eyes of the IRS, a program's economic viability should be the most important aspect of the investment for a limited partner and the first priority in the assessment of the DPP. While the IRS considers programs designed solely to generate tax benefits abusive, they do allow for some in terms of writing off passive income and allowable tax credits, so these factors would be the next concern for an investor. Because there is a very limited secondary market for DPPs, liquidity and marketability should be a low priority, and because there is no SEC approval of any investment, it would be of no concern. LO 11.g

Secondary trends

3-12 weeks

What's the minimum maintenance requirement in a short margin account?

30%

FINRA minimum maintenance requirement on short positions

30% NOT 25% like long positions

*SEC Rule 17a-3 requires delivery of a copy of account info ___ days after opening and _____ months thereafter

30; 36 mo.

for updating purposes, the member firm must send a copy of the account record to customers every _____ months

36

tax-equivalent yield formula

=municipal bond coupon/(100%-investor's tax bracket)

A debenture is issued with the backing of A. the general credit of the corporation B. pledge of real estate C. pledge of equipment D. the ability to levy taxes

A

All of the following may be used to verify a customer's identity except A) a certified birth certificate. B) a current drivers license. C) a valid military ID card. D) a valid passport.

A

All of the following statements regarding a transfer on death (TOD) account are correct except A) estate taxes are reduced. B) only those assets held at the broker-dealer are transferred. C) the owner of the account may change beneficiaries at will. D) probate is avoided.

A

SEC rules require that open-end management companies distribute dividends to their investors from the firm's A) net investment income. B) gross revenue. C) capital gains. D) portfolio earnings.

A

Under ERISA, a plan trustee wishing to write uncovered calls may do so A) under no circumstances. B) if approved by the IRS in writing. C) without restriction. D) if explicitly allowed in the plan document.

A

If you were reading sales literature about a mutual fund that claimed its objective is to be a single source investment for most equity investors, it would most likely be describing A) a blend/core fund. B) a specialized fund. C) a target date fund. D) a growth/income fund.

A A blend/core fund combines equities with different market characteristics. There are stocks fitting a growth style and others with a value style. It will include some high quality "blue-chip" stocks and some on the other end of the risk spectrum. Target date funds have different portfolios for each targeted date and will include both equity and fixed income securities. The portfolio of a growth/income would have much overlap with that of a blend/core fund, but more emphasis is placed on including stocks for their dividend payout. A specialized fund would concentrate in either a specific geographic area or industry and is not going to be recommended as the only fund for the investor's portfolio. LO 8.g

A retiree is paid an annual amount equal to 30% of the average of his last three years' salary. Which of the following retirement plans offers this type of payment? A) Defined benefit B) Profit-sharing C) Defined contribution D) Deferred compensation

A A defined benefit retirement plan establishes, in advance, the payout to be received by the retiree. LO 1.h

If an investor purchases 500 shares of an aggressive growth stock, which strategy would limit his downside risk? A) Buying five puts on the stock B) Writing five straddles C) Writing five puts on the stock D) Buying five calls on the stock

A A put gives the investor the right to sell stock at a set price (the strike price) for a period of time, and it protects against losses below the strike price. Buying calls can protect a short stock position. If the customer is long stock, the purchase of calls on that security increases leverage and risk. Writing a put creates the obligation to buy more stock at the strike price, which increases downside risk. LO 10.d

A variable-rate municipal bond investment's main advantage is that A) its price should remain relatively stable. B) it is noncallable. C) it is likely to increase in value. D) its interest is exempt from all taxes.

A A variable-rate bond has no fixed coupon rate. The coupon is tied to a market rate (e.g., T-bond yields) and subject to change at regular intervals. Because the interest paid reflects changes in overall interest rates, the bond price remains relatively close to its par value. Its coupon is always representative of the current market rate. As rates rise, the coupon is adjusted upward. As rates fall, the coupon is adjusted downward. LO 6.b

Compared to U.S. government agency-backed CMOs, CDOs have A) less prepayment risk. B) greater liquidity. C) generally more secure collateral. D) less credit risk.

A Although there is some prepayment risk with CDOs, it is minimal when compared to CMOs. Unlike mortgages, which are frequently paid off early when homeowners move, those who move can take their cars or their credit cards with them and continue to make the payments. The same is true with refinancing. You don't see ads for people to refinance their auto loan to the extent you do with home mortgages. In generally, CDOs carry greater credit risk than CMOs backed by FHA and VA loans. The nature of CDOs, with the enormous variety of collateral options available, means that the liquidity of separate offerings tends to be less than that of these CMOs. Both of these are complex securities, but the experts in the field consider CMOs to be somewhat more so. The credit quality of government insured mortgages is more secure than that of credit card debt or automobile loans. LO 12.c

If a customer transfers his holdings from one fund to another within the same family of funds, what are the tax consequences? A) On the transaction date, any gain or loss is recognized for tax purposes. B) No gain or loss is recognized until redemption. C) Gains are taxed, and losses are deferred. D) Losses are deducted, and gains are deferred.

A An exchange is a taxable event. The cost basis of the shares in the original account must be compared to their redemption value. Any gain or loss is recognized in the year of the exchange. The exchange privilege allows the investor to avoid paying an additional sales charge. It does not allow the investor to avoid taxes. LO 8.e

All of the following are risks of investing in publicly traded mortgage-backed securities except A) the market for mortgage-backed securities is illiquid. B) falling interest rates might accelerate early repayment of principal. C) borrowers might default on their mortgage payments. D) rising interest rates might extend the date of repayment of principal.

A As publicly traded securities, liquidity risk is not a major concern to investors in mortgage-backed securities. When interest rates decline, there is generally an increase in mortgage refinancing and that results in investors receiving repayment of principal ahead of schedule. That is prepayment risk. Although it is nice to get the money back, there is reinvestment risk because these lower interest rates mean that reinvesting the principal into new securities will now be at a lower return than previously earned. The opposite happens when interest rates increase. Homeowners are unlikely to refinance, causing principal repayments to slow. Without the repayment, investors holding these securities are receiving below market returns. This is an example of extension risk. As is the case with any loan, there is always default risk. Recent history has shown that it is possible for a large number of foreclosures where the lender does not recoup the full principal. LO 12.c

There are many different types of asset-backed securities, but the common theme uniting all of them is A) they are supported by a contractual obligation to pay. B) they are usually backed by a single source of payment. C) they are a form of equity financing. D) they tend to be exchange traded.

A Asset-backed securities (ABS) are structured debt financing backed by a contractual agreement to pay. They are "first cousins" to MBS (mortgage-backed securities), with the primary difference being that the collateral is not real estate. These securities trade in the OTC market. One of the benefits of the structured package is that the investor is not relying on payments from a single borrower; rather, there is a pool of loans, whether they be auto, credit card, or others. LO 12.c

Which of the following statements regarding revenue bonds issued by a state or municipality is true? A) Interest will be paid only if the enterprise owned and operated by the state or municipality has sufficient earnings to cover the interest payments or the debt service reserve. B) Interest and principal payment is guaranteed. C) The bonds carry an unqualified promise to pay interest and principal backed by the power of the issuer to levy taxes. D) Interest and principal payment is backed by the full faith and credit of the issuer.

A Because revenue bonds are not backed by the full faith and credit of the municipality that issues them, the earnings of the revenue-producing project must be large enough to cover the interest and principal payments. LO 6.b

When it comes to issuing a debt security, which of the following features will generally enable the issuing corporation to borrow at the lowest interest rate? A) Convertible B) Callable C) Zero-coupon D) Cumulative

A Because the convertible feature offers potential growth through the exercise of the conversion option, the interest rate on these securities is generally lower than other debt issues of the same corporation. The call feature increases the reinvestment risk and that is compensated for with a higher coupon. The descriptive adjective cumulative refers to dividend payments on preferred stock, but not to bonds. Because zero-coupon bonds pay nothing until maturity, that added risk requires a higher yield to attract investors. LO 5.c

A 38-year-old investor places $25,000 into a single premium qualified deferred variable annuity. Twenty years later, with the account valued at $72,000, the investor withdraws $50,000. If the investor is in the 25% marginal income tax bracket, the total tax liability is A) $17,500. B) $16,450. C) $12.500. D) $11,750.

A Because this is a qualified annuity, the entire withdrawal is taxable. In this case, it is all $50,000. That $50,000 is taxed at the marginal rate of 25%. Furthermore, because the investor is younger than 59½ (38 + 20 = 58), there is the additional 10% penalty tax. Effectively, this is a 35% tax on $50,000. LO 9.d

ABC Company has issued $20,000,000 of convertible bonds with a coupon of 5% and a current market value of 120. The conversion price is $40. If all the bonds are converted, how many additional shares of common stock will ABC have outstanding? A) 500,000 B) 1,000,000 C) 600,000 D) 400,000

A Each bond will convert to 25 shares of common stock ($1,000 ÷ $40). 20,000 bonds were issued ($20,000,000 ÷ $1,000). Therefore, 500,000 additional shares (20,000 × 25) will be outstanding if all the bonds are converted. LO 5.d

Your customer owns a variable annuity contract. The assumed interest rate (AIR) stated in the contract is 5%. In January, the realized rate of return in the separate account was 7%, and she received a check in February based on this return for $200. In February, the rate of return was 10%, and she received a check in March for $210. For her April check to be $210, what rate of return would the separate account have to earn in March? A) 5% B) 3% C) 7% D) 10%

A Each month's payout depends on the actual earnings compared to the AIR. If the actual rate of return equals the assumed interest rate, the check will stay the same. We don't compare one month's return to another's; we compare the actual to the assumed. If the actual is higher, the following month's check goes up. If the actual is lower, the following month's check goes down. And, as stated earlier, if the actual equals the assumed, there is no change. LO 9.c

*A FINRA member firm wishes to encourage its registered representatives to sell more limited partnership DPPs. As an incentive, the firm offers an all-expenses-paid trip to a popular vacation resort for those reaching certain sales targets. FINRA rules provide that A) the target must be based on the total production of associated persons with respect to all direct participation programs offered by the member. B) sales incentives are limited to gifts that do not exceed $100 in value. C) the member can weight the credits differently for different investment companies. D) the target must be based on the total production of associated persons with respect to specific investment company securities distributed by the member.

A FINRA made a slight modification to its rules on noncash compensation because of the SEC's Regulation BI (best interest). Specifically, if there is to be any kind of sales contest or other method of incentivizing registered representatives, sales of the particular product type must give equal weighting to all of those investments sold by the firm. This applies largely, but not exclusively, to sales of investment companies, variable products of life insurance companies, and direct participation programs. Previously, firms could give higher weighting to sales of proprietary products, but that ended on June 30, 2020. **This question deals with material not covered in your LEM, but it relates to recent rule changes and/or student feedback. LO 11.h

The principal tax benefit of investing in an exploratory oil and gas drilling program is derived from A) intangible drilling costs (IDCs). B) capital appreciation. C) depreciation expenses. D) recapture.

A IDCs, which are a significant portion of all drilling costs, are a major tax advantage to a limited partner and are tax deductible in the year in which they are incurred. IDCs are costs that, after incurred, hold no salvage or ongoing value. Examples include labor and geological survey. LO 11.f

A corporate bond with a nominal yield of 6% is currently trading at a yield to maturity (YTM) of 5.8%. It would be accurate to state that this bond is trading at A) a premium. B) par. C) a discount. D) parity.

A If YTM is less than the nominal or coupon yield, the bond is trading at a premium. LO 4.e

an investor redeems 200 shares of ABC Fund, which has no redemption fee. If the quote is $12.05 bid $13.01 asked, what amount will the investor receive? A) $2,410.00 B) $2,602.00 C) $2,275.50 D) $1,098.00

A If a mutual fund has no redemption fee, the investor will receive the bid price per share (net asset value) multiplied by the number of shares being redeemed. In this case, the investor would receive $2,410 ($12.05 × 200 shares). LO 8.c

An investor wishes to open a cash account and give trading authorization to a sibling. The required documentation would include which of the following? A) A limited power of attorney B) CIP information on the sibling C) A joint account agreement D) A new account form signed by the investor

A If a new client wants to open a cash account and give trading authority to a third party, a limited power of attorney is required. A limited power of attorney gives the third party trading authority but prohibits that party from withdrawing assets from the account. Because this is a new account, a new account form is required, but it does not have to be signed by the client. CIP information on the new client is required, but not on the person with the POA. LO 1.d

The XYZ Corporation's A-rated convertible debenture is currently selling for 90. If the bond's conversion price is $40, what is the parity price of the stock? A) $36 per share B) $22.50 per share C) $44 per share D) $40 per share

A If the bond's conversion price is $40, it means the bond is convertible into 25 shares ($1,000 par value divided by the $40 conversion price). Parity means equal, so what does each share have to be worth so that 25 of them are equal to $900? Dividing $900 by 25 shares results in a parity price of $36. That does not mean the stock is selling for $36 per share (probably a bit less), but at $36, holding the bond or converting into the stock gives the investor equal value. Some students quickly see that the bond is 10% below its par value, so the stock, to be equal, must be 10% below the conversion price. Take 10% off $40 and the result is $36. Either way works. LO 5.d

In a negotiated municipal bond underwriting, all of the following are true except A) the underwriters may also be financial advisors to the municipality and receive both advisory fees and underwriting fees. B) the municipality appoints an investment banker or broker-dealer to underwrite the offering. C) the underwriter works with the issuer to establish the interest rate. D) the underwriter works with the issuer to establish the offering price.

A In a negotiated underwriting, the municipality appoints an investment banker or broker-dealer to underwrite the offering. The underwriter works with the issuer to establish the interest rate and the offering price in light of the issuer's financial needs and market conditions. Generally, those acting in the capacity of financial advisor to a municipality may not simultaneously act as underwriters. This is true of both negotiated and competitive bid underwritings. While the Municipal Securities Rulemaking Board rules do allow for certain exceptions, the fees collected would be limited to only those already agreed to for the advisory services and would not include any additional fees for performing underwriting functions or services. LO 6.h

The term wildcatting refers to A) drilling for oil or gas where none has occurred previously. B) buying new-construction real estate for speculative appreciation value. C) small-cap mutual fund diversification. D) limiting your investment portfolio to initial public offerings.

A In an oil and gas drilling program, the term wildcatting is used to describe the most speculative type of program, which is drilling where none has occurred before (i.e., in an unproven location). This is the riskiest of the oil and gas programs. LO 11.e

Under FINRA rules, members are prohibited from soliciting votes from limited partners in connection with a proposed rollup unless any compensation to be received by the member A) does not exceed 2% of the value of the securities to be received in the exchange. B) does not exceed 15% of the value of the securities to be received in the exchange. C) does not exceed 10% of the value of the securities to be received in the exchange. D) does not exceed 5% of the value of the securities to be received in the exchange.

A In connection with a DPP rollup, member firms may not solicit votes from limited partners unless the compensation is 2% or less. The 10% limitation is the maximum compensation in the sale of a DPP. The 15% limitation is the maximum percentage of the gross proceeds of a DPP that may be used for the organization and offering expenses. The 5% is likely an attempt to make you think about the FINRA 5% markup policy. That does not apply to DPPs. LO 11.h

Your client's position is long 100 MNO purchased at 90. Which of the following strategies will limit the customer's loss to $700? A) Long one MNO 90 call at 4, long one MNO 90 put at 3 B) Short one MNO 90 call at 4, short one MNO 90 put at 3 C) Buy a MNO 90 call at 7 D) Sell a MNO 90 call at 7

A It is the long put in this straddle position that limits the maximum loss on the long stock position. If the MNO stock drops to $0, the customer loses $9,000 on the long stock position but retains the right to sell the stock to someone at $9,000, to prevent loss beyond the premium of $300. The call would expire out of the money, for a total loss of $700. LO 10.f

An investor purchases 100 shares of JKL common stock at a price of $42 per share on April 22, 2018. On June 27, 2019, JKL's market price is $51 and the investor liquidates the position. Which of the following transactions made on October 17, 2018, would have an effect on the investor's tax treatment of this gain? A) Buying a Feb 45 JKL put B) Buying a Feb 45 JKL call C) Selling a Feb 45 JKL put D) Selling a Feb 45 JKL call

A Long-term capital gains tax rates are available when one has a holding period of more than 12 months. Although this investor held the JKL stock for more than 14 months, the purchase of the put caused the holding period to be tolled (the IRS term for suspended). It means that the holding period from April 22 to October 17 (almost 6 months) is put on hold until the put is disposed of or expires. When that happens in February, the "clock" picks up where it left off and runs another 4+ months until June 27. The total time period is approximately 10 months, less than the 12 months required for long-term treatment. None of the other positions affects the holding period of a long stock position. LO 10.i

Which of the following risk factors would be least important to disclose in recommending collateralized mortgage obligation (CMO) securities to public customers? A) Credit risk B) Extended payment risk C) Interest rate risk D) Prepayment risk

A Most CMOs offered to the public are backed by mortgages held by government-sponsored corporations like Fannie Mae, Ginnie Mae, Freddie Mac, et cetera. Credit risk would be a minimal consideration. The other risks are inherent to mortgage-backed securities. LO 12.d

A quotation on a municipal security between dealers is assumed to be A) a bona fide quote. B) an indication of interest. C) a nominal quote. D) a workable quote.

A Municipal bond quotations between dealers are required to be bona fide, or firm, quotes. They are required to be fair and reasonably related to the current market. LO 6.a

An investor in an oil and gas limited partnership program is subject to the economic consequences of all of the following except A) nonrecourse loans. B) recourse loans. C) depreciation on tangible assets. D) operating losses.

A Nonrecourse loans only have economic consequences for investors in real estate programs. LO 11.f

Investors in zero-coupon corporate bonds would find all of the following to be true except A) the discount must be accreted annually with taxation deferred until maturity. B) the discount is in lieu of periodic interest payments. C) the bond's duration is equal to its length to maturity. D) the discount must be accreted and is taxed annually.

A On a corporate zero-coupon bond, the discount is accreted on an annual basis and investors receive a Form 1099-OID indicating the amount of taxable accretion earned for the year. That is one of the reasons why these bonds are favored for tax-sheltered accounts, such as an IRA, or for UTMA accounts where the child's income might be very low. One of the important characteristics of these bonds is that their duration is equal to the length to maturity. This gives them a longer duration than coupon bonds of the same length and is the reason for their greater price volatility. LO 5.e

Under the Uniform Transfer to Minors Act (UTMA), how can stock subscription rights be handled in a custodial account? A) The custodian can exercise or sell the rights as he deems prudent. B) The rights can be exercised or sold only if the custodian is also the donor. C) The custodian can exercise, sell, or allow the rights to expire as he deems prudent. D) The custodian cannot exercise rights; they can only be sold.

A One thing that is never considered prudent is to let the rights expire. Even if the custodian does not believe adding more of the stock to the account is proper, there is a value to the rights, and the best interest of the minor is served by turning those rights into cash. Custodians in these accounts are able to sell or exercise the right, regardless of any relationship existing between them and the donor. LO 1.b

Which of the following investment companies registered under the Investment Company Act of 1940 can include senior securities in its capital structure? A) Closed-end management investment companies B) Face-amount certificate companies C) Open-end management investment companies D) Unit investment trusts

A Only the closed-end company is legally permitted to issue senior securities (preferred stock and bonds). LO 8.b

All of the following statements regarding the 5% markup policy are true except A) a riskless transaction is not generally covered by the 5% markup policy. B) the markup policy does not apply to securities sold at a specific price and with a prospectus. C) the type of security is a factor to consider. D) a transaction in common stock customarily has a higher percentage markup than a bond transaction of the same size.

A Riskless transactions are covered by the 5% markup policy. LO 3.j

Which of the following securities can generate phantom income? A) TIPS bonds B) Treasury bonds C) Treasury notes D) Treasury bills

A TIPS bonds adjust the principal value each six months based on the inflation rate. If the inflation rate is positive, the value increases. Those increases are reported as income each year even though the investor does not receive the appreciation until the bonds mature (or are sold). LO 7.a

An incorporated business model that allows flow-through of business income and losses directly to shareholders in order to avoid double taxation is A) an S corporation. B) a C corporation. C) a limited partnership. D) a general partnership.

A The S corporation, the general partnership, and the limited partnership are business models where all income or loss flows through to the owners. This avoids the double taxation on the business level and owner level, as is the case with the C corporation. With C corporations, corporate earnings taxed once at the business level and again when they are paid out to shareholders as dividends. Because the question is asking about the incorporated business model, the correct choice is the S corporation. LO 1.c

A customer buys 200 XYZ at 32, 2 XYZ JUN35 calls at 3, and 1 XYZ JUN 35 put at 6.50. Two months later, the customer purchases 1 XYZ JUN 35 put at 4. Before expiration, with XYZ trading at 37, he sells his stock and closes his calls at 2.10 and his puts at 0.25 for A) a loss of $180. B) a gain of $180. C) a loss of $450. D) a gain of $450.

A The customer opens four positions with debits to his account: 200 shares at $32 per share equals a debit of $6,400; two calls at $300 each equals $600; one put at $650 equals a debit of 650; and finally, an additional put at $400. The stock position is sold for $37 per share for a credit of $7,400. The calls are closed for 2.10 each (a credit of $420), and the puts are closed for a credit of $25 each. LO 10.d

Which of the following would be of least concern to a registered representative recommending a municipal security to a customer? A) Availability of the security B) Customer's state of residence C) Municipal security's rating D) Customer's tax status

A The customer's state of residence and tax status are essential when determining suitability of a municipal security. The security's rating is also important because it measures the bond's safety and quality and should align with the customer's risk tolerance. While the availability may pose a challenge for the broker-dealer and could potentially add to the cost of the transaction, it would be of the least concern regarding suitability unless the cost was in some way prohibitive. LO 6.c

An investor purchases an original issue discount general obligation municipal bond (OID) on the offering and holds it to maturity. The IRS treats the accretion of the discount as A) tax-free interest income. B) long-term capital gain. C) taxable interest income. D) short-term capital gain.

A The discount on an OID municipal bond is considered the bond's interest. Because interest on a GO municipal bond is tax free, when that interest is ultimately paid at maturity date, the tax treatment is the same as if interest was paid semiannually. There is a more complicated situation when the OID bond is purchased in the secondary market or when regular municipal bonds are purchased at discount in the market. LO 6.f

Your customer is interested in buying call options on CDL common stock. The client asks you, "Who issues CDL options?" The proper response is A) the Options Clearing Corporation. B) the exchange where the option is traded. C) the seller of the option. D) CDL Corporation.

A The issuer and guarantor of the options covered on the exam is the Options Clearing Corporation (OCC). Unlike other derivatives, such as rights and warrants, a corporation does not issue options on its own stock. Please do not confuse this with employee stock options, which is a different topic. As the guarantor, the OCC guarantees that the writer (seller) of the option will perform. That is, if exercised on a call, the stock will be delivered at the strike price, and if exercised on a put, the seller will pay the strike price. LO 10.b

A commonly used investment to provide a defined sum in the future, such as for college education or retirement, is A) zero-coupon bonds. B) convertible bonds. C) warrants. D) common stock.

A The key to this question is the defined sum. Zero-coupon bonds are usually purchased at a deep discount, which helps a small initial deposit grow into a substantial sum at maturity. Common stocks can't promise a specific sum when it is time for college. Warrants simply give the holder an opportunity to purchase the specified stock in the future; they have no defined value. Although convertible bonds with a maturity at the target date will provide the defined sum, the internal compounding of the zero-coupon bond will generally provide a higher return. LO 5.a

When an investor opens a new account at a member firm, FINRA rules require A) the signature of the principal signifying that the account has been accepted. B) the applicant's Social Security number. C) the applicant's date of birth. D) the applicant's signature.

A The only signature required on the new account form for an individual client is the signature of the partner, officer, or manager (a registered principal) denoting that the account has been accepted in accordance with the member's policies and procedures for acceptance of accounts. It is the customer identification program (CIP) that requires the date of birth and Social Security or tax ID number. All FINRA requires is a statement that the applicant is of legal age. FINRA states that each member shall also make reasonable efforts to obtain, prior to the settlement of the initial transaction in the account, the applicant's Social Security or tax ID number. Please notice that the question is differentiating between what is "need to know" and what is "nice to know." LO 1.d

An investor is looking to add some fixed-income securities to their portfolio. A registered representative suggests either the ABC 6s of 2050, or the XYZ 6s of 2043. Should there be an increase to market interest rates, A) the ABC bonds will suffer a price decline greater than the XYZ bonds. B) the XYZ bonds will suffer a price decline greater than the XYZ bonds. C) the XYZ bonds will enjoy a price increase greater than the XYZ bonds. D) the ABC bonds will enjoy a price increase greater than the XYZ bonds.

A This is a basic duration problem. When interest rates change, the bond with the longest duration will have the greatest price change. When there are two bonds with the same coupon rate (6%), the bond maturing latest has the longest duration. That tells us that the ABC bonds will fluctuate more than the XYZ bonds. Then, we need to remember that when interest rates increase, bond prices fall. That means that while both bonds will decline in price, the decline of the ABC bonds will be greater. LO 4.e

If a U.S. corporation exports machine tools to Switzerland and will be paid in Swiss francs (SF), to protect against foreign-exchange risk, it should A) buy SF puts. B) buy SF calls. C) sell SF calls. D) sell SF puts.

A What is the concern of this corporation? If, when the Swiss company pays for the tools, the value of the SF has fallen against the U.S. dollar, the company will receive less money. For example, if the bill is for 100,000 Swiss francs and, at the time of the sale, that amount of SF is worth $110,000 (each SF is worth $1.10), a decline in the value of the SF to $1.05 means the exporter will receive $105,000 instead of the expected $110,000. To hedge against a decline value (regardless of the asset), we buy puts. In the case of exporters from the Unitred States, they buy puts on foreign currency. There are two important acronyms to remember, EPIC and IPEC. Referring to U.S. based companies, Exporters buy Puts and Importers buy Calls. Because there are no options on the U.S. dollar, the Swiss company should buy calls on its own currency to hedge In other words, EPIC works in reverse if the question is dealing with a foreign company. Then it is IPEC for Importers buy Puts and Exporters buy Calls. LO 10.g

Other than the 52-week bills, the U.S. Treasury conducts auctions for Treasury bills A) weekly. B) monthly. C) only when the U.S. Treasury Department deems it necessary. D) bimonthly.

A With the exception of the 52-week bills, T-bills are auctioned by the U.S. Treasury weekly. The 52-week T-bills are auctioned every four weeks (which is not the same as monthly). LO 7.b

Which of the following plans are covered by ERISA? A. A defined benefit plan offered by a manufacturing company to employees B. A pension plan offered to the employees of the City of Detroit, MI C. A deferred compensation plan offered to select employees of a CPA firm D. A 457 plan

A a defined benefit plan offered by a manufacturing company to employees is covered by ERISA. Only private sector plans are covered by ERISA no state or local gov. 457-state plan

To find the maximum market value to which a short sale position can increase before a maintenance call is issued

Total credit balance/ 130%

Life Annuity with Period Certain

A Life Annuity that guarantees to provide income payments for a minimum period of time or life. Payments will continue to a beneficiary should the annuitant die during the specified period. -3rd smallest chec

Pass-Through Certificate

A certificate that represents a portion of ownership in a pool of federally insured mortgages -pass through refers to the mechanism of passing homebuyers interest and principal payments from mortgage holder to the investors -Fannie Mae, Freddie Mac, and Ginnie Mae do this

an investment company product that trades on an exchange and may trade at a premium or discount to NAV would be a? A. closed-end IC b. UIT c. open-end IC d. FAC company

A close end

treasury stock

A corporation's own stock that has been reacquired by the corporation and is being held for future use. -can reissue it or retire it -no dividends and no voting rights

Under the conduit theory of taxation, which of the following statements are true? A fund is not taxed on earnings it distributes if it distributes at least 90% of its net investment income. Investors are not taxed on earnings they reinvest. A fund is only taxed on interest income. Investors are taxed on earnings they receive in cash.

A fund is not taxed on earnings it distributes if it distributes at least 90% of its net investment income. Investors are taxed on earnings they receive in cash. By qualifying as a regulated investment company (the conduit, or pipeline, tax theory), the fund is liable only for taxes on retained income if it distributes at least 90% of its net investment income to shareholders. Investors will pay taxes on distributed income, whether it is received in cash or reinvested. LO 8.f

Index Fund

A mutual fund that tries to match the performance of a particular index by investing in the companies included in that index. -lower management costs than other types of funds

*Recourse Loan

A note or loan signed by a limited partner, for which the signer is personally responsible (at risk). Recourse loans are included in the partner's basis for tax purposes.

credit agreement

A part of an overall margin agreement that discloses the terms of the credit extended by the broker dealer.

Which of the following statements regarding municipal securities quotations are true? A quotation can be an indication of interest. A quotation cannot be an indication of interest. A quotation can be a one-sided request for a bid or offer (bids wanted and offers wanted). A quotation cannot be a one-sided request for a bid or offer (bids wanted and offers wanted).

A quotation can be an indication of interest. A quotation can be a one-sided request for a bid or offer (bids wanted and offers wanted). Municipal Securities Rulemaking Board rules pertaining to quotations cover all bona fide bids and offers, including one-sided requests for bids wanted and offers wanted, which are considered indications of interest. LO 6.a

qualified plan

A retirement plan that meets the IRS guidelines for receiving favorable tax treatment. -tax deferred

Targeted Amortization Class (TAC)

A type of CMO tranche that provides protection from refinancing risk through the use of support tranches. These support tranches absorb prepayment in excess of a targeted amount up to a specified point where the TAC may experience a shorter maturity schedule??????

Blank Check Company/Special purpose acquisition companies (SPAC)

A type of company designed to be used by private corporations intending to issue publicly traded shares through "reverse mergers" without the high expenses involved in making their own initial public offering. Blank check companies often have few assets, engage in little business activity, and have no business plan or experienced management. -discloses very little or non information on what companies they are investing in

On September 1, an investor sold 100 shares of KLP Corporation common stock for a loss of $1 per share. On September 15, he purchased a KLP convertible bond with a conversion price of $40. How much of the original loss may he now declare for tax purposes? A) $75 B) None C) $40 D) $100

A) $75 Because he purchased the convertible bond less than 30 days after realizing the loss, the sale of the stock falls under the wash sale rule: investors who sell securities at a loss and repurchase them, including their equivalents, 30 days before or after the sale will have the loss disallowed by the IRS. With a conversion price of $40, the bond could be converted into 25 shares (1,000 ÷ 40) of KLP common stock. Hence, the investor has bought back the equivalent of 25 shares and may only declare a $75 loss, as the remaining $25 loss will be disallowed. LO 13.h

ZOO is trading at 50.63. Your customer, who owns 100 shares of the stock, places an order to sell ZOO at 50.25 stop limit. The tape subsequently reports the following trades: ZOO 50.63 50.75 50.13 50.17 50.27 Your customer's order could first be executed at A) 50.27. B) 50.75. C) 50.13. D) 50.17.

A) 50.27. The sell stop limit order is elected (triggered) at the first trade of 50.13; when the stock trades at or below the stop price of 50.25, the order becomes a sell limit order at 50.25. The order can be executed at that price or higher (the limit placed by the customer). The next trade reported after the trigger is reached is below the limit price. The order could be executed at the next trade of 50.25 or higher, and that is the trade at 50.27. LO 16.a

All sell orders must indicate whether they are long or short. In which of the following cases would the sell order be marked long? A) An investor has purchased the stock being sold but the trade has not yet settled. B) An investor owns debentures convertible into the stock to be sold but has not tendered them. C) An investor has a call option to buy the stock to be sold but has not exercised the option. D) An investor owns warrants to buy the stock to be sold but has not yet exercised them.

A) An investor has purchased the stock being sold but the trade has not yet settled. A customer is considered long a security only if she owns the stock or has entered into an unconditional contract to buy the stock and will deliver the shares; owns convertible securities, has converted them, and will deliver; or owns an option, has exercised it, and will deliver. Otherwise, the sale is marked as a short sale on the order memorandum. LO 16.c

A customer is long 650 shares of DEF stock trading at $32 per share in a margin account, and the debit balance in the account is $9,200. If DEF pays a 10% stock dividend, what will the effect be on the customer's account? A) The equity will remain the same. B) The equity will increase. C) The market value will increase. D) The debit balance will be reduced.

A) The equity will remain the same.

Shareholder approval is required for all of the following corporate events except A) dividends. B) the issuance of convertible bonds. C) the acceptance of a tender offer from a nonaffiliated company. D) stock splits.

A) dividends. Shareholder approval is not required for the payment of dividends but is normally required for actions that increase (or potentially increase) the number of shares outstanding, such as stock splits and the issuance of convertible bonds. A corporation's acceptance of a tender offer requires shareholder approval. LO 14.b

All of the following statements regarding a mark to the market are true except A) it requires the use of a due bill. B) it often occurs in connection with margin transactions. C) it occurs because of a change in the stock's market value covered by a contract. D) it may result in a request for additional collateral.

A) it requires the use of a due bill. A mark to the market occurs when one party to a contract becomes partially unsecured due to a change in the stock's market value covered by a contract. A mark to the market is a request for additional collateral. LO 16.d

Earlier in the day, you entered a customer order to buy 300 XYZ at 26.45 good til canceled (GTC). By late afternoon, you notice that XYZ is trading at your customer's limit price. At the close of trading, you contact the order desk and get a Nothing Done report because A) of stock ahead. B) of the small size of the order. C) of the normal time delay between execution and execution reports. D) the order was canceled at the close of trading.

A) of stock ahead. All limit orders stand in time priority. LO 16.a

In portfolio theory, the alpha of a security or a portfolio is A) the difference between the expected return of the portfolio, given the portfolio's beta, and the actual return the portfolio achieved. B) a measurement of a portfolio's performance versus a standard benchmark such as the S&P 500. C) the risk of the portfolio associated with the factors that affect all risky assets. D) a measure of the variance in returns of a portfolio divided by its average return.

A) the difference between the expected return of the portfolio, given the portfolio's beta, and the actual return the portfolio achieved. Alpha is the difference between the expected return of the portfolio, given the portfolio's beta, and the actual return the portfolio achieved. The higher the alpha, the better the portfolio has done in achieving excess or abnormal returns. The risk of the portfolio associated with the factors that affect all risky assets is systematic risk. LO 13.b

All of the following are required by the Municipal Securities Rulemaking Board on customer confirmations except A) the price the dealer originally paid for the bond. B) the amount of any commission received on an agency transaction. C) the source of any commission received on an agency transaction. D) the amount of markdown or markup on a principal transaction.

A) the price the dealer originally paid for the bond. The dealer's cost of a bond sold to a customer is not required to be disclosed. LO 15.a

Stop orders may be used for each of the following except A) to lock in a specific price to close out a position. B) to establish positions. C) to protect profits on short positions. D) to protect profits on long positions.

A) to lock in a specific price to close out a position. Stop orders are contingent orders that are triggered when the stock trades at or through a stated price. When triggered, they become market orders to buy or sell. They are used by technical traders to establish positions above or below resistance and support levels, respectively. Stop orders never guarantee a specific execution price. LO 16.a

Which of the following statements regarding SMA balances is true? A)SMA balances may be withdrawn provided the withdrawal does not bring the account below minimum maintenance. B) SMA balances, which are considered a line of credit, may only be used to purchase additional securities. C) SMA balances are free credit balances available to be withdrawn upon demand. D) SMA balances may be withdrawn without restriction regardless of the account status being below or brought below minimum maintenance by the withdrawal.

A)SMA balances may be withdrawn provided the withdrawal does not bring the account below minimum maintenance. SMA is a line of credit that may always be withdrawn (even in a restricted account), provided the withdrawal does not bring the account below minimum maintenance. LO 16.d

which of these risks would be reduced most by diversification? A. business risk B. interest rate risk C. inflation risk D. market risk

A. business risk

S&P's bond rating

AAA AA A BBB

Moody's Bond rating

Aaa Aa A Baa

Sell limit orders are placed

Above the current market

Many life insurance companies offer variable products. Determining benefits usually depends on the actual performance of the selected separate account subaccount(s) compare to an assumed interest rate (AIR). Which of the following statements reflects that determination? Actual performance compared to the AIR affects the cash value of a variable life insurance policy Actual performance compared to the AIR affects the death benefit of a variable life insurance policy Actual performance compared to the AIR affects the value of an accumulation unit of a variable annuity Actual performance compared to the AIR affects the value of an annuity unit of a variable annuity

Actual performance compared to the AIR affects the death benefit of a variable life insurance policy Actual performance compared to the AIR affects the value of an annuity unit of a variable annuity When the actual performance of the separate account exceeds the AIR, the death benefit of a variable life insurance policy will increase. When the performance is less than the AIR, the death benefit reduces, but never below the guaranteed minimum. There is no assumed interest rate for the cash value. That is, the insurance company makes no projections as to its growth. With variable annuities, it is the annuity unit where the performance versus the AIR is important. In order to set up lifetime payments, the insurance company makes certain assumptions about returns. If the returns are higher, the value of the annuity (payout) unit increases and vice-versa. During the accumulation period, there are no assumptions; the insurance company never projects how much the money will grow. LO 9.c

Life Annuity/straight life

An annuity that provides a periodic income to the annuitant during his lifetime. A straight Life Annuity has no beneficiary and is considered to be the riskiest type of annuity. The annuitant is betting that he will live a long time, but the insurer is betting he is going to die. -unused funds goes to the issuer -highest payout

A customer is short a DMF 50 call for which he received a premium of 4. Seven months later, the call was exercised when the current market for DMF was 56. Under the Internal Revenue Code, what were the proceeds of his sale? A) $5,600 B) $5,400 C) $5,000 D) $4,600

B He wrote a call and received a premium of 4. He later sold the security at $50, which made his total receipts for the stock $54. Proceeds in this case refers to the total amount he took in (a $400 premium plus $5,000 upon the sale). LO 10.i

deadline for contributions

April 15

Treasury Notes (T-Notes)

Are a type of government bond issued with shorter maturities than T-bonds (intermediate) Typically offered to investors with two-, three-, five-, seven- or ten-year terms Interest rates are lower than those offered to T-bond investors T-note investors receive a six months interest payment, same as the T-bonds investors

Diversified management company

As defined by the Investment Company Act of 1940, a management company that meets certain standards for percentage of assets invested. These companies use diversification to manage risk. Related item(s): management company; 75-5-10 test.

The basis of a bond with a 5% nominal yield maturing in twenty years and selling at 85 is approximately A) 5.88%. B) 6.22%. C) 4.59%. D) 5.75%.

B A bond's basis is its yield to maturity (YTM). It is not necessary to do the YTM calculation because it could only be one choice. We can easily compute the current yield by dividing the $50 annual interest by the $850 current market price. That is about 5.88%. The YTM must be higher than that because it includes the eventual profit realized when the bond matures at par. There is only one selection that is higher than 5.88%. The calculation would follow our formula of: Annual interest + (discount ÷ number of years to maturity) ÷ (Current market price + par) ÷ 2) Plugging in the numbers, we have: ($50 + ($150 ÷ 20 years) = ($50 + $7.50) divided by ($850 + $1,000 ÷ 2) = $57.50 ÷ $925 = 6.22% LO 4.e

An investor buys two ABC Nov 50 calls, three ABC Dec 45 calls, and one ABC Jan 50 call. The best way to describe the portfolio is that it consists of A) six options of the same series. B) six options of the same class. C) two options of one class, three of another class, and one of a third class. D) six options of the same type.

B A class of options is when they are all of the same type (in this case, calls) and all on the same underlying security (in this case, ABC). Yes, they are all of the same type, but, in a question like this, FINRA is asking for the most specific answer. Same class is more specific than same type. The same series would be if they all had the same expiration date and exercise price. LO 10.a

On a single day, a customer purchases 15 TPL Sep 50 puts at 6 and 15 TPL Sep 50 calls at 1. If the price of TPL is $45 per share and the customer has no other security positions, what is this position called? A) Covered B) Straddle C) Spread D) Combination

B A long straddle is the purchase of a call and a put on the same stock with the same strike price and expiration. A straddle differs from a combination in that the strike prices and/or the expiration dates on a combination are different. A spread is a long put and a short put or a long call and a short call, rather than a put and a call. LO 10.f

The performance of the XYZ Growth Fund has been in the top 1% of all funds in its category for the past 1-, 5-, and 10-year periods. Which of the following would be the biggest risk factor to an investor investing in this fund? A) A dividend yield of less than 2% B) The manager's tenure is six months C) Past performance is no assurance of future results D) Lack of diversification in the portfolio

B Although one cannot predict the future from the past, when a portfolio manager has consistently been ranked at the top, it is not considered a major risk to bet on a winner. The problem here is that almost all of that performance was achieved under the direction of previous management. With only six months on the job, the new manager is untested and there is no way to know how the future performance will rank. You might see this referred to as tenure risk. Diversification is one of the benefits, not risks, of a mutual fund. In a growth fund, one does not expect a high dividend yield. LO 8.i

One of the most important roles played by registered representatives is making suitable recommendations to their customers. Doing that requires gathering as much information about the customers as possible. Which of the following factors would likely be the least important when dealing with a couple in their late twenties with two children? A) Education goals for the children B) Expected retirement age C) Current employment stability D) Values

B Although saving for retirement is the single most common investment objective, determining an expected retirement age for a couple this young is unrealistic—it is just too far away to make an accurate determination. Meeting the children's educational needs is something that needs to be addressed now. Knowing the reliability of the family's income stream is critical for financial planning. Selecting investments matching the customers' attitudes is necessary to ensure that their values are being met.

A client interested in the returns offered by CMOs asks you which type has the lowest prepayment risk. What should you say? A) Z-tranche B) PACs C) TACs D) Plain vanilla

B Although there can be exceptions, in general, the planned amortization class (PAC) has the lowest prepayment risk. The Z-tranche is the most unpredictable because it is paid off only after all of the other tranches. LO 12.d

Your new customer lists tax-free income as an investment objective but notes that he will need access to $50,000 within the next four to six months for a down payment on a vacation home he is purchasing. To meet the objective of tax-free income, a registered representative considers municipal securities for the $50,000. Which of the following municipal securities recommendations would be the least suitable? A) A tax anticipation note (TAN) B) An auction rate security (ARS) C) A variable rate demand note (VRDN) D) A bond anticipation note (BAN)

B An ARS is a long-term instrument tied to short-term interest rates and, therefore, would not be suitable for someone with a short-term time horizon. Each of the remaining answer choices are short-term notes aligning better with the customer's need to access the funds in the next four to six months. LO 6.b

For U.S. investors holding American depositary receipts (ADRs), dividends received are A) taxed as a capital gain in the United States. B) subject to a foreign withholding tax. C) tax free in both the country of origin and in the United States. D) tax free in the country of origin.

B Any tax taken on dividends received from ADRs is taken in the country of origin. This is a foreign withholding tax for U.S. investors. The foreign withholding tax may later be taken as a credit against any U.S. income taxes owed by the U.S. investor. LO 3.g

All of the following statements regarding collateralized mortgage obligations (CMOs) are true except A) principal repayments are applied to earlier tranches first. B) interest is paid semiannually. C) interest payments are distributed pro rata when received. D) CMOs are a derivative security.

B CMO holders are paid interest monthly. As payments are received from the underlying mortgages, interest is paid pro rata to all tranches, but principal repayments are paid to the first tranche until it is retired. Subsequent principal repayments are then applied to the second tranche until it is retired and so on. CMOs are a derivative security because the value of each tranche is derived from the timing of principal repayments to that tranche. LO 12.d

A registered representative has a customer looking to invest in stock for income. The customer is looking for the highest fixed rate of return available based on her risk profile. Which of the following would be least suitable? A) Cumulative preferred B) Convertible preferred C) Straight preferred D) Callable preferred

B Convertible preferred stock is convertible into the issuer's common stock. This conversion feature has value if the market price of the underlying stock should increase. Because of that feature, issuers are able to attract investor interest with a lower dividend on this preferred stock compared with preferred stock that has no conversion feature. Therefore, it would be the least suitable investment for this client. LO 3.e

An experienced investor wants to allocate 10% of an existing portfolio to real estate but does not want to maintain properties, be a landlord, or wait if cash is needed. Which of the following choices would be suitable, given the investor's objectives? A) Purchase property and assign a management company B) Purchase shares of an equity REIT C) Purchase shares of mortgage companies D) Purchase shares of a mortgage REIT

B Equity real estate investment trusts (REITs) are a way to invest in real estate without having to own or manage properties. REITs trade on exchanges and over the counter; therefore, they are liquid investments. Both of these characteristics meet the investor's objective and make equity REITs the most suitable recommendation of those offered here. LO 11.a

If ABC Corporation reports a loss for the year, it is obligated to pay interest on all of the following except A) convertible bonds. B) adjustment bonds. C) variable rate bonds. D) debentures.

B Even if a corporation reports a loss, the corporation is obligated to pay interest on all of its outstanding debt except for income (adjustment) bonds. Adjustment bonds require interest to be paid only if ABC has sufficient earnings and the payment is declared by the board of directors. LO 5.a

A customer has invested a total of $10,000 in a nonqualified deferred annuity through a payroll deduction plan offered by the school system where she works. The annuity contract is currently valued at $16,000, and she plans to retire. On what amount will the customer be taxed if she chooses a lump-sum withdrawal? A) $10,000 B) $6,000 C) No taxes owed because annuity was nonqualified D) $16,000

B Explanation Payments into a nonqualified deferred annuity are made with after-tax money; taxes must only be paid on the earnings of $6,000. LO 9.d

Interest received from which of the following federal agency securities is exempt from all state and local taxation? A) Government National Mortgage Association pass-through securities B) Farm Credit System bonds C) Federal National Mortgage Association bonds D) Federal Home Loan Mortgage Corporation bonds

B FCS securities are part of a small group of federal agency securities where the interest is exempt on a state and local (but not federal) level. A key to remembering is that any agency with the title mortgage is fully taxable!!!! LO 7.e

FINRA Rule 2310 defines a direct participation program as "a program which provides for flow-through tax consequences regardless of the structure of the legal entity or vehicle for distribution including, but not limited to, oil and gas programs, real estate programs, agricultural programs, cattle programs, condominium securities, Subchapter S corporate offerings and all other programs of a similar nature, regardless of the industry represented by the program, or any combination thereof." The rule places limits on the amount of broker-dealer sales compensation considered fair and reasonable. That limit is A) 15% of the gross proceeds. B) 10% of the gross proceeds. C) 5% of the gross proceeds. D) 2% of the gross proceeds.

B FINRA limits the amount of the sales compensation to 10% of the gross proceeds of the offering. If the organization and offering expenses exceed 15% of the gross proceeds, FINRA considers that too high. The 2% is the maximum charge in a DPP rollup if the firm wishes to solicit votes from the limited partners. The 5% is the FINRA markup policy and that does not apply to DPPs. LO 11.h

Which of the following securities would most likely have the lowest expense ratio? A) Balanced mutual fund B) Exchange-traded fund C) Variable annuity D) Closed-end fund Explanation

B Generally, most exchange-traded funds (ETFs) have a lower expense ratio than comparable mutual funds. Variable annuity expense ratios tend to be higher than mutual funds, and the expense ratio for closed-end funds is similar to that of open-end (mutual) funds. LO 8.h

Which of the following is not a characteristic of hedge funds? A) They are privately organized and generally unregistered. B) They offer managers high fixed fees. C) They use leverage, short positions, and concentrated positions. D) They invest in private securities, real assets, derivatives, and structured products.

B Hedge funds attempt to attract the top managers because they offer performance-based fees, which vary based on fund performance. The typical fee structure is 2% + 20%, where 2% is the fixed fee and 20% of the profits is the performance portion. LO 12.b

A customer who seeks to supplement his retirement income and has a high risk tolerance would find which of the following securities most suitable? A) Municipal GOs B) High-yield bond funds C) Investment-grade bond funds D) Treasury STRIPS

B High-yield bonds yield more than investment-grade bonds. Because the client has a high risk tolerance, these bonds are more appropriate than investment-grade bonds, which yield less. Not only do the Treasury STRIPS provide zero income, but they certainly are not suitable for those with a high risk tolerance. Similarly, the municipal GO bonds are generally quite safe and, at least for test purposes, municipal bonds are never a suitable investment unless the investor is in a high tax bracket. LO 8.g

A customer wrote 10 KLM Jun 80 calls for a premium of 4.75 at a time when the market value of KLM was 81.75. What is his gain or loss if he now closes out his positions at 2.12? A) A $2,630 loss B) A $2,630 gain C) A $4,750 gain D) A $4,750 loss

B If the customer sold at 4.75 and purchased at 2.12, he nets 2.63, which is multiplied by 100 to yield a $263 gain per contract: 10 × $263 = $2,630 total gain. LO 10.h

The market attitude of a customer who establishes a credit call spread is A) bullish. B) bearish. C) neutral. D) speculative.

B In a call spread, a customer is buying one call and selling another with different strike prices and/or expirations. In any spread, one of the options is dominant. In a short call spread, the short call position is dominant because it has the higher premium; writing calls is bearish. LO 10.e

In a scheduled premium variable life insurance policy, all of the following are guaranteed except A) the right to exchange the policy for a permanent form of insurance, regardless of health, within the first 24 months. B) a minimum cash value. C) the ability to borrow at least 75% of the cash value after the policy has been in force at least three years. D) a minimum death benefit.

B In a variable life insurance policy, a minimum death benefit is guaranteed, but no cash value is guaranteed. There is a contract exchange privilege during the first 24 months, allowing the conversion of the variable policy to a comparable form of permanent insurance, and the 75% cash value loan minimum applies after the third year of coverage. LO 9.a

In a discretionary account where the investment objective is preservation of capital with moderate income, all of the following practices are unsuitable except A) frequent and profitable short-term trading in volatile stocks. B) maintaining a fixed asset allocation mix, which includes some underperforming sectors. C) marking order tickets solicited or unsolicited when discretion is used. D) marking the investment objective on the new account form as high risk.

B In some test questions, the best way to select the correct choice is when three of the four options are clearly wrong. This is an example of that case. Preservation of capital is certainly not a high-risk objective and does not call for frequent trading in any stock, volatile or not. Orders in a discretionary account are not considered unsolicited (the client is not the one placing the orders). Allocating the portfolio to fixed-income assets (bonds and preferred stock) would seem to be the most appropriate step to take. LO 2.g

An investor purchases $10,000 worth of Treasury bills on November 27 and holds them until they mature on March 30 of the following year. For purposes of taxation, the interest from those Treasury bills is treated as A) partially ordinary income and partially capital gain. B) ordinary income subject to federal income tax. C) a short-term gain. D) tax-free income.

B Interest on Treasury bills, notes, and bonds is taxable as ordinary income at the federal level. It is exempt from state and local taxation. LO 7.e

The interest on which of the following municipal securities may be considered preference income for alternative minimum tax purposes? A) TANs B) Private purpose bonds C) Original issue discount bonds D) PHAs

B Interest on private activity municipal bonds is included in the taxable income of an investor who is subject to the alternative minimum tax. LO 6.f

Which of the following debt securities would be most likely to offer a conversion feature into common stock? A) A mortgage bond B) A debenture C) Preferred stock D) Commercial paper

B Invariably, when it comes to convertible debt securities, they are debentures rather than secured bonds. The conversion concept makes no sense with money market securities—they mature in one year or less. Preferred stock is often convertible, but it is an equity security, not a debt security. LO 5.c

There are certain securities offerings that are limited to those who meet the definition of accredited investor. The SEC requires that the issuer shall take reasonable steps to verify that purchasers of securities sold in those offerings are accredited investors. One way in which this may be accomplished for natural persons is obtaining a written confirmation from certain persons or entities that such person or entity has taken reasonable steps to verify that the purchaser is an accredited investor within the prior three months and has determined that such purchaser is an accredited investor. Confirmation from which of the following would not meet the SEC's requirements? A) A licensed attorney who is in good standing under the laws of the jurisdictions in which he or she is admitted to practice law B) An investment adviser registered and in good standing under the laws of the state of its principal office C) A registered broker-dealer D) A certified public accountant who is duly registered and in good standing under the laws of the place of his or her residence or principal office

B It is only investment advisers registered with the SEC, not the state(s), for whom the written confirmation of their accredited investor status is acceptable. LO 2.c

Many options traders give attention to the VIX. The VIX A) rises when call buying on the SPX (the S&P 500) increases. B) rises when put buying on the SPX (the S&P 500) increases. C) tends to rise during periods of high investor confidence. D) was created by the Options Clearing Corporation (OCC).

B The VIX is an indication of volatility. When there is more put buying than call buying, the VIX increases. Conversely, more call buying than put buying results in a decrease to the VIX. It is often called the "fear" index and, in fact, had its highest reading during the Covid-19 crisis of 2020. It was created by the CBOE, not the OCC. LO 10.g

Opening a margin account involves a number of different documents. The document describing how the interest on the margin debt is calculated is generally known as A) the hypothecation agreement. B) the credit agreement. C) the risk disclosure document. D) the loan consent agreement.

B It is the credit agreement, sometimes referred to as the margin agreement, that describes the creditor-debtor relationship. This includes the method of computing interest on the debit balance (the amount owed). The hypothecation agreement allows the broker-dealer to maintain possession of the margined securities as collateral for the loan, and the loan consent agreement allows the broker-dealer to lend out the client's margined securities. The risk disclosure document is provided to make sure the client understands the risks of margin trading. LO 2.g

Investors who are subject to AMT must have which of the following preference items added to adjusted gross income to calculate their tax liability? A) Distributions from a corporate bond mutual fund B) Interest on a private purpose municipal bond C) Interest on a municipal bond issued to finance highway construction D) Income from a municipal security issued to finance parking garages

B On the exam, whenever you see a private purpose municipal bond, the interest on the bond is a tax preference item for the purpose of the alternative minimum tax. LO 6.f

A customer opens an account, and payment and delivery instructions are established. Beyond the opening of the account, these instructions may A) be changed at any time for all transactions going forward. B) be changed for individual transactions, or going forward, for all transactions. C) not be changed unless a new account is established. D) be changed for individual transactions only.

B Once payment and delivery instructions are established at the time the account is opened, they can be changed for any individual transaction or for all transactions going forward. LO 1.a

All of the following statements regarding planned amortization class (PAC) collateralized mortgage obligations are true except A) PACs have companion tranches. B) PACs have higher yields than comparable TACs. C) PACs have a more certain maturity date than comparable TACs. D) PACs have a lower-than-average prepayment risk.

B PACs have two companion tranches: one to absorb prepayments and one to buffer against extension risk. Because there is less risk and a more certain maturity date, PACs tend to have lower yields than comparable TACs. LO 12.d

Disregarding commissions, and investor purchasing $10,000 face amount of Treasury notes at a price of 98.12 would expect to pay A) $983.75. B) $9,837.50. C) $981.20. D) $9,812.00.

B Please note that the purchase is not for $1,000, but for $10,000. Treasury notes (and bonds) are quoted in 32nds. This quote of 98.12 is 98 12/32 or 98 3/8% of $10,000. LO 7.b

Your customer owns a leveraged ETF having a performance goal of 200% of the underlying index. When purchased two days ago, the ETF was priced at 50. If the index was down 10% the first day and up 20% the second day, what is the value of the ETF today if it performed as it was intended to? A) 45 B) 56 C) 36 D) 54

B Priced at 50 when purchased, after the first day's 10% decrease in the index, the 2× leveraged ETF would be down 20% (20% × 50 = 10-point decrease) to 40. After the second day's 20% increase in the index, the 2× leveraged ETF would be up 40% (40% × $40 = 15-point increase) to 56. On the calculator provided at the testing center, you could arrive at the correct answer by taking the original 50, multiplying times 80% (the 10% decrease is doubled means the stock is only 80% of what it was) and then multiplying that result by 140% (2× the 20% increase). LO 8.h

Services offered by prime brokers include all of the following except A) supplying clearing services. B) complying with FINRA's advertising rules. C) providing back office support. D) processing transactions.

B Prime brokers provide services primarily to institutional investors. They have nothing to do with that institution's advertising. They do supply clearing services, lending services for marginable transactions, as well as back office support including cash management, account statements and transaction processing. LO 1.b

A couple's home has an assessed value of $40,000 and a market value of $100,000. What will the tax be if a rate of 5 mills is used? A) $500 B) $200 C) $5,000 D) $2,000

B Real property tax is based on the assessed value assigned to the property by the municipality's tax assessor (in this case, $40,000). Property tax rates use the mill as a base unit. One mill equals $1 of tax per year for each $1,000 of assessed value. Five mills would equal $5 for each $1,000 of assessed value. Because there are 40 thousands, 40 × $5 equals $200 in annual tax. A shortcut method is as follows: take the assessed value, remove the last three 0s, and multiply by the number of mills of tax ($40 × 5 mills = $200). LO 6.c

Among the requirements of Regulation SP is that a broker-dealer must provide an initial and annual privacy policy statement to A) retail consumers. B) retail customers. C) shareholders. D) business customers.

B Regulation SP deals with protection of client information. Initially, and annually, the firm's privacy policies must be disclosed to its retail (individual) customers. Remember, consumer has a special meaning under SP. It is a one-time relationship, such as someone who received stock as a gift and wants to sell it through the member firm, take the money, and conduct no further dealings. Business and institutional clients are not covered by the regulation. LO 1.d ONE TIME CONSUMER RECURRING CUSTOMER

Which of the following is an example of sovereign debt? A) Bank of England notes B) U.S. Treasury bonds C) Royal Bank of Canada CDs D) Sony Corporation debentures

B Sovereign debt represents loans to governments. On the exam, it is likely that the examples will be foreign governments, not U.S. Treasury securities. The Royal Bank of Canada is a privately owned corporation and its debts are not those of the Canadian government. Bank of England notes are the paper currency issued (e.g., the ₤10 and ₤20 notes). LO 4.d

A corporation has an outstanding issue of 8% convertible debentures with a conversion price of $25. The bond indenture contains an antidilutive clause guaranteeing the debt holders the right to maintain proportionate equity conversion in the corporation. If the company pays a 10% stock dividend to its common shareholders, how will that affect the debenture holders? A) They will receive four shares of the common stock. B) The bonds will now be convertible at approximately 22.73. C) Each debenture holder will receive a check for $100. D) The interest rate on the debentures will increase to 8.8%.

B The antidilutive provision means the debenture holders will be able to convert into an equivalent share value as before. With a conversion price of $25, the bond is convertible into 40 shares ($1,000 ÷ $25). After the 10% stock dividend, they should be able to have 10% more shares, or 44 shares. That means the conversion price must be reduced. Divide $1,000 by 44 shares and the result is $22.73. Remember, anytime there is a stock dividend, prices go down. LO 5.c

A customer establishes the following positions: Buy 100 ABC at 28 Buy 1 ABC Dec 25 put at 2 What is the breakeven point? A) 26 B) 30 C) 23 D) 27

B The breakeven point is where an investor neither makes nor loses money. In this hedged position, the buyer must recover the cost of the stock and the premium paid to break even (28 + 2 = 30). Please note that the call up and put down rule does not apply when there is a stock position. LO 10.h

A customer purchases an ABC 6½% convertible preferred stock at $80. The conversion price is $20. If the common stock is trading 2 points below parity, the price of ABC common is A) $12. B) $14. C) $18. D) $16.

B The conversion ratio is computed by dividing par value by the conversion price ($100 par ÷ $20 = 5). Parity price of the common stock is computed by dividing the market price of the convertible by the conversion ratio ($80 ÷ 5 = $16). $16 − 2 = $14. LO 5.d

All of the following statements regarding industrial revenue bonds (IRBs) are true except A) they can be issued by municipalities to build facilities that will be owned by the municipality but leased to a local corporation. B) the credit rating of the bonds is dependent on the credit rating of the municipality. C) interest is paid from rental payments received from corporations that have leased the property or equipment from the municipality. D) they can be issued by municipalities to provide local industries with funds for expansion.

B The debt service for IRBs is derived from the lease payments made by the leasing corporation to the issuing municipality. Therefore, the credit rating of the bonds is dependent on the credit worthiness of the leasing corporation, not the issuing municipality. LO 6.b

An investor acquires limited partner status in a direct participation program when A) his money is received by the general partner. B) he and the general partner have both signed the subscription agreement. C) he submits a signed copy of the subscription agreement. D) the certificate of limited partnership is filed in its home state.

B The investor must sign a copy of the subscription agreement, but he is not considered a limited partner until the agreement is also signed by the general partner indicating acceptance of the limited partner. LO 11.c

Who has the final responsibility for debt service on an industrial revenue bond? A) The municipal authority established by the issuer B) The corporation leasing the facility C) The municipal issuer of the bonds D) The MSRB

B The issuer of industrial revenue bonds is a municipality or an authority established by a municipality. However, no municipal assets or general revenues are pledged to secure the issue. The net lease payments by the corporate user of the facility are the source of revenue for debt service. Therefore, the ultimate responsibility for the payment of the principal and interest on an industrial revenue bond rests with the corporate lessee. LO 6.b

Limited partners in a real estate partnership have all of the following rights except A) the right to sue the general partner for violating the partnership agreement. B) the right to decide which properties the partnership purchases. C) the right to monitor the partnership on an ongoing basis. D) the right to receive their pro rata share of income or loss.

B The limited partners have the right to inspect partnership records and the right to sue a general partner who acts outside the partnership agreement. The general partner normally sets her own compensation in the original agreement and makes all management decisions relative to the partnership's interests. LO 11.d

In a variable life annuity with 10-year period certain, a contract holder receives A) fixed payments for 10 years, followed by variable payments for life. B) a minimum of 10 years of variable payments, followed by additional variable payments for life. C) variable payments for 10 years, followed by fixed payments for life. D) 10 years of variable payments.

B The owner of a life annuity with 10-year period certain will receive payments for life, subject to a minimum of 10 years. If the contract holder dies before the period expires, the remaining payments are made to the beneficiary. An example would be if a contract holder of a life annuity with 10-year period certain died after 5 years, payments would continue for 5 more years to the beneficiary and then stop. LO 9.b

A corporation pays a 10% stock dividend to common stockholders. All the following are true regarding this dividend except A) the total value of the position is unchanged when the dividend is paid. B) the beauty of stock dividends is that they are nontaxable. C) the dividend is taxable in the year the sale of the shares takes place. D) the cost basis per share is adjusted based on the stock dividend.

B The stock dividend is taxable, but unlike cash dividends, which are taxed when received, stock dividends are taxable in the year the shares are sold. When the stockholder receives the additional shares, the cost basis is adjusted on a per-share basis with the total value of the position remaining unchanged. For example, if an investor owned 100 shares purchased at a price of $22 per share and the company paid a 10% stock dividend, the numbers would look like this. The number of shares owned is now 110 [100 + (10% of 100)] = 100 + 10. The adjusted cost basis per share (used when any of the shares are sold) is now $20 per share. The original cost is $2,200 (100 shares times $22 per share). After the stock dividend, the customer owns 110 shares, but there was no additional cost. Divide that original $2,200 by the new number of shares ($2,200 divided by 11) to arrive at an adjusted cost basis of $20 per share. The account value is still $2,200 (110 shares times $20 per share = $2,200). LO 3.i

FINRA Rule 2330 deals with a member's responsibility in the sale of certain insurance company-based products. Specifically the concern is with A) taking loans against the cash value of a variable life insurance policy. B) purchases and sales of deferred variable annuities. C) subsequent changes to the separate account allocations in a deferred variable annuity. D) purchases and sales of deferred annuities.

B This FINRA rule applies to purchases and sales of deferred variable annuities and initial allocations to the separate account. It does not apply to fixed annuities or variable life insurance. Because it only deals with the purchase or sale or initial allocations, a client with a deferred variable annuity wishing to change the separate account allocation does not come under the rule. LO 9.e

FINRA Rule 2111 places three obligations on members when determining if a specific recommendation to a customer is suitable. FINRA's suitability rules would likely find a registered representative is not in violation of complying with those three if A) control relationships were disclosed. B) the recommendation made would be suitable for at least some customers. C) the recommendation was profitable for the investors. D) proper disclosures were made of the representative's compensation received.

B This question refers to the three specific obligations under Rule 2111. Those three are reasonable-basis suitability, customer-specific suitability, and quantitative suitability. Complying with the first of the three means the registered representative has to have a reasonable basis to believe that a recommendation is suitable for at least some investors. Control relationships must always be disclosed, but that is not part of the three obligations. Compensation may have to be disclosed, but, once again, that is not part of the three obligations. Be sure to focus on answering the question being asked. LO 2.f

Which of the following details would not be found on the bond resolution for a revenue bond? A) The maintenance covenant B) The tax covenant C) The rate covenant D) The insurance covenant

B Unless something in the question refers to special taxes, revenue bonds do not have tax backing. The other items are included in the bond resolution (or trust indenture). The rate covenant is a promise to maintain rates sufficient to pay expenses and debt service. The maintenance covenant is a promise to maintain the equipment and facility/facilities. The insurance covenant is a promise to insure any facility. LO 6.b

A customer owns a 7.5% ABC convertible bond currently trading at 115. The conversion price is $40. What is the parity price of the common? A) $28.75 B) $46.00 C) $44.00 D) $34.00

B What does parity price mean? Here is what it says in the LEM: Calculating Conversion Parity Parity means that two securities are of equal dollar value (in this case, a convertible bond and the common stock into which it can be converted). The question is looking for the parity price of the common stock. That is the market price per share, where the total value of the stock received upon conversion equals the market price of the bond. There are two ways to do this. The first is generally the easiest to understand. We are told that the bond has a conversion price of $40. That means you can get 25 shares if you wish to convert. That is because the issuer is basically saying, "We owe you $1,000 and will let you spend it on our stock at $40 per share." Now that we know we can get 25 shares, what does each share have to be worth to equal $1,150? If you divide $1,150 by 25 shares, the result is $46. The other method to do this is as follows: The bond is selling at a 15% premium. To be equal to that, the stock must be selling at a 15% premium over the conversion price. $40 times 115% equals $46. If that makes sense to you, it is much faster than the first method. LO 5.d

A bond convertible at $50 is selling at 105% of parity, while the common stock has a current market value of $45. What is the market value of the bond? A) $1,000 B) $945 C) $1,045 D) $900

B When a bond is convertible at $50, it means the holder can exchange each $1,000 par value bond for the company's common stock at a rate of $50 per share. Dividing $1,000 (always use the par value, not the market value) by $50 results in a conversion rate of 20 shares per bond. With the bond convertible into 20 shares and the market price of each share currently $45, the parity price, the price at which the value of the stock and the bond are the same, is $900, (20 x $45). The question tells us that the bond is selling for 105% of the parity price. That would be $900 x 105% = $945. An alternative method is to recognize that the stock is selling for 10% below its conversion price ($45 is $5 less than $50 and $5 ÷ $50 = 10%). That means the parity price of the bond must be 10% below the par value, or $900 (which is 10% less than $1,000). Once you have the $900, multiply by 105% to arrive at the correct answer of $945. LO 5.d

Advertisements for the Abstemious Balanced Fund (ABF) describe the investment as a no-load fund. In order to make this claim, the fund must A) not have a front-end load in excess of 0.10% B) not have a conditional deferred sales charge C) not have a 12b-1 charge in excess of 0.75% D) have its first breakpoint no higher than $10,000

B When a fund promotes itself as a no-load fund, not only must there be no front-end load, there cannot be a back-end load (CDSC) either. The 12b-1 charge maximum is 0.25%. The concept of breakpoints applies solely to Class A shares (front-end load). LO 8.d

A municipal bond, issued with a covenant that states, "If revenue collections are not sufficient to meet debt service requirements, the issue will be backed by the full faith and credit of the municipality," is known as A) a contingent liability bond. B) a double-barreled bond. C) a Section 8 bond. D) a moral obligation bond.

B When a municipal bond is backed by both a source of revenue and the taxing ability of the issuer, this is referred to as a double-barreled bond. LO 6.b

From time to time, an investor's situation arises where they may need to liquidate a portion of the portfolio. It could be a medical need, an emergency repair, or a joyous event such as a wedding. Getting the necessary cash would be most difficult from which of the following holdings? A) A unit investment trust B) A DPP C) A mutual fund D) A listed option

B When it comes to liquidity, at least for the exam, DPPs rank near the bottom of the list. Mutual funds and UITs are redeemable by the issuer, and closing a position in an option contract settles the next day. LO 11.g

When does a customer have to receive the options disclosure document? A) Within five business days of the first options trade B) At or before the first order C) Within 15 days of account approval by the firm's registered options principal D) With the confirmation of his first options transaction

B When opening an account to trade options, the owner must be told about the risks involved with trading options. By providing the owner with an options disclosure document entitled Understanding the Risks and Uses of Options, the broker-dealer satisfies the risk disclosure requirements. Furthermore, no member or person associated with a member shall accept an order from a customer to purchase or write an option contract, or approve the customer's account for the trading of options unless the broker-dealer furnishes or has furnished to the customer the ODD, and the customer's account has been approved for options trading. The 15-day rule applies to the customer's need to return the options account agreement. LO 10.j

Which of the following individuals could most likely open an account at a FINRA member firm without notifying or receiving permission from their employer? A) A life insurance agent who sells variable annuities B) An individual who sells only fixed annuities C) A purchases and sales clerk of a member D) A government security trader employed by a member

B Whenever an employee of a FINRA member wants to open a securities account with another FINRA member firm or financial institution, the employee must give prior written notice to his employer and receive prior written consent from his employer before the account can be opened. Someone selling fixed annuities only is most likely not associated with a member firm, while anyone selling variable annuities must be. LO 1.d

When speaking to a customer about exchange-traded funds (ETFs), a registered representative could accurately state that these funds A) can be purchased only by paying a sales charge added to the net asset value. B) do not have the same potential tax consequences as mutual funds, such as making capital gains distributions annually. C) cannot be purchased using traditional limit or stop orders. D) cannot be bought on margin.

B With ETFs, portfolio turnover rate is minimized because they do not have to buy and sell shares within their portfolio to accommodate shareholder purchases and redemptions. This can affect the potential tax consequences. While an ETF can make a capital gains distribution, they generally do not—unlike a mutual fund, which generally would make such distributions on an annual basis. ETFs can be traded like other exchange products using traditional stock trading techniques and are priced by supply and demand. Customers pay commissions, not sales charges. LO 8.h

Lindsey Wolfe, a public school teacher, has been contributing to a 403(b) TSA plan for the past 20 years. Contributions total $50,000 and the current value is $200,000. Wolfe is still teaching full time for the school system. When does Wolfe have to begin taking required minimum distributions? A) At age 59½ B) At age 72 or when no longer working for the school system, whichever is later C) At age 72 D) Required minimum distributions are never required for annuities

B Wolfe is invested in a qualified annuity. Therefore, the minimum distribution requirements are the same as for any qualified account. RMDs must begin at age 72 but can be postponed as long as continuously employed by the same employer. Unless qualifying for an exception, any withdrawals from a qualified annuity before reaching age 59½ are taxed as ordinary income with the additional 10% penalty. LO 1.h

A corporate bond is quoted in the Wall Street Journal as follows: Bid: 100½ Asked: 100¾ Bid Chg.: -⅛ Yield: 5.75 From this information, you know the nominal yield is A) 5.75%. B) greater than 5.75%. C) less than 5.75%. D) 5.625%.

B YIELD=YTM=BASIS The bid and asked prices show that the bond is being quoted at a premium (above par), with a yield to maturity of 5.75%. When bonds are trading at a premium, the nominal yield (coupon rate) is greater than the yield to maturity. LO 4.e

An investor seeking current income would find which of the following LEAST suitable? A. callable bond B. convertible debenture C. equipment trust certificate D. mortgage bond

B convertible bond has reduced income

An investor purchases a TIPS bond with a 3% coupon. During the first year, if the inflation rate is 8%, the principal value of the security at the end of that year will be closest to A) $1,030.00. B) $1,081.60. C) $1,080.00. D) $1,030.23.

B the principal value of a TIPS bond is adjusted semiannually by the inflation rate. The exact calculation would be $1,000 × 104% × 104%, which equals $1,081.60. Each six months, the interest is paid on that adjusted principal and that is why the security keeps pace with inflation. There is a shortcut that will always work on the exam. Just recognize that the principal value increases based on the inflation rate compounded semiannually. Take the simple interest rate and choose the next highest number. In this example, 8% simple interest would be $80 (which would always be one of the choices). Because the computation is done twice per year, the compounding effect makes the correct choice slightly higher. LO 7.a

Which of the following is not a characteristic of certificates of deposit (CDs)? A) A CD may be payable to the bearer or registered in the name of the investor. B) The Federal Deposit Insurance Corporation (FDIC) provides insurance for CDs to $500,000. C) A CD is often issued by a bank. D) A CD can be negotiable or nonnegotiable.

B $250,000

The Bond Buyer compiles several indexes of municipal bonds. Which of the following is limited to bonds with the highest ratings? A) The Revdex 25 B) The 11 Bond Index C) The 20 Bond Index D) The 40 Bond Index

B) The 11 Bond Index This index takes the highest rated (AA or better) bonds from the 20 Bond Index of bonds with A ratings or better. LO 13.f

In a new margin account, a customer buys 300 shares of ABC at $40 per share, 100 shares of the Ajax Mutual Fund at $24, and 10 PDQ Aug 30 calls at 4. The customer will receive a margin call for A) $4,400. B) $12,400. C) $10,800. D) $9,200.

B) $12,400. The customer must pay 50% of the value of the stock and 100% of the value of the mutual fund shares and the options because these securities are nonmarginable and require full payment. To calculate the total payment required, add $6,000 (50% of $12,000) plus $2,400 (Ajax) plus $4,000 (PDQ calls) to arrive at $12,400. LO 16.d

In a cash account, if a customer buys 300 XYZ at 48 and simultaneously writes 3 XYZ Jan 50 calls at 1, the customer must deposit A) $14,400. B) $14,100. C) $14,700. D) $14,550.

B) $14,100. In a cash account, the customer must deposit 100% of the value of the stock purchased ($14,400). However, to determine the actual deposit, subtract the $300 in premium income received. By depositing $14,100, the customer will have $14,400 in the account—the difference being the premium income credited to the account on settlement date. LO 16.d

A customer opens a new margin account, and the first trade is the short sale of 100 shares of ABC, at a price of $18 per share. What is the required margin deposit? A) $9,000 B) $2,000 C) $1,800 D) $900

B) $2,000 The minimum of $2,000 is never waived for short sale margin requirements. If the short sale calculation is less than $2,000 ($1,800 in this case), the customer is still required to deposit the $2,000 minimum. LO 16.d

A customer has the following accounts: Market value: long account $35,000; short account $40,000 Balance: long account (DR) $23,000; short account (CR) $60,000 SMA: long account $3,000 Regulation T: 50% What is the combined minimum maintenance requirement for the long and short margin positions? A) $18,750 B) $20,750 C) $12,750 D) $8,750

B) $20,750 The minimum maintenance for long accounts is 25% of the LMV (25% × $35,000 = $8,750). The minimum maintenance for short accounts is 30% of the SMV (30% × $40,000 = $12,000). In this case, $8,750 plus $12,000 equals a combined maintenance of $20,750. LO 16.d

If a customer fails to meet a Regulation T margin call of $2,500, securities may be sold out of the account with a value of A) $3,333. B) $5,000. C) $8,000. D) $2,500.

B) $5,000. Securities valued at twice the Regulation T cash call must be sold out if a customer fails to meet a Regulation T margin call ($2,500 × 2 = $5,000). LO 16.d

A broker-dealer can hypothecate (pledge) up to A) 50% of the equity balance in a margin account. B) 140% of the debit balance in a margin account. C) 25% of the debit balance in a margin account. D) 100% of the long market value (LMV) in a margin account.

B) 140% of the debit balance in a margin account. The amount of securities a broker-dealer can hypothecate (pledge) in a customer's margin account is equal to 140% of the account debit balance. LO 16.d

A customer, age 62, wants to retire at age 64 and has accumulated investments in an IRA currently valued at $500,000. The IRA portfolio consisting of all mutual funds is allocated as follows: 70% growth funds, 10% corporate bond funds, and 20% sector funds. Still wanting to use mutual funds, which might be the most suitable reallocation of the portfolio as this customer nears retirement? A) 80% broad market index funds, 10% corporate bond funds, 10% U.S. government bond funds B) 60% U.S. government bond funds, 30% broad market index funds, 10% growth funds C) 70% municipal bond funds, 30% broad market index funds, 10% sector funds D) 30% municipal bond funds, 30% corporate bond funds, 40% growth funds

B) 60% U.S. government bond funds, 30% broad market index funds, 10% growth funds Moving toward retirement, the reallocation should move the portfolio away from equities and sector funds toward fixed-income funds. U.S. government securities funds accommodate that, and the U.S. government securities within the fund are considered safe with no default risk. Coupling the U.S. government bond funds with smaller percentages in broad market index and growth funds that mirror the market can help the portfolio keep pace with inflation. Remember that utilizing municipal securities in a tax-favored account, such as an IRA, would be considered unsuitable because the interest paid by municipal bonds is already tax free. LO 14.a

Which of the following actions would increase SMA in a long margin account? A) A long purchase B) A long sale C) A stock dividend received D) A decline in CMV

B) A long sale Of these choices, an increase in SMA can only be accomplished by a sale of securities held long in the account. A purchase would decrease the SMA if it is used to make the purchase. Stock dividends have no effect on the balances in a long margin account; only the number of shares is changed. A decline in the CMV would not change the SMA. LO 16.d

A customer is very concerned about investments that may not keep pace with inflation. He asks which securities would have the least exposure to inflation risk. Which of the following would be the best answer? A) Fixed annuity B) Common stock C) Preferred stock D) Cash

B) Common stock The returns on common stock have historically outperformed inflation, making them less vulnerable to loss of purchasing power than the other choices presented. Cash is a store of present purchasing power that inflation will erode. Fixed annuities have more exposure to inflation than common stock because their payments are fixed in nominal dollars. Preferred stock has the same exposure to inflation risk as do all fixed-income instruments. LO 14.a

ABC Corporation raised capital through an offering of equity securities. Which component of the balance sheet has changed as a result? A) Current liabilities B) Current assets C) Fixed assets D) Long-term liabilities

B) Current assets When equity securities are issued, cash (a current asset) and net worth increase. Fixed assets and liabilities remain unchanged as a result of the offering. LO 13.d

Liquidity ratios measure the solvency of a firm, or the firm's ability to meet short-term financial obligations. Which of the following is a liquidity ratio? A) Gross profit divided by net sales B) Current assets divided by current liabilities C) Dividends divided by earnings per share D) Net income divided by average total equity Explanation

B) Current assets divided by current liabilities Current assets divided by current liabilities is the current ratio, a ratio that measures the liquidity of a firm. Gross profit divided by net sales is a profitability ratio that measures the gross profitability of the firm's business operations, not its liquidity. Net income divided by average total equity is the return on stockholders' equity, which measures the efficiency of common shareholders' investment or equity in the firm. Dividend amount divided by earnings per share is the dividend payout ratio, which measures how much of a company's earnings are distributed to common stockholders. LO 13.d

The Investment Company Act of 1940 contains a number of terms used to describe investment companies. When used as an adjective, the term diversified would apply to which type of investment company? A) Business development company B) Management company C) Unit investment trust D) Face-amount certificate company Explanation

B) Management Companies The Investment Company Act of 1940 divides investment companies into three principal classifications. Those are the face-amount certificate company, the UIT, and the management company. Management companies are further divided into open-end and closed-end companies. The act goes one step further and has management companies divided again into diversified and nondiversified companies. It is not expected that you will have to know what a BDC (business development company) is. LO 8.a 75-5-10 RULE DIVERSIFIED RULE

Fairweather Securities Corp. (FSC), a registered broker-dealer, has invited several IARs from Econometric Advisory Services (ESA), a registered invested adviser that directs transactional business to FSC, to a seminar featuring a disquisition on current economic trends being presented by a leading economist. It would be permitted for FSC to cover which of the following expenses? A) None of these; because ESA directs commission business to FSC, it would be an unethical business practice for FSC to pay any portion of the expenses B) Registration fees for the seminar C) Registration fees for the seminar plus travel expenses D) Travel and transportation fees, but not the seminar fee

B) Registration fees for the seminar Payment for seminar fees, but not travel and transportation expenses, is permitted under the safe harbor provisions in Section 28(e) of the Securities Exchange Act of 1934. LO 14.d

In general, FINRA rules prohibit member firms from improper use of customer funds. One example is intentionally holding up an account transfer. Another is holding on to funds that belong to the customer. One of the features of FINRA Rule 2165 dealing with senior exploitation is the ability of a member firm to place a temporary hold on disbursements from the account of a specified adult. This serves as a safe harbor for funds held in the manner described above. A member relying on this rule must A) report all temporary holds to FINRA within 15 days of the end of the month in which the hold took place. B) develop and document training policies or programs reasonably designed to ensure that associated persons comply with the requirements of this rule. C) place temporary holds on disbursements of funds or securities from the accounts of specified adults whenever there is suspected exploitation. D) segregate customer funds from those of the firm to avoid commingling of assets.

B) develop and document training policies or programs reasonably designed to ensure that associated persons comply with the requirements of this rule. In FINRA's eyes, this is all about making sure that associated persons of the firm are adequately (and frequently) trained. Although customer and firm assets must be segregated, that is not part of the senior exploitation rule. The rule permits, but does not require, that these holds be placed on disbursements from the affected accounts−it is voluntary. There is no reporting of this activity, but detailed reports must be made and retained, containing the relevant information leading to the decision to enforce the hold. LO 14.c

A client of a member broker-dealer is disgruntled with the attitude of some of the firm's staff. A complaint is sent by email to the manager of the branch servicing the account. FINRA rules require that a record of this complaint be maintained for no less than A) three years. B) four years. C) whatever time period the firm wishes, because the only time a complaint is recognized is if it is in writing. D) six years.

B) four years. FINRA rules require that written complaints (and a complaint filed electronically is considered written) be kept for a minimum of four years. This is the only four-year FINRA requirement you are apt to see on the exam. The MSRB rule on retention of customer complaints is six years so be sure to keep these two straight because both are testable. LO 15.e

A confirmation of each customer trade must be given or sent A) before the trade date. B) on or before the settlement date. C) on the trade date. D) before the settlement date.

B) on or before the settlement date. A confirmation must be sent to a customer on or before the completion of the transaction (the settlement date). LO 15.a

All of the following will affect the working capital of a corporation except A) an increase in current assets. B) payment of a cash dividend. C) a decrease in current liabilities. D) declaration of a cash dividend.

B) payment of a cash dividend. Working capital is defined as current assets minus current liabilities. On the declaration date, the future dividend payment is "booked" as a current liability (dividend payable). When the payment date comes, disbursement of the cash dividend will reduce current assets (cash) and current liabilities (dividend payable) by the same amount, leaving working capital unchanged. LO 13.D

A prospect is heavily invested in the common stock of an employer's company, ABC, relative to other investments. The stock has performed well over the last 15 years and the prospect is very happy with the investment. After reviewing financial and nonfinancial criteria, you have determined that A) he should begin to liquidate the ABC stock using the FIFO accounting method. B) selling a portion of ABC and using the proceeds to purchase mutual funds will reduce his nonsystematic risk. C) because ABC has performed well over a 15-year period, he should keep the stock but sell it if inside information indicates a fall in value is imminent. D) owning too much ABC stock has increased credit risk to an unacceptable level.

B) selling a portion of ABC and using the proceeds to purchase mutual funds will reduce his nonsystematic risk. This prospect is exposed to a significant amount of business (nonsystematic) risk, as indicated by the large investment in ABC common stock. Business risk can be reduced by diversifying the portfolio; therefore, recommending the sale of a portion of the ABC stock and using the proceeds to purchase mutual funds is suitable. There will be tax considerations, but the use of FIFO accounting will likely expose the prospect to higher capital gains taxes than other accounting methods and may not be the best approach to liquidation. LO 14.a

Marking-to-the-market is A) the calculation of the difference in value between the current market price and the Regulation T requirement. B) the revaluing of securities held long or short in the account based on the actual CMV of the securities. C) the revaluing of a margin account's equity based on the market price on the settlement date. D) the calculation of the difference between the maintenance margin and the Regulation T requirement.

B) the revaluing of securities held long or short in the account based on the actual CMV of the securities. Marking-to-the-market is the revaluing of an account based on the CMV of all securities positions in the account. In order to determine the necessity of additional maintenance calls or the availability of SMA, this revaluing is done at least once per business day in accounts that have margin balances. LO 16.c

An analyst reports that a stock's price is consolidating. This means A) no distinct pattern can be observed. B) the stock's trendline is moving primarily in a horizontal direction. C) the stock's trendline is moving primarily in a upward direction. D) the stock's trendline is moving primarily in a downward direction.

B) the stock's trendline is moving primarily in a horizontal direction. In general, when the trendline of a stock's market price is moving within a very narrow range (the chart is basically a pattern of horizontal movement), the technician views that as a consolidation. Within a relatively short time after the consolidation has been verified, it is expected the price will move. What isn't determined yet is if the movement will be up (bullish) or down (bearish). LO 13.e

Tax basis formula

Basis=investment in partnership + share of recourse debt (+ nonrecourse debt in real estate DPPs) - cash or distributions

serial maturity

Bonds within an issue mature on different dates according to a predetermined schedule. The sample serial maturity structure table below shows an example of $100 million State of Illinois GO serial issue. (See Sample Serial Maturity Structure on pg. 110) Serial bonds are quoted on the basis of their yield to maturity, called basis, to reflect the difference of maturity dates within one issue. A price/yield of 100% indicates the yield to maturity is equal to the coupon rate, which means the bond is being offered at par.

Buy order: if the firm acts as a principal it ______ the stock ________...

Buys the stock from the market maker at the offering or asking price & adds a markup to its cost

*Where are footnotes found?

Bottom of the financial statement & can be several pages long

Credit put spread is bullish or bearish?

Bull Spread

Debit call spread is bullish or bearish?

Bull Spread

positive beta means what?

Buy recommendation

Growth Style

Buy stocks whose earnings are growing faster than most other stocks. Some may think the stocks are overvalued.

Full POA

Buy/sell stock; Remove money from account; Remove securities from account.

Freeriding

Buying and immediately selling securities with- out making payment. This practice violates the SEC's Regulation T. penalty: account will be frozen for 90 days or until cash or marginable securities show up

Value Style

Buying undervalued or out of favor securities whose price is low relative to company's earnings or book value.

Sell order: if a firm is acting as a principal, it

Buys from the customer and sells to the market ma at the bid price

A corporation is having a rights offering. The terms of the offering require six rights plus $60 to purchase one share. With the stock's current market price at $74 per share, the theoretical value of one right before the ex-rights date is A) $2.33. B) $0.23. C) $2.00. D) $0.20.

C ($74 - $60) ÷ (6 + 1) = $14 ÷ 7 = $2.00.

In the trading of options, there are a number of different multiple option strategies. An investor has the following position: Buy one RIF Apr 120 call Buy one RIF Jul 130 put Which strategy is the investor using? A) Long straddle B) Time spread C) Long combination D) Diagonal spread

C A combination is composed of a long call and long put, or a short call and a short put, each having different strike prices and/or expiration months on the same underlying security. A straddle is when the expiration dates and exercise prices are the same. A spread consists of a long and short position in the same options class (two puts or two calls). In a diagonal spread, the exercise price and the expiration dates are different. In a time spread, everything is the same except the expiration dates. LO 10.f

Which of the following statements regarding a member firm's handling of a discretionary account is true? A) The registered representative must obtain written authorization from the customer before placing each order. B) Margin may not be used in a discretionary account. C) The registered representative may not effect transactions excessive in size or frequency in view of the customer's resources. D) A principal must approve each discretionary order before execution.

C A discretionary account allows the registered representative to place orders without consulting the customer. It does not relieve him of the obligation to execute only suitable orders. LO 2.g

An investor writing an XYZ Oct 50 put could cover that put with A) an XYZ Oct 45 put. B) an XYZ Jul 55 put. C) an XYZ Oct 55 put. D) an XYY Oct 60 put.

C A short put can be covered with a long put. The long put must have an exercise price the same or higher than the short put and an expiration date at least as long as the short put. The only choice meeting both criteria is the October 55 put. Two other ways to cover a short put is with short stock and cash. An XYY put will not cover an XYZ put. **This question deals with material not covered in your LEM, but it relates to recent rule changes and/or student feedback. LO 10.d

A municipal issuer is frequently able to diversify a single municipal bond issue by maturity because A) many municipal securities are very marketable. B) municipal securities are mostly long term. C) many municipal bonds are serial issues. D) every state issues municipal bonds.

C A way for a municipal issuer to potentially make the issue more attractive is to diversify by having a range of maturities. That way, the issue will appeal to those investors whose needs might be short term, immediate term, or long term. Serial maturity means that within a single issue, portions of the issue mature at intervals, some short term, others intermediate term, and the balance long term. Municipal bonds typically mature serially. LO 6.a

A customer purchases $100,000 of original issue discount municipal bonds. How will this trade be considered for tax purposes when the bonds mature? A) Taxable as short-term gain B) Fully taxable on capital gain C) No capital gain D) Taxable as long-term gain

C Original issue discount profit at maturity is treated as part of the tax-free interest on a municipal bond. However, for a municipal bond bought at a discount in the secondary market, the discount is considered ordinary income subject to tax. LO 6.f

Which of the following is not a right conferred upon ownership of common stock? A) Transferability of shares B) Dividends, if declared by the board of directors C) Limited liability D) Voting in person or by proxy

C Although ownership of common stock means the holder's maximum loss is limited to the original investment, it is not a stockholder right. The doctrine of limited liability is a legal construct and shields stockholders from being responsible for debts of the company. Being able to vote the shares; being able to sell them without needing the issuer's permission; and dividends, if declared, are considered rights of owning stock. LO 3.c

An investor has purchased 100 shares of common stock of the UOM Corporation. UOM Corporation is a Japanese company. Rather than receiving a UOM stock certificate, the investor receives an American depositary receipt (ADR). The investor calls his registered representative and wants to know why he did not receive the stock certificate. The registered representative tells the client that A) receiving the stock certificates would cost the investor more money. B) records of ownership in UOM stock is book-entry only. C) the ADR is a substitute for the stock certificate and represents the investor's ownership in the foreign corporation's stock. D) UOM stock certificates were not available.

C An ADR is a negotiable certificate that evidences an ownership interest the shares of a non-U.S. company that are on deposit with a foreign branch of a U.S. bank. It is similar to a stock certificate representing shares of stock. ADRs trade in U.S. dollars and clear through U.S. settlement systems, allowing ADR holders to avoid having to transact in a foreign currency. LO 3.g

Which of the following would have the least market risk? A) Corporate or municipal bonds with long-term maturities B) Fannie Maes C) Revenue anticipation notes D) AAA corporate debentures

C Anticipation notes are the shortest term, which gives them the least market risk (the risk that price will fluctuate during the time left to maturity). LO 6.b

Interest income from all of the following are exempt from state and local taxation except A) Series EE savings bonds. B) Treasury bonds. C) FNMA mortgage-backed issues. D) Treasury bills.

C As a general rule, the interest income from U.S. government and agency securities is subject to federal taxation only; it is generally exempt from state and local taxation. However, the interest income from mortgage-backed securities is fully taxable. LO 7.e

Trade confirmations must show yield to call on which of the following bonds? A) 6½%, 7% basis, maturing 2038 B) 5½%, 5½% basis, maturing 2038 C) 5½%, 5% basis, maturing 2038 D) 6½%, at par, callable 2025, maturing 2059

C Bond confirmations must disclose the lower of the yield to maturity (YTM) or yield to call (YTC). On a premium bond, the YTC is the lower of the two. The terminology here shows the coupon, the basis (YTM), and the maturity date (and, in one case, the call date). The 5½% bond with a 5% basis is the only bond trading at a premium because the YTM (or basis) is lower than the coupon. Even though the 6½% bond maturing in 2059 is callable relatively soon, because the bond was purchased at par, CY, YTM, and YTC are all equal to the coupon (nominal) rate, so the investor won't suffer a loss of principal with an early call. LO 6.h

In a strong bull market, which of the following positions utilizing leverage has the potential for the highest percentage gain? A) Selling short B) Writing puts C) Holding calls D) Holding stocks

C Both a long call and a long stock position are profitable in a rising market. However, because options use leverage, the profit relative to the money invested is larger with option positions. A put writer also profits in a rising market, but only by the amount of the premium. A short seller loses money if the stock rises. LO 10.d

A registered investment company whose share price fluctuates independently of its net asset value is most likely A) a unit investment trust. B) an open-end fund. C) a closed-end fund. D) an index mutual fund.

C Closed-end funds' share prices can differ significantly from their NAVs. Open-end (mutual) fund shares are purchased and redeemed at their NAVs. UITs are redeemable at NAV. LO 8.c

Which of the following would not be a valid use of the partnership democracy? A) Removing the general partner B) Consenting to a legal judgment against the partnership C) Deciding which partnership assets should be liquidated to pay creditors D) Consenting to an action of a general partner that is contrary to the agreement of limited partnership

C Deciding which partnership assets should be liquidated to pay creditors involves limited partners in the active management of partnership affairs. This would result in being treated as general partners with respect to liability and possible loss of limited partner status. LO 11.c

A double-barreled bond would be defined as A) a bond of a foreign issuer that is backed by the U.S. government, with the interest payable either in dollars or in foreign currency. B) a corporate bond that pays interest from ordinary income and revenue received from operating a facility for a municipality. C) a bond that has its principal and interest backed by revenues of a facility and the general taxing authority of a municipality. D) a bond that is exempt from both federal and state taxes.

C Double-barreled bonds have two sources of revenue to support them. They are municipal bonds. LO 6.b

Which of the following municipal bonds may be paid by a state's legislative apportionment of funds to service the debt? A) Special assessment B) Industrial development revenue C) Moral obligation D) Special tax

C Explanation If a moral obligation bond goes into default, bondholders do not have the right to sue to force a tax to pay off the bonds. The only way bondholders can recover the principal is through legislative apportionment. The issuer's legislative body has to appropriate funds to pay off the bonds. With a moral obligation bond, issuers have the moral, but not legal, obligation to service the debt. LO 6.b

A registered representative cannot adequately advise a client without knowing the client's financial status. When determining that status, it is important to differentiate between financial and nonfinancial considerations. Which of the following would be considered a financial consideration rather than a nonfinancial one? A) The client's membership in Greenpeace B) The fact that both parents were smokers who died of lung cancer C) The client's rare coin collection D) The client's marital status

C Financial considerations are those that can be categorized as an asset or a liability (something that can be assigned monetary value). Rare coins are certainly something of monetary value. The other choices are nonfinancial because you really can't put a number on them. The Greenpeace membership and the lung cancer deaths of the parents are likely indicators of certain investments that would probably not be suitable due to the values of the client. LO 2.b

A characteristic of hedge funds that would not be found in a mutual fund is A) professional management. B) the ability to be purchased on margin. C) a lock-up period. D) a diversified portfolio.

C Hedge funds generally employ a lock-up provision. This is to ensure that capital invested by shareholders will remain with the fund long enough for the manager to implement the intended fund strategy. There is no standard lock-up period; it can differ from fund to fund. It should always be noted that during the lock-up period, the investment is essentially rendered illiquid. Hedge funds and mutual funds have professional management and diversified portfolios. Although hedge funds can use margin in portfolio transactions, they, like mutual funds, cannot be purchased on margin. LO 12.b

An investor desiring a limited partnership investment with capital gains potential would most likely select one investing in A) shopping centers. B) oil and gas. C) raw land. D) equipment leasing.

C Historically, raw land has been a source of appreciation. Shopping centers might appreciate in value but are oriented more toward current income. Equipment leasing offers income, and the asset ultimately depreciates. Oil and gas provide income as well, and the asset ultimately depletes. LO 11.e

Bob Smith, who is in his 40s, has just become covered by an extremely generous defined benefit retirement plan at his company. He has decided he no longer needs his variable annuity for retirement purposes and wants to use the money for a trip to Africa. Over the past 10 years, he has invested $60,000 in the annuity, and its net value is now $80,000. If Bob should go ahead and surrender the annuity, the tax consequences will be A) capital gains tax on $60,000 and a $6,000 penalty. B) capital gains tax on $20,000 and a $2,000 penalty. C) ordinary income tax on $20,000 and a $2,000 penalty. D) ordinary income tax on $60,000 and a $6,000 penalty.

C If an annuity is cashed in, the growth and accumulation portion of its value ($20,000 in this case) is taxable as ordinary income. If the annuitant is under the age of 59½, he must also pay a 10% penalty on the growth withdrawn, a penalty of $2,000 in this case. LO 9.d

Which of the following choices would generate the largest first-year deductions in an oil and gas exploratory drilling program? A) Depreciation B) Recapture C) Intangible drilling costs D) Depletion allowance

C Intangible drilling costs (IDCs) would be the largest deduction in an oil and gas exploratory drilling program. These are also known as a wildcat program. This type of program attempts to discover oil or gas in an area where proven reserves have yet to be discovered. IDCs are deductible in the year incurred. The unpredictability and huge costs associated with drilling for oil and gas make this the largest deduction in an oil and gas wildcat program. The allowance for depletion comes into play only when the well begins producing and product is sold. Depreciation of the equipment can be taken in the early years, but it is not as significant as the full deduction for the IDCs. Recapture is not a deduction; it is reporting as income something previously deducted. LO 11.f

For both U.S. Treasury notes and Ginnie Maes, A) interest income is taxed at the federal level only. B) interest is computed on an actual-day basis. C) quotes are as a percentage of par in 32nds. D) settlement is next business day.

C Interest from U.S. T-notes is taxed at the federal level only, while interest on Ginnie Maes is taxed at all levels. GNMA bonds are treated like corporate bonds in many ways. T-notes settle next day, while Ginnie Maes normally settle T+2. Interest on T-notes is computed on an actual-day basis, and Ginnie Mae interest is computed on a 30-day month/360-day year basis. Both Ginnie Maes and T-notes are quoted in 32nds. LO 7.c

One of the major concerns of the regulatory bodies is the growing problem of senior exploitation. To combat this issue, at the time of an account opening, whether margin or cash, a member firm must disclose in writing that the member or an associated person is A) unable to further update the customer record with regard to a trusted contact person without written consent from the contact person. B) required to meet the requirements with respect to opening a new account, including obtaining the contact information of a trusted person for any account, retail or institutional. C) authorized to contact the trusted contact person provided on the new account form by the customer and to disclose information about the customer's account in addressing possible exploitation. D) unable to open the account in the absence of the name and contact information for a trusted contact person.

C Member firms must obtain the name and contact information for a trusted contact person who is 18 or older. This person may be contacted about the customer's account. This is required for retail but not institutional accounts. At the time of the account opening, the member firm must disclose in writing to the customer that the member or an associated person is authorized to contact the trusted contact person. Contacting is done to disclose information about the customer's account to address possible exploitation. The absence of the name and contact information for a trusted contact person does not prevent a member from opening or maintaining an account for a customer, as long as the member makes a reasonable effort to obtain this information. LO 1.d

Three family members each hold sizable call option positions with the same underlying equity security in their individual accounts. Over the course of three days (Monday through Wednesday), each of the customers calls your broker-dealer and gives instructions to exercise all of their call options in that security. You recognize this as a potential violation of A) front running rules. B) the Code of Procedure. C) Options Clearing Corporation (OCC) exercise limit rules. D) the Uniform Practice Code.

C OCC exercise rules limit the maximum number of contracts in the same underlying security that can be exercised within a five-business-day period. Three customers—all related and all giving instructions to exercise their long calls in the same underlying security within three business days—should, at a minimum, raise the question of whether or not they are acting in concert to circumvent the OCC exercise limit rules. LO 10.j

Programs allowing for the direct pass-through of losses and income to investors include all of the following except A) oil and gas drilling direct participation programs. B) new-construction real estate direct participation programs. C) real estate investment trusts (REITs). D) S corporations.

C REITs allow for the direct pass-through of income, but not losses. The other choices are forms of business that allow for pass-through of income and losses. LO 11.b

All of the following are true of real estate investment trusts (REITs) except A) they must invest at least 75% of their assets in real estate-related activities. B) shares are publicly traded. C) they must pass along losses to shareholders. D) they must, to qualify under Subchapter M, distribute at least 90% of their net investment income.

C REITs engage in real estate activities and can qualify for favorable tax treatment if they pass through at least 90% of their net investment income to their shareholders. While they can pass through income, they cannot pass through any losses; they are not direct participation programs. LO 11.b

The Securities Exchange Act of 1934 applies to all of the following except A) registration of broker-dealers. B) the extension of credit on purchase of securities. C) regulation of new issues. D) secondary market trading.

C The Securities Act of 1933 deals with new issues. The Securities Exchange Act of 1934 created the SEC, required the registration of broker-dealers, empowered the Federal Reserve to control the extension of credit on securities transactions, and created rules dealing with secondary market trading. LO 3.h

All of the following statements about targeted amortization class (TACs) CMOs are correct except A) they have high extension risk. B) in exchange for higher price risk, they generally offer a slightly higher interest rate. C) they are the most volatile of the tranches. D) they have transferred prepayment risk to companion tranches.

C The Z-tranche, just like a zero-coupon bond, is the most volatile. TACs transfer the prepayment risk to companion tranches like PACs do, but they retain extension risk. This causes greater price risk in the market that is compensated for with a somewhat higher interest rate. LO 12.d

If a customer writes 1 Jul 80 put at 7, and the put is exercised when the market price is at 70, for tax purposes, what is the effective cost basis of the stock put to the seller? A) $87 B) $70 C) $73 D) $80

C The cost basis is 80 (the price at which the writer must buy) minus 7 (the premium the writer was paid), or $73 per share. LO 10.i

When a corporation issues a debt security, the terms of the loan are expressed in a document known as the bond's deed of trust. The deed of trust is sometimes referred to as A) the debenture. B) the bond resolution. C) the indenture. D) the loan agreement.

C The indenture, sometimes also referred to as the deed of trust, states the issuer's obligation to pay back a specific amount of money on a specific date. A debenture is a debt security containing an indenture. Bond resolution is a term used for municipal bonds, not corporate debt. LO 4.a

If TCB is trading at 43 and the TCB Apr 40 call is trading at 4, what are the intrinsic value and the time value of the call premium? A) Intrinsic value: 3; time value: 4 B) Intrinsic value: 4; time value: 0 C) Intrinsic value: 3; time value: 1 D) Intrinsic value: 1; time value: 3

C The option is in-the-money by 3 points because the strike price is 40 and the market price is 43. This sets a minimum premium of $3 per share. Because the actual premium is 4, the balance of 1 represents time value. The premium, minus the intrinsic value, equals the time value. This is true whether the option is a put or a call. LO 10.c

A customer buys five municipal bonds maturing in 20 years for 104. If he sells the bonds after 10 years at 103, the customer has A) a $100 capital gain. B) a $100 capital loss. C) a $50 capital gain. D) a $50 capital loss.

C The premium on the municipal bonds must be amortized. The tax rules require that when you purchase a bond at a premium, you have to reduce the cost basis of the bond each year. Even though there are five bonds in the question, here's the math on one bond and then we'll multiply by five to get the total amount. The investor buys the bond at 104 or $1,040 and the bond is due to mature in 20 years. Take the $40 premium divided by the 20 years to maturity and that will tell us the amount that we amortize/reduce the cost basis by each year. $40/20=$2. It then tells us that the bond is sold after 10 years. Ten years of amortization is $2 per year x 10 years = $20. That lowers the basis of the bond to $1,020 ( $1,040 - $20 = $1,020). The bonds are sold at 103 or $1,030, so the gain is $10 per bond times five bonds for a total gain of $50. LO 6.e

alll of the following statements about variable annuities are true except A) such an annuity is designed to combat inflation risk. B) the rate of return is determined by the underlying portfolio's value. C) a minimum rate of return is guaranteed. D) the number of annuity units becomes fixed when the contract is annuitized.

C The return on a variable annuity is not guaranteed; it is determined by the underlying portfolio's value. Variable annuities are designed to combat inflation risk. The number of annuity units becomes fixed when the contract is annuitized; it is the value of each unit that fluctuates. LO 9.c

An investor owns ten ABC 6s of 2045. The debentures have a conversion price of $50 with an anti-dilution provision. After ABC distributes a 20% stock dividend, the investor's position will be A) ten ABC 6s of 2045 convertible into 16.67 shares. B) ten ABC 6s of 2045 convertible into 20 shares plus forty additional shares. C) ten ABC 6s of 2045 with a conversion price of $41.67. D) twelve ABC 6s of 2045 with conversion price of $50.

C This question deals with the anti-dilution provisions of a convertible security. When there is a stock dividend or a stock split, the holder of the convertible maintains the same equity proportion as before. With a conversion price of $50, the debenture is convertible into 20 shares ($1,000 ÷ $50). After a 20% stock dividend, the holder should be able to acquire 20% more shares. That makes the security convertible into 24 shares. Divide the $1,000 par value by 24 shares and the conversion price is now $41.67. LO 5.c

XYZ Widgets is a publicly traded corporation. Upon the death of one of the founders of the company, a donation of 100,000 shares of XYZ stock is made by the executor of the deceased's estate. This would now be considered A) unissued stock. B) outstanding stock. C) treasury stock. D) retired stock.

C Treasury stock is issued and outstanding stock of the corporation that has been reacquired by the company. In some cases, such as this question, the stock is donated back to the company, but in most cases, it has been purchased by the company in the open market. Regardless of how stock is acquired, it is no longer outstanding, and it is held in the company's treasury. Once stock has been issued, it is always issued, even when it is no longer outstanding. How do we know the stock was donated back to the company and not to a charity? The key is the word now in the final sentence. Now means that something has changed. If the shares were donated to a charity, they would still be outstanding—nothing has changed. It is only because they were donated back to the issuer that there has be a status change. LO 3.a

The City of Podunk has an outstanding 25-year maturity issue that is callable in seven years. It has prerefunded the issue and established an escrow account containing the proper government securities with face amounts and maturities approximating the call provisions of the original issue. In quoting the original issue, which of the following must be used? A) Current yield B) The lower of the yield to call or the yield to maturity C) Yield to call D) Yield to maturity

C When a bond issue is prerefunded, the issuer is going to redeem the bond on the first call date. The yield must be quoted to call. LO 6.h

Customers will have a potentially unlimited loss if they are A) short 1 ABC Jan 50 put and long 100 shares of ABC stock. B) long 1 ABC Jan 50 put and long 100 shares of ABC stock. C) short 1 ABC Jan 50 call and short one ABC Jan 50 put. D) long 100 shares of ABC stock and short 1 ABC 50 call.

C When trading options, there is one way in which to have a potentially unlimited loss. That is the uncovered (naked) call. When a call option is written without a corresponding long position in the underlying, the writer loses when the price goes up. Because there is theoretically no limit as to how high a stock's price can go, the potential loss is unlimited. In this short straddle position, it is the short call that creates this possibility. With a short put, the lowest a stock's price can go is to zero. With a 50 put, that is a maximum loss of $50 less the premium received. The maximum loss on any long position, stock or option, is what the investor paid for it. LO 10.h

Several investors open an account in joint tenancy. Which of the following statements regarding the account is true? A) Only one designated account holder need sign a margin agreement or other forms pertinent to the account. B) Checks may be made payable to one tenant of the account. C) Mail need only be sent to one of the parties to the account. D) Checks need not be endorsed by all parties to the account in order to be deposited.

C While mail only needs to be sent to one of the parties to the account, checks for disbursements from the account must be made payable to all parties and endorsed by all parties in order to be deposited. Any required forms pertinent to the account, such as a margin agreement or options agreement, must be signed by all parties. LO 1.b

all of the following might be used service municipal revenue debt except A. excise tax B. business license taxes C. ad valorem taxes D. alcohol taxes

C property taxes are associated with GOs.

A corporation has $12 million net income after taxes, 5 million common shares outstanding, and $10 million of 6% preferred stock ($100 par). What is the corporation's earnings per-share (EPS)? A) $2.52 B) $2.40 C) $2.28 D) $1.20

C) $2.28 Explanation Begin by calculating how much of the net income is available for common stockholders (net income after taxes minus preferred dividends equals earnings available for common stockholders). The preferred stockholders received $600,000 in dividends (100,000 pfd shares × $6 per share dividends = $600,000). After subtracting $600,000 from the net income of $12 million, this leaves $11.4 million (earnings available for common stockholders). Compute EPS (earnings available for common stockholders / number of common shares outstanding = $11.4 million / 5 million shares = $2.28 per-share EPS). LO 13.d

A corporation has $20 million net income after taxes, 7 million common shares outstanding, and $15 million of 6% preferred stock ($25 par). What is the corporation's earnings per share (EPS)? A) $2.86 B) $2.54 C) $2.73 D) $2.83

C) $2.73 Begin by calculating how much of the net income is available for common stockholders (net income after taxes minus preferred dividends results in the earnings available for common stockholders). The preferred stockholders received $900,000 in dividends ($15 million par × 6% = $900,000), or 600,000 shares × $1.50 per share = $900,000). After subtracting $900,000 from the net income of $20 million, this leaves $19.1 million (earnings available for common stockholders). Compute EPS (earnings available for common ÷ number of common shares outstanding = $19.1 million ÷ 7 million shares = $2.73 per share EPS). LO 13.d

If a customer has a long margin account with a market value of $20,000, a debit balance of $12,000, and SMA of $5,000, how much cash can the customer withdraw from the account? A) $2,500 B) $5,000 C) $3,000 D) $1,500

C) $3,000 SMA is a line of credit with one restriction: it may not be used if account equity would fall below minimum maintenance. In this account, maintenance equity is $5,000 (25% of $20,000) and the current equity in the account is $8,000 ($20,000 LMV − $12,000 DB). Because the debit balance cannot exceed $15,000, only $3,000 may be withdrawn to keep the equity at the minimum of $5,000. LO 16.d

Which of the following statements regarding yield shown on a bond confirmation for a bond that has been called is true? A) A bond confirmation will show YTM if the bond has been called under an in-whole call provision. B) A bond confirmation will show the higher of YTC or YTM if a bond has been called under an in-part call provision. C) A bond confirmation will show YTC if the bond has been called under an in-whole call provision. D) A bond confirmation will show the lower of YTC or YTM if the bond has been called under an in-whole call provision.

C) A bond confirmation will show YTC if the bond has been called under an in-whole call provision. A bond confirmation for a bond called under an in-whole call provision will show yield to call (YTC), as the bond being called away is certain. However, in the event of an in-part call, there is uncertainty as to whether that particular bond will be called. Therefore, the lower of the YTC or yield to maturity would be shown on the confirmation. LO 15.a

A customer opens a margin account by purchasing 300 shares of XYZ at $60 and deposits the required margin. The stock rises to $70 on the following day. On the third day, after the release of a disappointing earnings report, the stock falls to $50. Which of the following describes the account after the changes in market value? A) Equity of $12,000; special memorandum account (SMA) of $1,500 B) Equity of $6,000; special memorandum account (SMA) of $0 C) Equity of $6,000; special memorandum account (SMA) of $1,500 D) Equity of $9,000; special memorandum account (SMA) of $0

C) Equity of $6,000; special memorandum account (SMA) of $1,500 Explanation The account starts out as follows: $18,000 − $9,000 = $9,000 (LMV − DB = EQ). After the stock rises to 70, the account looks like this: $21,000 − $9,000 = $12,000; SMA = $1,500. For every $1 increase in market value, 50 cents of SMA is created. After the stock falls to 50, the account looks like this: $15,000 − $9,000 = $6,000; SMA = $1,500. An increase in market value creates SMA but a subsequent decline has no effect. LO 16.d

Which of the following is not considered when trying to diversify a municipal bond portfolio? A) Maturity B) Geographical location C) Price D) Quality

C) Price One of the purposes of diversifying a municipal bond portfolio is to spread the risk among the portfolio's issues. This can be accomplished by buying bonds of differing maturities, geographical locations, and quality. LO 14.a

A customer has his broker enter an order to buy GHI stock at the opening. Though transmitted promptly, the order does not reach GHI's trading post in time to be filled at the opening. How is the order handled? A) The order is handled as a market order. B) The order automatically becomes an at-the-open order in the following trading session. C) The order is canceled. D) The order is executed in the day, at a price as close to the opening price as possible.

C) The order is canceled. An at-the-open order is to be filled at the opening price or not at all. An at-the-open order arriving later must be canceled. LO 16.a

SEC Rule 17a-4 describes the retention requirements for records made by registered broker-dealers. Which of the following records has a retention requirement of six years? A) Articles of incorporation if the member firm is a corporation B) Records of the disposition of written customer complaints C) The record of a transfer of a customer account to another member firm D) Trade confirmations for transactions with institutional customers

C) The record of a transfer of a customer account to another member firm When using the ACATS system to transfer a customer account from one member to another, the transferring member must keep records of the transfer for six years. The reason is that customer account records have a six-year retention requirement, and the transfer documentation contains the account information. Customer complaints are a four-year record—and corporate documents, such as charters or articles of incorporation, are for the lifetime of the firm. Trade confirmations are a three-year record, regardless of the nature of the customer. LO 15.e

Under Municipal Securities Rulemaking Board (MSRB) rules, which of the following yields for a callable bond would be shown on a confirmation? A) Yield based on catastrophe call B) Yield based on in-part call C) Yield based on nearest in-whole call D) Yield based on farthest in-whole call

C) Yield based on nearest in-whole call MSRB rules require that yield to call based on the nearest (near-term) entire issue (in-whole call) be disclosed on a customer's confirmation. With a partial call, the bond being purchased may or may not be included in the call, and with a catastrophe call provision, a call would only occur if an unpredictable event requires the issuer to call the bonds. LO 15.a

Under SEC Rule 10b-10, customer confirmations must be sent at or before A) the date of the transaction. B) the end of the month in which the transaction took place. C) completion of a transaction. D) the settlement date plus two business days.

C) completion of a transaction. Rule 10b-10 is the SEC's rule on delivery of customer confirmations, and it requires that they be sent no later than the completion of the trade (synonymous with the settlement date). The settlement date plus two business days refers to Regulation T payment. LO 15.a

A confirmation to a customer purchasing a new issue of bonds must disclose all of the following except A) customer's name. B) coupon rate and maturity date. C) current yield. D) settlement date.

C) current yield. To conform with industry rules, confirmations must include the customer's name, trade and settlement dates, coupon rate and maturity, and the yield to maturity or yield to call (whichever is lower). The current yield (annual interest / current market price) is not included on confirmations. LO 15.a

Each of the following is affected by the sale of securities in a restricted margin account except A) debit balance. B) market value. C) equity. D) SMA.

C) equity. When securities in a margin account are sold, the market value in the account will decline, the debit balance will be reduced by the cash proceeds, and SMA will increase by 50% of the sale. Equity in the account will not be affected unless the customer decides to withdraw some of the proceeds through the use of SMA. LO 16.d

When investing in securities, there are many potential risks. When recommending a specific security to clients, a member firm A) may disclose the existence of a control relationship between the firm and the subject company if it is material. B) must have a reasonable belief that the recommended security will outperform the overall market. C) must disclose the existence of a control relationship between the firm and the subject company. D) may follow the dictum of caveat emptor.

C) must disclose the existence of a control relationship between the firm and the subject company. While it is not often thought of as a risk, disclosing the existence of a control relationship between the member firm and the subject security is something that must be done. There is no question that this is material. Not all recommendations are for securities that will outperform the market. What about a stock recommendation to sell short? Buyer beware (caveat emptor) has no place in this industry. LO 14.b

An investor, age 57, wants to amend an existing portfolio to have a greater percentage be in fixed-income (debt) instruments. Current market sentiment is that interest rates are very high and likely to begin contracting soon. The investor agrees and asks for your thoughts regarding what those debt instruments might be. The most suitably aligned with the market sentiment would be A) variable-rate municipal bonds. B) money market fund. C) noncallable corporate bonds. D) callable corporate bonds.

C) noncallable corporate bonds. If one anticipates that interest rates will be falling, noncallable bonds would be better, as there is no risk of them being called and you can continue to earn the higher rate the bonds were issued with. Anything with a variable rate will have the interest payable adjusted to align with current rates, and therefore, would not desirable when rates are falling. Money market funds are not debt instruments, and again, the returns they pay reflect trending interest rates. LO 14.a

When customers receive their account statements, they will generally not include A) interest charged on debit balances in margin accounts during the statement period. B) total cost of purchases and net proceeds of sales made during the statement period. C) trade dates of all transactions during the statement period. D) security positions at the end of the statement period.

C) trade dates of all transactions during the statement period. Trade dates appear on the trade confirmations. LO 15.a

An open-ended investment company's net assets are 1B. it has elected to be treated as a diversified investment company. the company would be in noncompliance with the rules if it A. invested $50M in ABX stock B. invested $300M in DBY stock C. invested $350M in DNI stock D. purchased 100% of voting shares in ABC stock

C. 25% however it wants to invested=$250M 75%-5% rule=$50 million per stock $250+50M=$300M $350 not okay

the OTC market is a negotiated market. This is best described by saying that buyers and sellers A. bid & offer in a public place B. bid & offer continuously C. arrive at an agreeable price by bargaining D. must accept prices set by dealers

C. unlike the exchanges, which are auction markets, prices are determined by negotiation &bargaining between BDs

*hardest corporate characteristic to avoid for a partnership

Central management-no business can function without it

A customer sells short 100 shares of XYZ at 58 and buys 1 XYZ Jan 60 call for 3. If the stock price falls to $52, the customer buys back the stock and closes the option at 1 for A) a loss of $400. B) a gain of $300. C) a loss of $300. D) a gain of $400.

D The customer made $600 on the short stock position ($58 to $52) and lost $200 on the call (bought for 3, sold at 1). Overall, the gain is $400. LO 10.h

The client sells short 1,000 shares of ABC at $70,000 and meets Reg T requirement. The security falls to $60 Show margin chart

CR=$105,000. SMV=$70,000 EQ=$35,000 CR=$105,000. SMV=$60,000 EQ=$45,000

CR=$105,000. SMV=$60,000 EQ=$45,000 Security value rises to $80000 How much cash must the customer deposit?

CR=$105,000. SMV=$80000 EQ=$25,000 Reg t=80,000x0.5=40,000 Minimum maintenance requirement=0.3x80,000=24,000 No cash needed

all of the following risks are considered diversifiable except A. currency risk B. liquidity risk C. purchasing power risk D. sovereign risk

C. purchasing power risk

CEF risk-leverage risk

CEF can issue bond -borrowed money can help increase returns when the manager guesses correctly -if the market moves the wrong way, losses are magnified

close end fund benefits-leverage

CEFs can issue debt securities -borrowed money creates financial leverage which can multiply gains

Client sells short 1,000 shares of ABC at $70,000 and meets Reg T requirement. The security falls to $60 Show margin chart

CR=$105,000. SMV=$70,000 EQ=$35,000 CR=$105,000. SMV=$60,000 EQ=$45,000

Purchase of 1000 shares of THV for $20. Margin purchase

Customer borrows 50% from BD and deposits the remaining 10,000 equity's

A fund seeks maximum capital appreciation by investing in common stocks of companies located outside the United States. The management selects well-established companies that are listed on their national stock exchanges and that have demonstrated high earnings potential. This information describes which of the following mutual funds? A) ATF Biotechnology Fund B) ATF Capital Appreciation Fund C) ABC Stock Index Fund D) ATF Overseas Opportunities Fund

D

A broker's broker does all of the following except A) acts as agent for dealers. B) assists in placing securities. C) conceals the identity of the principals. D) makes a market in securities.

D A broker's broker acts as the agent in transactions by facilitating the movement of blocks of bonds. The broker's broker is allowed to conceal the identities of the contra-parties, thus protecting investment strategies. A broker's broker does not make a market in securities. LO 6.d

Which of the following investment companies is limited to offering investors a single class of common stock representing ownership in the company? A) A unit investment trust B) A face amount certificate company C) A closed-end management company D) A mutual fund

D A mutual fund is an open-end management company that raises capital solely through the issuance of a single class of common stock. A face amount certificate sells interests in a pool of bonds that all mature on the same date. A unit investment trust is a fixed portfolio of debt or equity securities that sells redeemable units, not shares, to investors. Those units represent the investor's share in the trust's assets. A closed-end management company does a one-time IPO of common stock, after which its shares trade in the secondary market just like any corporate stock. Closed-end companies can also have a preferred stock issue and a bond issue. Please note: The term "single class" of stock is not related to the different classes for sales charge and expense purposes. Those classes merely define the costs the investor will incur to acquire and own the shares. LO 8.a

An investor receiving a quote of 102 for a municipal security is probably interested in A) a general obligation bond. B) a bond anticipation note. C) a serial bond. D) a term bond.

D A quote of 102 is referred to as a dollar quote ($1,020) rather than a yield quote. The most common dollar bonds are those with a term maturity. The other choices are most often quoted on a yield basis rather than a price basis. LO 6.a

Regulation BI established a new standard of conduct under the Securities Exchange Act of 1934 for broker-dealers and associated persons of a broker-dealer when making a recommendation of any securities transaction or investment strategy involving securities (including account recommendations) to a retail customer. All of the following are examples of account recommendations except A) opening an UTMA account for a grandchild. B) taking a distribution from an employer-sponsored plan and executing a rollover into a self-directed IRA. C) opening a margin account to go along with an existing cash account. D) changing the asset allocation in an existing account.

D Account recommendations include recommendations of securities account types generally (e.g., to open an IRA or margin account), as well as recommendations to roll over or transfer assets from one type of account to another (e.g., a workplace retirement plan account to an IRA). It has nothing to do with changing the strategy in an existing account. Rather, the desired result of an account recommendation is a new account. **This question deals with material not covered in your LEM, but it relates to recent rule changes and/or student feedback. LO 2.f

A town's ad valorem tax rate is 20 mills on 60% of the assessed value. Your client owns a property with a market value of $500,000, and the town has assessed it at $400,000. The taxes due on this property are A) $800. B) $480. C) $8,000. D) $4,800.

D Ad valorem tax rates are based on mills with one mill being equal to $0.001 (1/10th of a cent).The amount of taxes to be paid on the property is determined by multiplying the millage rate—in this case 2 cents (20 mills at $0.001 = $0.02) times the taxable assessed property value. Note that this town is only taxing on 60% of the assessed value. That would be 60% of the $400,000 assessed value, or $240,000. Therefore, the tax is $0.02 times $240,000. The market value is irrelevant. For those who have difficulty determining where the decimal point goes, on any question like this, drop the last three 000s from the assessed value and multiply by the millage rate. In this question, that would be $240 times 20 and that equals the correct answer of $4,800. LO 6.c

In general, commercial paper, a popular money market instrument, has a maturity not exceeding A) 30 days. B) 365 days. C) 90 days. D) 270 days.

D Although there are rare cases where the maturity extends as long as 12 months (365 days), for exam purposes, think of CP with a maximum maturity of 270 days (9 months). LO 4.c

If an investor has a fixed-annuity contract with an insurance company, which of the following risks is assumed by the investor? A) Investment risk B) Mortality risk C) Value of each annuity unit each month D) Purchasing power risk

D An investor who purchases a fixed-annuity contract assumes purchasing power risk. Fixed annuities pay a fixed monthly benefit that loses purchasing power if there is inflation. LO 8.e

The unqualified legal opinion on a municipal bond states that A) the bond counsel needs more time to qualify the opinion. B) the bond has passed the additional bonds test (parity test). C) the bond is marketable. D) the issuer has the authority to issue bonds that are legal, valid, and enforceable obligations of the issuer.

D An unqualified opinion means the bond counsel attests that, to the best of its knowledge, the issuer has the legal right to issue the securities in question. In the case of tax-exempt bonds, the interest the issuer will pay on the bonds is exempt from federal taxation and the bonds are exempt from federal registration requirements. The legal opinion does not evaluate the issue's marketability, or safety, debt service requirements. LO 6.c

An investor is looking for a fixed-income investment that can provide a reasonable income while offering potential inflation protection. Which of the following would be the best recommendation to meet this investor's objective? A) Common stock B) Cumulative preferred stock C) High-yield bonds D) Convertible bonds

D Convertible bonds offer the best solution for this client. The bond carries a fixed interest rate, meeting the goal of reasonable income. The ability to convert the bond into common stock offers the potential to keep pace with inflation. Common stock historically has been the best inflation hedge. Why isn't it the correct answer here? Because common stock is not considered a fixed-income investment. Although the cumulative preferred stock is a fixed-income investment, there is no inflation protection (the cumulative feature only provides assurance that past dividends are likely to be paid). High-yield bonds will provide income but, once again, there is no inflation protection. What's more, these are known as junk bonds, and the steady income might be questionable due to the low quality of many of these bonds.

Among the differences between a real estate investment trust (REIT) and a real estate limited partnership investment (a DPP) is that A) REITs generally trade on the listed exchanges, while DPPs actively trade OTC. B) the DPP takes an ownership interest in the property, while the REIT only makes mortgage loans. C) the DPP generally has more investors than the REIT. D) only the DPP is a flow-through vehicle.

D DPPs are the investment vehicle providing flow-through of income and loss. REITs are required to return at least 90% of their net investment income to investors, but losses do not pass through. Both investments can take equity or debt positions. They can also be hybrid and take both. Because DPPs are almost always private placements, the number of investors is usually small. On the other hand, because most REITs are publicly traded, they can have a large number of investors. As private placements, there is no active secondary market for DPPs. LO 11.f

A quote of 2.20 bid 2.18 ask would most likely be a quote on A) a GO bond. B) a T-bond. C) a T-note. D) a T-bill.

D Discounted instruments (such as T-bills) are quoted on a discount-to-yield basis. Even though the number representing the bid is higher than the ask, it would be lower when converted into dollars. That is, the asking price is the face amount less a discount of 2.18%, while the bid price is the face amount less a discount of 2.20%. Just like anything we buy, the greater the discount, the lower the price. LO 7.b

An example of overlapping debt would be a school district and A) corporate debt of the county's largest employer. B) a water pollution control facility. C) a local utility power plant. D) county general debt.

D Do not combine revenue bonds with GOs to determine overlapping debt. Overlapping debt occurs in real estate taxing situations. Only GOs are backed by real estate taxes. LO 6.b

A bond with a 9% coupon, maturing in 18 years and 6 months, is selling at 120. The yield to maturity is closest to A) 9.00%. B) 11.66%. C) 7.50%. D) 7.05%.

D Don't waste time trying to do the yield to maturity computation. This bond is selling at a premium (120% of par). Therefore, all of the computed returns must be lower than the 9% nominal (coupon) yield. Only two of them are. The 7.50% represents the current yield ($90 ÷ $1,200). We know from our charts that, just like a seesaw, the farther from the center you go, the bigger the move at the end. That means the nominal yield is the highest, followed by the current yield (CY), the yield to maturity (YTM), and finally the yield to call (YTC) as the lowest. Because only one choice is lower than the CY, you get the correct answer with minimal effort. LO 4.e

One of your existing clients wishes to open a new account in the name of his spouse and enter orders on her behalf. Which of the following statements is true? A) This action is prohibited unless the customer signs a trading authorization on behalf of his spouse. B) This practice is ordinary and acceptable. C) The agent could be liable if the stock declines in value. D) This action is prohibited unless the spouse signs a trading authorization.

D Effecting transactions without specific written authority from the beneficial owner of the account is prohibited. This customer cannot sign a trading authorization on behalf of his spouse. The spouse must sign the authorization. LO 1.d

An investor purchases a municipal bond at par for $10,000 on February 15, 1997. On August 15, 1997, if the investor sells the bond for $10,500, for tax purposes, the $500 profit is recognized as A) a tax-free capital gain. B) interest income. C) a long-term capital gain. D) a short-term capital gain.

D Explanation When municipal bonds are purchased at par and subsequently sold at a higher price, the resulting profit is taxed as a capital gain. Only interest income from municipal bonds is exempt from taxation. This gain is not classified as long-term because the investor did not hold the bond for more than one year. LO 6.f

FINRA Rule 2330 states: no member or person associated with a member shall recommend to any customer the exchange of a deferred variable annuity unless such member or person associated with a member has a reasonable basis to believe the exchange suitable, taking into consideration whether A) the new annuity has a higher assumed interest rate. B) the customer's health has declined since the purchase of the initial annuity. C) the customer is at least 59½ and will not be subject to the 10% tax penalty. D) the customer has had another deferred variable annuity exchange within the preceding 36 months.

D FINRA Rule 2330 specifies the suitability conditions surrounding the recommended exchange of an existing deferred variable annuity contract for a new one. Included in the list of considerations is determining if the customer has made another variable annuity exchange within the previous 36 months. Unlike life insurance, where the insured's health is important, there are no health questions on an annuity application. It is safe to assume that any annuity exchange will be done under the provisions of Section 1035. This means there are no tax consequences. The assumed interest rate is for internal purposes; it is not a differentiator when deciding which variable contract to purchase. LO 9.e

A sophisticated client has expressed an interest in becoming more aggressive with their investment strategy. Her current portfolio consists of the following: $50,000 cash $200,000 in retirement accounts $100,000 in various individual stocks in different industries $100,000 in a balance fund She is willing to invest $25,000 for a minimum of 7 to 10 years and accepts that the investment can and will fluctuate in value over time. Which of the following investments would be the most appropriate? A) XYZ Value Equity Fund B) MNO High-Yield Bond Fund C) DEF Asset Allocation Fund D) ABC Capital Appreciation Small-Cap Fund

D For someone who is willing to take the risk and invest for the long haul, a small- or mid-cap growth fund would be appropriate. LO 8.g

Government agency bonds issued by which of the following carry a minimum denomination of $1,000 with $1 increments? A) Freddie Mac B) Federal Home Loan Bank C) Sallie Mae D) Ginnie Mae

D GNMA securities are available with a minimum denomination of $1,000 and in increments of $1.00. That means, for example, that a client can purchase one for $1,003.00 or $1,337.00 if desired. The only other agency with that type of pricing is the FNMA. LO 7.d

An investor establishes the following positions: Long 1 XYZ Apr 40 call for 6 Long 1 XYZ Apr 50 put for 8 If both options are sold for intrinsic value when XYZ trades at 44, the investor realizes a loss of A) $200. B) $1,000. C) $100. D) $400.

D If the opening purchase of the XYZ Apr 40 call was made at 6, and the closing sale of that call was made at 4, the difference of 2 represents a $200 loss. If the opening purchase of the XYZ Apr 50 put was made at 8, and the closing sale of that put was made at 6, the difference of 2 represents a $200 loss. The total loss for the account is $400. LO 10.f

If a customer sold 1,000 shares of XYZ at a loss, a wash sale will result within 30 days of the date of sale if your customer A) buys 10 XYZ at-the-money puts. B) writes 10 XYZ at-the-money puts. C) writes 10 XYZ at-the-money calls. D) buys 10 XYZ at-the-money calls.

D If, within 30 days of the date of sale, the customer buys back the security or the right to buy it back (a call option), the loss is disallowed. It will also be disallowed if, within 30 days, the customer writes deep in-the-money puts on the security sold. A deep in-the-money put will likely be exercised, forcing the customer to buy stock. LO 13.h

A municipal A & O bond is issued on October 1, 2010, with a 10-year stated maturity. If a trade in this bond settles on April 1, 2020, how many days' worth of accrued interest will be added to the price of the bond? A) 180 B) 1 C) 90 D) 0

D Interest on a municipal bond begins to accrue on the previous payment date and ends the day before settlement date. Always assume a bond pays interest on the first of the month unless told differently. In this case, interest is payable on April 1 and October 1 each year. Whenever a bond trade settles on a payment date, it trades flat (without accrued interest). LO 6.e

Each of the following is a defined contribution plan except A) a profit-sharing plan (qualified). B) a 401(k) plan. C) a money-purchase pension plan. D) a stock option plan.

D Money-purchase pension plans, 401(k) plans, and qualified profit-sharing plans are all examples of defined contribution plans. An employer may offer stock options that give an employee the right to purchase a specified number of shares of the employer's common stock at a stated exercise price over a stated time period. No actual contribution is made, just payment when the employee decides to exercise the option. Unlike the other choices, this is not a qualified plan. LO 1.h

If a customer buys a municipal bond at 110, maturing in eight years, but sells the bond six years later at 103½, the customer will have A) a $65 per bond loss. B) a $35 per bond gain. C) a $10 per bond loss. D) a $10 per bond gain.

D Municipal bonds that are purchased at a premium must be amortized. This bond has a premium of $100, which over eight years, amounts to $12.50 per year. The cost basis of the bond at the time of the sale is $1,100 − (6 × $12.50), or $1,025. If the bond is sold for $1,035, the customer has a gain of $10 per bond. LO 6.e

All of the following would be reasons for an employer to choose a nonqualified plan over a qualified plan except? A) the nonqualified plan can discriminate in favor of highly compensated employees. B) the nonqualified plan is not subject to ERISA reporting and disclosure requirements. C) the nonqualified plan provides greater flexibility. D) the nonqualified plan provides an immediate income tax deduction for the employer.

D Nonqualified plans do not provide a tax deduction to the employer until the employee receives the economic benefit as income at some point in the future. They are, however, more flexible because they do not have to comply with ERISA reporting and nondiscrimination requirements. LO 1.f employer can deduct contributions made to qualified plans

A new customer has been approved for all levels of options trading and has signed the options disclosure document. Even though approved for all levels of options trades, she notes that she will not be employing, and the registered representative (RR) should not recommend, any strategies with unlimited maximum loss potential. Given this criteria, an RR could suitably recommend A) short or long straddles. B) short or long uncovered calls. C) short stock or short stock/short put hedge. D) short or long spreads.

D Of the pairings offered, only short and long spreads both have defined loss potentials. Short stock, short calls, short straddles, and a short stock/short put hedge positions all have unlimited loss potentials. LO 10.e

A unit investment trust has 90% of its portfolio invested in high-grade bonds with an average maturity of almost 25 years. If the industry consensus was that long-term interest rates were about to increase sharply, which of the following actions would most likely be taken? A) Switch to short-term bonds B) Liquidate and begin to move into cash or cash equivalents C) Ladder the maturities D) No action would be taken

D One of the key distinctions of a UIT is its lack of management. Once the portfolio has been created, it is fixed until maturity, in the case of debt securities, or until some predetermined liquidation point, in the case of an equity trust. LO 8.a

A 38-year-old investor places $25,000 into a single premium deferred variable annuity. Twenty-two years later, with the account valued at $72,000, the investor surrenders the policy. If the investor is in the 25% marginal income tax bracket, the total tax liability is A) $18,000. B) $25,200. C) $16,450. D) $11,750.

D Only the deferred growth is taxable. In this case, it is the difference between the surrender value of $72,000 and the cost basis of $25,000. That $47,000 is taxed at the marginal rate of 25%. Because the investor is older than 59½ (38 + 22 = 60), there is no additional 10% penalty tax. Effectively, this is a 25% tax on $47,000. LO 9.d

A new account is opened at your firm and you notice that there is a trusted contact person form attached to the documentation. You could safely surmise that this is an account for A) a minor. B) a trust. C) a corporation. D) a specified adult.

D Per FINRA Rule 2165, a specified adult "is a natural person age 65 or older or a natural person age 18 or older who the member reasonably believes has a mental or physical impairment that makes the individual unable to protect his own interests." FINRA requires members to make reasonable efforts to obtain the name of and contact information for a trusted contact person. This person must be someone age 18 or older who may be contacted about the customer's account. The rules do not require a customer to provide trusted contact information, only that the firm make the effort. LO 1.d

In a rising interest rate environment, which of the following risks associated with mortgage-backed securities such as a collateralized mortgage obligation (CMO) is of least consequence to a potential investor? A) Interest rate risk B) Credit risk C) Extension risk D) Prepayment risk

D Prepayment risk is the risk that mortgage holders will refinance or repay their mortgages early as a result of falling interest rates. Therefore, in a rising interest rate environment, it would be less of a concern for a CMO investor. Extension risk is the risk that mortgage payments will be missed or slower than anticipated in a faltering economic environment. Credit and interest rate risks are always of concern with CMOs. LO 12.d

If a 40-year-old customer earns $65,000 a year, and his 38-year-old spouse earns $40,000 a year, how much may they contribute to IRAs? A) They may not contribute because their combined income is too high. B) Only the higher wage earner may contribute to an IRA. C) They may contribute up to the maximum annual allowable dollar limit split evenly between both accounts. D) They may each contribute 100% of earned income or the maximum annual allowable dollar limit, whichever is less, to an IRA.

D Regardless of the amount, individuals or couples may contribute to their IRAs if they have earned income. Each is entitled to contribute 100% of earned income up to the maximum allowed. However, if either or both of them are covered under a qualified plan, limits may exist on the deductibility of the contributions. LO 1.g

SEC Regulation Best Interest (BI) focuses on A) customer transaction costs. B) ensuring that customers receive the highest possible interest on their cash balances. C) ensuring broker-dealer profitability. D) recommendations to customers.

D Regulation BI became effective on June 30, 2020, and states, "When making such a recommendation to a retail customer, you must act in the best interest of the retail customer at the time the recommendation is made, without placing your financial or other interest ahead of the retail customer's interests." **This question deals with material not covered in your LEM, but it relates to recent rule changes and/or student feedback. LO 2.f

Regulation T controls the extension of credit from A) broker-dealer to broker-dealer. B) bank to customer. C) bank to broker-dealer. D) broker-dealer to customer.

D Regulation T controls the extension of credit from broker-dealer to customer, with customer securities providing the collateral for such loans. LO 2.g

Which of the following would be considered an equity security? A) Negotiable CDs B) Exchange-traded notes C) Equity-linked notes D) Preemptive rights

D Rights (and warrants) are included in the term equity security. Confusingly, equity-linked notes are debt securities, even though the term equity is in the name. On this exam, notes always represent a form of debt security. LO 4.g

An important feature of scheduled premium variable life insurance policies is that A) the cash value can never fall below the guaranteed minimum amount. B) better than expected performance of the separate account can lead to reduced premiums. C) purchasers must understand that there are no guarantees with these policies. D) the death benefit can never fall below the guaranteed minimum amount.

D Scheduled (fixed) premium variable life always has a guaranteed death benefit. Cash values cannot be guaranteed, only the death benefit. Better than expected performance of the separate account will lead to increased cash values, but it will not affect the premiums. LO 9.a

All of the following option strategies could be effectively used in a bear market except A) a credit call spread. B) a debit put spread. C) a short call. D) a short straddle.

D Short straddles are appropriate only in flat or neutral markets. The writer will lose in a rising market (the call will be exercised) or a falling market (the put will be exercised). Short calls and short call spreads are bearish, as are debit (long) put spreads. LO 10.f

The 5% markup policy applies to A) mutual funds. B) new issues. C) all of these. D) principal over-the-counter (OTC) trades.

D The 5% markup policy applies to agency and principal nonexempt securities and transactions, both exchange and OTC traded. It does not apply to prospectus offerings (mutual funds and new issues). LO 3.j

All of the following considerations apply to the 5% markup policy except A) the availability of the security. B) the cost of executing the transaction. C) the services rendered by the broker-dealer. D) the customer's ability to pay.

D The 5% policy is a guideline for markups and commissions for exchange and over-the-counter trades. A firm cannot charge a higher commission or markup to a customer on the basis of the customer's ability to pay. Factors relevant in determining the fairness of a commission or markup include the type of security, the services rendered by the firm, and the expenses and difficulty of executing a particular trade. LO 3.j

An individual purchases a variable life insurance policy. Under federal law, the individual is entitled to a complete refund of all premiums paid if the request is made within A) 10 days from the execution of the application, or for 45 days from the time the owner receives the policy, whichever is longer. B) the first 24 months after the policy was delivered to the owner. C) the first 30 days after the policy was delivered to the owner. D) 45 days from the execution of the application, or for 10 days from the time the owner receives the policy, whichever is longer.

D The Investment Company Act of 1940 specifies a free-look period for the purchaser of a variable life insurance policy. That period is the longer of 45 days after the execution of the application or 10 days after the actual policy is delivered to the owner. The 24 months is the minimum time limit for the exchange of the variable policy into another form of permanent insurance. LO 9.e

If interest rates increase, the interest payable on outstanding corporate bonds will A) change according to the inverse payout theory. B) decrease. C) increase. D) remain unchanged.

D The interest payable is the nominal yield, which is stated on the face of the bond. It is the percentage of face value the bond will pay each year regardless of the prevailing interest rates in the market. It is the market price of bonds, not the interest payable, that responds inversely to changes in interest rates. LO 4.b

Acme Pharmaceuticals previously had issued $200 million of common stock in an IPO. A year later, it issued $50 million of debentures at par value. Acme's leverage is what percentage of its total capital? A) 25% B) 400% C) 50% D) 20%

D The leverage is the extent to which borrowed funds make up the company's total capital. Total capital is the value of the equity and debt financing combined. Acme has issued $50 million of debentures (debt capital) and $200 million of equity capital (the common stock). That makes the total capitalization of Acme equal to $250 million. The leverage is $50 million divided by $250 million, or 20%. An analyst would consider this conservative leverage. LO 13.d

If a customer believes interest rates have peaked, and therefore, wants to buy long-term, fixed-income securities providing semiannual interest payments, you would recommend A) Treasury STRIPS. B) premium bonds with low call premiums. C) puttable bonds. D) bonds that do not have a call feature.

D The purchase of noncallable bonds provides the investor with a constant flow of semiannual interest income until maturity. Treasury STRIPS do not make regular interest payments. LO 7.a

An investor in which of the following products may not receive dividends? A) Units in a UIT B) Shares of common stock C) Shares of preferred stock D) Oil and gas limited partnership interests

D The structure of a limited partnership does not allow for the payment of dividends. If there is income, it flows through to the investor, but it is not considered a dividend. Common stock can pay dividends and preferred stock is purchased for its dividend payout. UITs pay dividends in a manner similar to mutual funds. LO 11.f

Libby sees a tombstone advertisement for a new issue of Southwest Barge subordinated convertible debentures. The bonds will carry an 11¼% coupon, are convertible into common stock at $10.50, and are being issued to the public at 100. The proceeds of the issue will be used specifically for purchasing new Southwest barges. Libby's concerns about the issue could include A) the conversion price might change, causing the parity price to rise. B) the new barges might sink and the collateral would be gone. C) the company might demand that she accept common stock for her bond. D) the issue may be junior-in-lien to another security issue.

D The word subordinated is the key to the question. A subordinated bond has other debt holders ahead of it in the event of liquidation. The barges do not serve as collateral because the bonds are identified as debentures. Having to convert to common stock is not a threat since she is the one that will, if she desires, exercise the conversion privilege. At least for exam purposes, the conversion ratio is always fixed and does not change. LO 5.b

A 45-year-old client of yours receives an inheritance of $100,000 and wishes to invest it without having to worry about any taxes being due until she reaches age 68. In addition, the client would like to have some protection against inflation. Which of the following would be the most appropriate recommendation? A) An immediate variable annuity B) An S&P 500 index fund C) A single premium variable life insurance policy D) A single premium deferred variable annuity

D There are two benefits to the deferred variable annuity. The first is that taxes on all earnings are deferred until withdrawal. The second is that, if the proper separate account subaccounts are selected, there is potential inflation protection. An index fund will meet the second objective, but, even though index funds tend to be tax efficient, there will be certainly be dividend distributions from an S&P 500 index fund and possibly some capital gains as well. An immediate variable annuity begins payout immediately, so taxes start immediately as well. Variable life will never be an answer to a question unless the question describes a need for life insurance coverage—it cannot be sold strictly as an investment. LO 9.b

Which of the following transactions would not be subject to the 5% markup policy? A) Your firm agrees to do an agency cross-transaction between two of your clients. Each client has been charged a commission. B) A client sells shares of an over-the-counter stock and uses the proceeds to purchase shares from your firm's inventory account. C) A client enters an order to purchase one share of a stock to be put in the name of her grandchild. You charge the client the minimum commission for your firm ($45) even though the stock is currently trading at $26 per share. D) Your client enters trades to purchase two different mutual funds in the same fund family. The combined purchases do not qualify for any breakpoints. The client is charged a sales charge of 6.5%.

D Transactions in securities that are sold by a prospectus are not subject to the 5% markup policy. A mutual fund will disclose its cost to the client in the prospectus and is therefore not subject to the rule. Five percent is a guideline, and markups can exceed it. LO 3.j

An investor owns 500 shares of JKL common stock. The investor's cost is $50 per share and the last quote on JKL was $60. One method the investor could use that would increase current income and offer some downside protection would be to A) buy a JKL $60 put @4. B) buy five JKL $60 puts @4. C) sell a JKL $60 call @4. D) sell five JKL $60 calls @4.

D Understand that with 500 shares, the protection needs five options. Selling five calls @4 generates $2,000 of premium income credited to the account one day after the sale. In addition, the sale has effectively lowered the cost from $60 to $56 per share. That is the partial protection offered when writing a covered call. Full protection would come from buying five JKL 60 puts, but the question specifically mentions receiving income and buying an option costs money rather than receiving money. Please note that some questions, like this one, do require you to consider the number of shares or options. Others, such as breakeven points, ignore the number of contracts. LO 10.h

Which of the following assets would be least likely used to back a collateralized debt obligation (CDO)? A) Auto loans B) Corporate receivables C) Credit card debt D) Mortgages

D Unlike CMOs, which are backed by mortgages (as the M indicates), CDOs are invariably backed by some other form of asset. Remember that what someone owes is their debt, while it is an asset to the creditor. LO 12.c

A respected analyst reports that last week's T-bill rate at 1% is lower than the rate for the preceding week and lower than the average for the past month. Which of the following is true? A) The general level of interest rates is increasing. B) Prices are descending. C) Investors are paying less for T-bills. D) Investors are paying more for T-bills.

D When the rate is lower, the price has gone up. This means investors are paying more as interest rates are going down. LO 4.b

KAPCO Manufacturing Corporation declares a 5-for-1 stock split on its outstanding shares of $20 par value common stock. This split will cause A) the price of the shares to change to $4 per share. B) a change to KAPCO's net worth. C) the dividend per share on KAPCO's preferred stock to be reduced. D) the par value of the shares to change to $4 per share.

D Whenever there is a stock split (forward, such as this, or reverse), the par value is adjusted for the split. With a $20 par and a 5-for-1 split, the par value now becomes $4 per share. The stock's market value will drop by approximately 20% (⅕) of the pre-split price. The question only tells us the par value, not the market value (and those two values are unrelated). A split to the common stock has no effect on the preferred stock. The net worth remains the same because no money is involved. LO 3.b

The writer of an IRX yield-based option, if exercised, must A) deliver T-bills. B) receive cash. C) receive T-notes. D) deliver cash.

D Yield-based options settle in cash if the option is exercised. The writer must deliver the in-the-money amount in cash. LO 10.g

If a customer sells a zero coupon bond before maturity, gain or loss will be the difference between sales proceeds and A) par value. B) discounted value. C) original cost. D) accreted value.

D Zero coupon bonds must be accreted for tax purposes. Each year, the annual accretion is taxable to the holder. In addition, the customer may adjust the cost basis of the zero upward by the amount of the annual accretion. LO 5.e

If a corporation, whose stock is publicly traded declares bankruptcy, holders of which of the following would most likely to recover the greatest amount of their investment? A. control stock B. senior preferred stock C. debentures D. collateral trust certificates

D collateral trust certificates is secured

Dollar cost averaging (DCA) will always result in a lower cost per share than the price paid per share except A) when the price for each purchase is increasing. B) when the price for each purchase is fluctuating.. C) when the price for each purchase is decreasing. D) when the price for each purchase is the same.

D there are two requirements for a dollar cost averaging program to work. The first is that the same amount must be invested at each specified interval. The second is that the price per transaction does not remain the same. If that is the case, then the average cost per share and average price paid per transaction are the same. The price needs to move for DCA to show a benefit. LO 8.e

Fundamental analysts

investors who attempt to find mispriced securities by analyzing fundamental information, such as accounting performance and earnings prospects

A customer receives a cash dividend of $1,000 in his margin account. How much of the dividend will be credited to the special memorandum account (SMA)? A) $0 B) $250 C) $500 D) $1,000

D) $1,000 Cash dividends and interest are considered nonrequired deposits to a margin account and are credited in full to SMA. Once credited to SMA, 100% of the deposit could be withdrawn unless doing so would create a margin call. LO 16.d

The ABC Corporation has a long-standing policy of maintaining a dividend payout ratio of 45%. ABC's net income for the year is $12 million, and there are 8 million shares of common stock outstanding. After a 3:2 stock split, the annual dividend per share is A) $1.0125. B) $1.50. C) $0.675. D) $0.45.

D) $0.45. Take this systematically. A dividend payout ratio of 45% means ABC will distribute 45% of its net income to its common shareholders. Forty-five percent of $12 million is $5,400,000 in dividends. If there were 8 million shares before the split, there are now 12 million (8 times 3/2 = 24 divided by 2 = 12). Divide the amount available for common ($5.4 million) by the number of shares (12 million) to arrive at a dividend per share of $0.45. There is another way to compute this. With net income of $12 million and 8 million shares, the pre-split earnings per share is $1.50 ($12 million divided by 8 million = $1.50). The company pays out 45% of its net income as a dividend. That would be $0.675 ($1.50 times 45%). With the 3/2 split, the number of shares has increased by 150%, meaning that the new dividend will be 2/3 of the previous amount. That computes to the same $0.45 per share. LO 13.d

A customer's long margin account contains the following securities: 100 shares of DEF, CMV $40 per share 200 shares of AMF, CMV $50 per share 100 shares of KLP, CMV $80 per share The current debit balance in the account is $10,800. If each of the securities held in the account were to appreciate by $5 per share, the equity in the account would be A) $11,200. B) $12,700. C) $8,200. D) $13,200.

D) $13,200. Before the $5 increase, the total long market value in the account is $22,000 and, therefore, the equity is $11,200 ($22,000 - $10,800 = $11,200). If each of the 400 shares in the account increases by $5, which represents an increase of $2,000 in long market value, the equity will increase by the same amount. After the increase, the long market value is $24,000 and the equity is $13,200 ($24,000 - $10,800 = $13,200). LO 16.d

A customer's margin account shows a debit balance of $10,000. Federal law permits the broker-dealer to rehypothecate a maximum of A) $20,000 of the customer's margin securities. B) $10,000 of the customer's margin securities. C) $5,000 of the customer's margin securities. D) $14,000 of the customer's margin securities.

D) $14,000 of the customer's margin securities. When a customer buys securities on margin, the broker-dealer holds the purchased securities as collateral for the margin loan. That is known as hypothecation. In most cases, broker-dealers rehypothecate the securities to a lending institution to recover the funds they loaned to the margin client. SEC rules limit the rehypothecation to 140% of the customer's debit balance (140% × $10,000 = $14,000). LO 16.d

In a margin account, your customer's long market value is $22,000. The debit balance is $8,000. If the customer enters an order to purchase $12,000 of stock, the margin call will be A) $0.00. B) $6,000. C) $4,000. D) $3,000.

D) $3,000. We determine the excess equity in this account by comparing the equity in the account with the Reg. T requirement. With a LMV of $22,000, Reg. T requires 50% equity, or $11,000. This account has equity of $14,000 ($22,000 minus the $8,000 debit balance). That is how we note the excess equity is $3,000 ($14,000 minus $11,000). Unless stated otherwise, that $3,000 will be journaled to SMA. The margin required on a new $12,000 purchase is $6,000, so this customer would need to deposit an additional $3,000 on top of the SMA. Alternatively, after this purchase, the LMV will be $34,000. Regulation T would require 50% of that, or $17,000. The account already has $14,000 of equity, so only $3,000 additional would be required. Yet another way is that $3,000 of SMA has a buying power of $6,000 of stock. The remaining $6,000 from the $12,000 purchase would require a cash deposit of $3,000. LO 16.d Previous Next

At the birth of a grandchild, your customers, the child's grandparents, purchased 1,000 shares of XYZ stock at $10 per share in their JTWROS account. The child is now an adult and the grandparents gift all the shares to their grandchild when the stock price is $50 per share. If the donee sold all the shares at $55 per share, the tax consequences would be a capital gain of A) $55,000. B) $5,000. C) $50,000. D) $45,000.

D) $45,000.00 The capital gain would be $45,000 ($55,000 - $10,000). If a gift is made of securities, the donee must use the original cost basis of the donor to calculate the gain or loss on a sale. In this case, the original cost basis for the grandparents was $10,000. The difference between that and the proceeds of $55,000 is a capital gain of $45,000. If the shares were inherited from the grandparents, the grandchild would have received the stock at a stepped-up cost basis, the price of the stock on the decedent's death. Using our numbers, that would be a capital gain of $5,000, the difference between the stepped-up basis of $50,000 and the proceeds of $55,000. LO 13.h

It is not uncommon for one company to attempt to take over another by acquiring a significant percentage of its voting shares. Under SEC rules, if the terms of the offer are changed, the revised offer must remain open for at least A) 10 business days from the commencement and 10 business days from the date the terms are changed. B) 10 business days from the commencement and 20 business days from the date the terms are changed. C) 20 business days from the commencement and 20 business days from the date the terms are changed. D) 20 business days from the commencement and 10 business days from the date the terms are changed.

D) 20 business days from the commencement and 10 business days from the date the terms are changed. The rule is that the offer must remain open for at least 20 business days from the time the tender offer begins, and if there should be a change to the terms of the offer, if must be held open for 10 business days from the change. This is more a principal level question and unlikely to be on your exam, but we want to be sure you are exposed to the information. LO 14.b

An investor is long 500 shares of DEFG common stock, short 200 shares of DEFG common stock, and short 300 shares of DEFG 5% preferred stock. A tender offer for DEFG common shares is announced. Under SEC rules, this investor is permitted to tender A) 800 shares. B) 500 shares. C) 0 shares. D) 300 shares.

D) 300 shares. The rules permit tendering of net long shares. That means the difference between the long and short positions. In this question, the investor is net long 300 shares: the difference between the 500 long and the 200 short. The preferred shares are not relevant because the tender is only for the common stock. LO 14.b

Which of the following statements best describes a hedge fund? A) A closed-end investment company employing leverage through the use of debt and preferred stock financing B) An investment company, registered under the Investment Company Act of 1940, that charges higher than usual management fees and employs sophisticated investment techniques in an attempt to provide level returns during periods of market uncertainty C) An investment pool that is generally unregistered and that, through the use of sophisticated market tools, offers investors returns that generally exceed those available elsewhere D) A private and unregistered investment pool that accepts the investor's money and employs sophisticated hedging and arbitrage techniques using long and short positions, leverage and derivatives, and investments in many markets

D) A private and unregistered investment pool that accepts the investor's money and employs sophisticated hedging and arbitrage techniques using long and short positions, leverage and derivatives, and investments in many markets As of the date of this course, hedge funds are not registered with the SEC (their managers generally are) and they are invariably sold in private offerings, usually under Regulation D of the Securities Act of 1933. Hedging and arbitrage techniques using long and short positions, leverage and derivatives, and investments in many markets are some of the primary techniques used by these funds. Although some hedge funds do outperform the market, a blanket statement cannot be made. LO 12.b

Term Life Insurance

Insurance that provides financial protection from losses resulting from a death during a definite period, or term.

There are a number of different types of orders that a registered representative can enter for a client. Of the following, which one would be most appropriate for a client wishing to protect a profit on a long stock position? A) A market order B) A sell limit order C) A buy stop order D) A sell stop order

D) A sell stop order Protecting a profit on a long position means getting out of the stock before its price declines below the original purchase price. That means selling the stock. The investor would enter the sell stop order with a stop price below the current market, but above the original cost. A sell limit order is placed above the market and that is of no help if the price drops. A market order is executed at once and buying the stock is not appropriate when the investor already owns it. LO 16.a

Which of the following would give a bearish sign to a technical analyst? A) A stock dropping below its resistance level B) An increase in the short interest C) A head and shoulders bottom pattern D) An increase in odd-lot purchases

D) An increase in odd-lot purchases Odd lots (less than 100 shares) are bought and sold almost exclusively by unsophisticated investors. Technicians believe them to always be on the wrong side of the market. When the odd lotters are buying, it is time to sell (bearish). High short interest is bullish. A head and shoulders bottom indicates that the stock has bottomed and is on its way back up (bullish). It would be bearish if a stock's price fell below the support level. LO 13.e

Which of the following statements regarding collateralized mortgage obligations (CMOs) is true? A) CMOs may not trade at a premium. B) CMO earnings are tax exempt. C) CMOs are considered high-yield bonds. D) CMO returns are affected by interest rate changes.

D) CMO returns are affected by interest rate changes. CMOs, like other mortgage-backed securities, respond to changes in interest rates. When interest rates decline, certain CMO tranches are subject to prepayment risk. CMOs are corporate instruments, and their interest is taxable at all levels. LO 12.d

ABC Corporation's earnings have increased by 10%, and its shares outstanding have increased 5%. This has what impact on ABC's EPS? A) EPS has decreased. B) EPS has stayed the same. C) There is not enough information to answer the question. D) EPS has increased.

D) EPS has increased. If earnings (net income) have increased faster than the shares outstanding, EPS will increase. For example, if ABC's earnings were $10 million and it had 1 million shares, the earnings per share were $10. Now, with a 10% increase to earnings, those earnings are $1.1 million. If the shares have increased by 5%, there are now 1,050,000 shares. That makes the new EPS $11,000,000 ÷ 1,050,000 shares which equals approximately $10.48 per share. This proves the correct choice that EPS has increased. LO 15.d

A client, age 27, is new to investing. With $20,000 saved thus far and $400 to allocate toward investing monthly, his goal is to purchase a home in three to five years. Which is the most suitable recommendation? A) Use leveraged index funds (2 or 3×) to maximize gains potential for the small investment amounts. B) Open a long margin account to take advantage of the leverage that margin purchases can create using small amounts of money. C) Invest in both equity and corporate debt mutual funds so that a portfolio of stocks and bonds is established. D) Invest in a money market mutual fund to build up more cash reserves.

D) Invest in a money market mutual fund to build up more cash reserves. Securities or strategies with longer time horizons than that of the goal to purchase a home should not be used until the client has established more liquid cash reserves. The money market mutual fund would be most appropriate because it is liquid and conservative. The balanced fund is not appropriate for someone with a two- to five-year goal, and margin accounts or leveraged funds entail risk unsuitable for a conservative investor with a short investment time horizon. LO 14.a

Last week one of your customers placed a good-til-canceled order to sell 200 shares of ABC with an 18 stop when the stock was trading at $18.85. It is now the ex-date for a $0.55 dividend and the order has not yet been executed. What has happened to your customer's stop order? A) It is canceled. B) It remains at $18. C) It is increased to $18.55. D) It is reduced to $17.45.

D) It is reduced to $17.45. Unless the customer has given DNR (do not reduce) instructions, open buy limit orders and open sell stop orders are reduced on the ex-dividend date by the amount of the dividend. LO 16.a

Which of the following is a risk most often encountered when investing in common stock? A) Interest rate risk B) Inflation risk C) Default risk D) Market risk

D) Market risk Common stocks have market risk because they move with the overall stock market. Market risk is a systematic risk, probably the one most investors think of when buying stock. Interest rate risk is associated with fixed-income securities. Their prices move inversely with interest rates. Debt securities also have default or credit risk. Investors use the ratings services to fit that risk with their tolerance. Historically, stocks have been a good hedge against inflation while debt instruments have not. LO 14.a

Peyton has been a client of Turing Technical Analytics (TTA), a FINRA member broker-dealer, for 10 years. Peyton has decided that it is time to move the account to a new firm, Enigma Mathematical Portfolio Modeling, (EMPM). Which of the following statements accurately reflects the requirements when using the ACATS system? A) EMPM has one business day to validate or take exception to the positions listed on the Transfer Initiation Form (TIF). B) EMPM has three business days to validate or take exception to the positions listed on the Transfer Initiation Form (TIF). C) TTA has three business days to validate or take exception to the positions listed on the Transfer Initiation Form (TIF). D) TTA has one business day to validate or take exception to the positions listed on the Transfer Initiation Form (TIF).

D) TTA has one business day to validate or take exception to the positions listed on the Transfer Initiation Form (TIF). ACATS requires the carrying firm (TTA) to validate or take exception to the securities listed on the TIF within one business day. If all is in order, the transfer must be completed within three business days. The receiving firm, EMPM, is responsible for sending the signed TIF to ACATS. LO 15.d

customer who has, as part of her account holdings, unlisted REITS, as well as a limited partnership interest in an oil and gas program, may expect her servicing member firm to show A) the amount shown on Tape B the business day before the account statement closing date. B) no valuation for the unlisted REITS and the original investment made in the DPP interest. C) an exact per share value as calculated on the last business day of the month. D) a per share estimated value of the securities.

D) a per share estimated value of the securities. A general securities member must include in a customer account statement a per share estimated value of a DPP or unlisted REIT security, in a manner reasonably designed to ensure that the per share estimated value is reliable. LO 15.b

All of the following statements about trading index options on the Chicago Board Options Exchange are true except A) floor brokers may execute orders for others on a commission basis. B) limit orders are maintained in an order official's book. C) market makers may trade for their own accounts. D) market orders entered by a market maker have priority over public orders.

D) market orders entered by a market maker have priority over public orders. Public orders must be filled before member orders. This is true if it is stock, bonds, and options of any kind. LO 16.a

If stock market indexes, such as the S&P 500 and the Dow Jones Industrial Average, are declining daily, and the number of declining stocks relative to advancing stocks is falling, a technical analyst will conclude that the market is A) unstable. B) overbought. C) becoming volatile. D) oversold.

D) oversold The momentum of the market decline seems to be easing as the number of decliners to advancers is leveling out. It looks like the advance/decline line is moving in a direction away from decliners. A technical analyst would conclude that the market is oversold and approaching a bottom. LO 13.e

The determination as to whether an over-the-counter stock is eligible for purchase on margin is made by A) the Federal Deposit Insurance Corporation. B) the Securities and Exchange Commission. C) the Financial Industry Regulatory Authority. D) the Federal Reserve Board.

D) the Federal Reserve Board. All decisions regarding initial margin eligibility are the role of the Federal Reserve Board. LO 16.d

Under Regulation T, action by the broker-dealer is not required when A) the total amount of the transaction does not exceed $1,000. B) the amount due does not exceed $100. C) the amount due does not exceed $200. D) the amount due does not exceed $1,000.

D) the amount due does not exceed $1,000. Regulation T permits a broker-dealer to disregard any amounts due less than $1,000. LO 16.d

A member firm's client has issued instructions for the assets held in an account at another member to be transferred to this account. The member firm has received the proper ACATS validation from the carrying firm. Therefore, it is expected that the transfer will be completed within A) five additional business days. B) three additional business days. C) one additional business day. D) two additional business days.

D) two additional business days. When using the ACATS system, validation takes place in one business day and the transfer in three business days. Please note the question states that validation has been received. That means the transfer will take place in two more business days (making it a total of three business days). Follow this trail. On Monday, the receiving firm sends the TIF to the carrying firm. On Tuesday, the carrying (delivering) firm validates the TIF and has three business days to make delivery (by Friday). On Wednesday, the validation gets to the receiving firm. Yes, the exam can be that tricky. LO 15.d

*Treasury bond maturities

Maturities that are 10 years or more -$100 to $5 million

TRACE can be best described as A. an execution system B. an order display and execution system C. quotation system D. a trade reporting system

D. a trade reporting system the TR in TRACE stands for trade reporting

Several investors open an account as TIC. For suitability purposes, financial info is based on A. most of the investors B. the largest investor C. the authorized trader D. all investors

D. all

Equity Linked Notes (ELNs)

Debt instruments where the final payment at maturity is based on the return of a single stock, a basket of stocks, or an equity index. -not suitable for MOST investors

Debit Spread

Difference in the value of two options, when the value of the one bought exceeds the value of the one sold.

*gift taxes for determining if there is taxable capital gain is that of the

Donor The client is considered to have acquired the security on the donors purchase price and at the donors purchase price

oils and gas partnership-developmental objectives

Drill near existing fields to discover new reserves

ETF benefits-expense ratio

ETFs tracking an index, the management risk is low

Earnings Per Share (EPS)

Earnings available to common / number of shares outstanding

who is responsible for the catch-up provision regarding IRA's?

Economic Growth and Tax Relief Re-consolidation Act of 2001 (EGTRR)

DMMS act as brokers when they

Execute orders other members leave with them

DMMS act as dealers when they

Execute trades for their own accounts

*T or F: employer contributions to 401k's and profit sharing plans are required

F, only defined benefit/contribution are required

*T or F: employer contributions to a defined benefit or defined contribution are NOT mandatory

F, they are MANDATORY

Other assets include

Goodwill, formulas, contract rights, brand names and trademarks

*Treasury Note Maturities

Maturities that are less than 10 years -mature at par or they can be REFUNDED which would be the government offering the investor a new security with a new interest rate & maturity as a alternative to a cash payment

*do s/o vote on dividends?

No -vote on BOD -stock splits -mergers -acqusitions

*REIT Taxation of the firm

Is similar to that of a Mutual Fund. REITs may qualify for conduit taxation on distributed net income. To qualify a REIT must obtain at least 75% of its income from real estate-activities and must distribute at least 90% of investment income to shareholders.

Which of the following statements regarding a Regulation T extension are true? It is granted by a self-regulatory organization. It is granted by the transfer agent. An extension is automatically granted once the extension request is made. Extensions are not automatically granted and may be denied.

It is granted by a self-regulatory organization. Extensions are not automatically granted and may be denied. In certain cases where the customer cannot make payment by the fourth business day following the trade date, the broker-dealer can request an extension from the self-regulatory organization (SRO) that is its designated examining authority (DEA), usually FINRA. Extensions are not automatic, may be denied, and are granted at the discretion of the SRO. LO 16.d

customer purchases 1000 shares of YFG at 60. stock declines to $50 update the following margin chart & produce Reg T & minimum maintenance: LMV: $60,000.00 DR: $30,000.00 EQ: $30,000.00

LMV: $50,000.00 DR: $30,000.00 EQ: $20,000.00 DR DOESN'T CHANGE Reg T: 25,000 Min. maintenance req.=12,500 The account is above minimum maintenance level but account is restricted.

Calculate excess equity after securities go up to 80000 LMV=60000 DR=30000 EQ=30000

LMV=80000 DR=30000 EQ=50000 Minimum maintenance req=.25x80000=20000 Reg T=.5x80000=40000 EE=50000-40000=10000

Tax basis for oil and gas

LPs must keep track of their tax basis or amount to determine their gain or loss upon the sale of their partnership interest -adjusts periodically

Tax filing requirements for limited partnership

Limited Partnership do NOT require a formal tax return since everything flows through to the investors

LP liability

Limited liability

Ratio Call Writing

involves selling more calls than the long stock position covers this strategy generates additional premium income for the investor, but also entails unlimited risk because of the short uncovered calls

oils and gas partnership-developmental tax features

Medium intangible drilling costs, immediate tax sheltering

oils and gas partnership-developmental risk

Medium to high not as high as exploratory

overlapping debt

Municipal accounting term referring to a municipality's share of the debt of its political subdivisions. Overlapping debt may be greater than the direct debt of a municipality, and both must be taken into account in determining the debt burden carried by taxable real estate within a municipality when evaluating MUNICIPAL BOND investments.

*Bond Funds

Mutual funds that invest primarily in bonds -bonds pay interest, they pay dividends if declared by the fund's BOD -div. paid on a quarterly basis -inverse relationship

SEC Regulation SHO mandates a locate requirement for short sales that is applicable to corporate bonds. NYSE issues. Nasdaq securities. municipal bonds.

NYSE issues. Nasdaq securities. This regulation mandates a locate requirement: before the short sale of any equity security, firms must locate the securities for borrowing to ensure that delivery will be made on settlement date. LO 16.c

*the more active a security, the ______the spread

Narrower

Trade Reporting Facility (TRF)

Nasdaq's service that receives trade reports from market makers in Nasdaq Global Market securities, Nasdaq Capital Market securities, CQA issues, OTC equity securities, and Nasdaq-listed convertible bonds

NASDAQ stands for

National Association of Securities Dealers Automated Quotations

interest paying bond duration

is always shorter than the time to its maturity since interest payments can be reinvested and earn additional interest

penny stocks trade on the

OTC bulletin board and the OTC link aka Pink sheets

municipal bonds are bought and sold in the

OTC market

Cashless collar

Occasionally, an investor may hedge his downside risk on a long position of stock for no out-of-pocket cash. Premiums are equal to each other Example: An investor is long 100 shares of XYZ at 50 and buys a 45 put at 3 and sells a 55 call for 3. The net cost is zero. In return, if the stock falls to a very low price, the investor can put the stock to someone at 45. He knows that he can never lose more than $500. The downside is that he sacrifices and upside potential beyond $55. This is known as a "cashless collar".

Primary trend

One year or more in stock prices

primary market

is the market where the proceeds go to the issuer of the securities sold

SEC regulations for securities issued by investment companies prohibit which of the following? Closed-end funds from issuing preferred stock Open-end funds from issuing preferred stock Closed-end funds from issuing bonds Open-end funds from issuing bonds

Open-end funds from issuing preferred stock Open-end funds from issuing bonds

Expense Ratio Formula

Operating Expenses / Average Net Assets -doesn't include sales charge

listed options

Options purchased through an exchange. Trades for listed options are cleared through the options clearing corporation.

Do Not Reduce (DNR) Order

Order that is not reduced by an ordinary cash dividend. Customer does not care if there is an execution due solely to her ex date reduction

Good-till-canceled order

Order to buy or sell at a specified price that remains in effect until it is executed by the broker or canceled by the investor All are canceled if unexecuted by end of business day April and October

Buy limits are used when the investor believes the stock price is currently

Overpriced Investor would like to buy if the stock dips in the market

Rights and Liabilities Between General Partners are listed in

Partnership agreement

LP is also known as a __________ investor

Passive

If interest rates are rising, which statements regarding collateralized mortgage obligations are true? Prepayment risk increases. Prepayment risk decreases. Extended maturity risk increases. Extended maturity risk decreases.

Prepayment risk decreases. Extended maturity risk increases. If rates are rising, homeowners are less likely to refinance. Therefore, prepayment risk will decrease. Similarly, if prepayments are declining, the estimated life of the underlying mortgages should increase. LO 12.d

Rule G-37 political contributions

Prohibits municipal securities firms from engaging in municipal securities business with an issuer for two years after any political contribution is made to an official of that issuer. In this context, municipal securities business refers to negotiated underwriting, not to competitive underwritings. The idea, is to limit large political contributions in return for being selected as underwriter for that issue. Contributions of up to $250 per election are permitted by municipal finance professionals as long as these individuals are eligible to vote for the issuer official.

inspection rights

Protect shareholders' interests by giving them right to inspect corporation's books and records after asking in advance to inspect and having proper purpose -no BOD minutes access

oils and gas partnership-income objectives

Provide immediate income from sale of existing oil

Credit Balance (CR)

Provides security to the broker/dealer that there will be cash available for the customer to purchase the securities if the market value of the securities rises.

An investor purchases 100 shares of XYZ common stock for $70 and sells it one year later for $50. Which of the following activities would violate the wash sale rule? Purchasing an XYZ call option 20 days after the sale Purchasing an XYZ put option 20 days after the sale Purchasing 100 shares of XYZ common stock 20 days after the sale Selling short 100 shares of XYZ common stock 20 days after the sale

Purchasing an XYZ call option 20 days after the sale Purchasing 100 shares of XYZ common stock 20 days after the sale The wash sale rule is violated when an investor sells a security at a loss and purchases the same or a substantially identical security within 30 days of the sale date. The IRS considers a call option substantially identical to the underlying stock because it represents the right to buy the shares. LO 13.h

Customer specific suitability

RR has reasonable basis to believe a recommendation is suitable for a SPECIFIC CUSTOMER -based on their investment profile

Income from which of the following investments is passive income? Real estate direct participation programs (DPPs) Vacation cottage rentals Real estate investment trusts (REITs) Collateralized mortgage obligations (CMOs)

Real estate direct participation programs (DPPs) Vacation cottage rentals Passive income results from DPPs and personal real estate rentals. REITs and CMOs are securities, and income from securities is considered portfolio income. LO 11.f

A customer Sells 100 shares at $15 a share short calculate Reg T and FINRA minimum

Regulation T=$750 FINRA=$2000 yes $2000 $2000 requirement

Blue Chip Stock

Well-established, usually large company that has a long history of earnings, growth, and dividend payments

Real estate program taxation

When a DPP shows a loss, that loss can be used to offset passive income and save on taxes

Short margin accounts

Sale of securities not owned by the seller, investor borrows securities from brokerage firm in anticipation that they will decline in value Credit balance=short sale proceeds +Reg T Deposit Equity=CR-SMV Minimum level of EQ-- If MV is less than $5 per share the greater of the MV or $2.50 if MV is $5 or more per share its the greater of $5 per share or 30% MV Initial Req't=$2000 or required RegT, whichever is greater

If your customer invests in a variable annuity and chooses to annuitize at age 65, which of the following statements are true? She will receive the annuity's entire value in a lump-sum payment. She may choose to receive monthly payments for the rest of her life. The accumulation unit's value is used to calculate the total value of the account. The annuity unit's value represents a guaranteed return.

She may choose to receive monthly payments for the rest of her life. The annuity unit's value represents a guaranteed return. When a variable contract is annuitized (distributed in regular payments, not as a lump sum), the number of accumulation units is multiplied by the unit value to arrive at the account's current value. An annuity factor is taken from the annuity table, which considers, for example, the investor's sex and age. This factor is used to establish the dollar amount of the first annuity payment. Future annuity payments will vary according to the separate account's performance. LO 9.c

trading flat

When a bond trades without accrued interest -occurs when the issuer is in default of interest payments

Reversal stock price

When a stock price is either moving up or down and it is halted and moves the opposite direction

MSRB governs the

issuance and trading of municipal securities -do not regulate issuers -requires to be ethical

unqualified legal opinion

issued by the bond counsel unconditionally

*HGU Zr 36 @ 6.45%

issuer= HGY Zr= sero coupon bond YTM= 6.45% maturity date= 2036

Which is the only debt security that is issued not at a discount but at face value?

jumbo CD

out of the money, sellers get to do what with the prem?

keep it & don't have to perform

in the money, buyer get to do what with the prem?

keep it & get the stock

the only difference between mutual funds and hedge funds is

lack of transparency

A popular way of reducing interest rate risk is by

laddering a bond or CD portfolio -as the shorter maturities come due, they are reinvested and now become the long ones

ETF benefits-portfolio specificity

large number of ETFs -has niches

*statutory voting benefits the

larger investors

accrued interest is calculated from the

last payment date up to but not including the settlement date (2 business days after the trade)

Trust Indenture

legal contract between the bond issuer and a trustee representing bondholders

the buyer of the money market is the

lender of the money market

purchasing a bond at a premium will always result in getting back par which means

less (a loss) than the original investment

penny stock

less than $5 and is not listed on any major exchange -speculative

*odd lot

less than 100 shares of stock

Which of the following would be found on a when-, as-, and if-issued confirmation? Trade date Settlement date Price Accrued interest

Trade date Price Information that does not appear on a when-issued confirmation can easily be remembered as SAT (settlement date, accrued interest, and total amount due). The trade date and price per bond are included on the when-issued confirmation. LO 15.a

DPP Roll-up

Transaction involving the combination or reorganization of one or more limited Partnership into securities of a successor corporation

discount & premium on bonds: if you pay more, you get ________ if you pay less, you get __________

less; more -the bigger the discount, the cheaper it is

long-term liabilities

liabilities owed for more than a year

investment constraints

limitations or restrictions that are specific to your client -liquidity -time horizon -ethical choices

*hedge funds are usually structured as a

limited partnership with no more than 100 investors

which of the 3 forms for margin accounts is optional?

loan consent form

non-amortizing loan

loans that require interest payments but no regularly scheduled principal payments -no fixed end date -backed by debt obligations

interest on MBS is taxed on

local, state and federal level

Investment objective of equipment leasing programs

Tax sheltered income

All customer securities must be held in ________________ to facilitate the process of hypothecation

Street name

oils and gas partnership-exploratory/wildcatting

locate undiscovered reserves of oil & gas

Strike prices (foreign currency options)

Strike prices of most foreign currency options are quoted in US cents. The Japanese yen is an exception and is quoted in 1/100th of a cent. Example: Strike price of .85 = 85 US cents except for Japanese yen where a strike price denoted as 121 = 1.21 US cents (121 X 1/100).

All investors interested in becoming LPs must compete a

Subscription agreement

T or F: dollar bonds are quoted as a percentage of par

T

Asset Allocation Funds

Split investments between stocks for growth, bonds for income, and money market instruments or cash for stability. -many asset allocations are target date funds

zero coupon bond is attractive because it

locks in a yield for investors

Call Spread

long call, short call

Put Spread

long put and a short put

If a customer buys $10,000 worth of stock in a cash account, then sells the shares for $12,000 without first paying for the buy side, and then requests the $2,000 profit, which of the following statements are true? The $2,000 profit cannot be sent to the customer until she pays for the buy side in full. The $2,000 can be sent to the customer, but the account will be frozen for 90 days. If the customer pays for the buy side in full on or before the fourth business day following trade date, status as a frozen account is lifted. Both trades must be switched to the customer's margin account, where buying and selling in this manner are acceptable practices.

The $2,000 can be sent to the customer, but the account will be frozen for 90 days. If the customer pays for the buy side in full on or before the fourth business day following trade date, status as a frozen account is lifted. Selling before paying is called freeriding and is prohibited. The penalty for freeriding in a cash account is that the account will be frozen for 90 days, and orders will not be accepted without cash or securities on deposit in advance. Transactions in margin accounts are subject to the same basic rule. LO 16.d

In July, a customer invested $10,000 in the ABC Mutual Fund. In December of the same year, ABC announced a long-term capital gains distribution. In May of the next year, the customer decided to redeem his shares for a capital gain. How are both of the capital gains treated for tax purposes? The capital gain distribution is treated as long term. The capital gain from redemption is treated as long term. The capital gain from redemption is treated as short term. The capital gain distribution is treated as short term.

The capital gain distribution is treated as long term. The capital gain from redemption is treated as short term. When long-term capital gains are distributed, the length of time an investor has owned the fund is not relevant; it's still a long-term distribution. However, redemption of shares follows the normal holding period rules. Therefore, when this customer sold shares 10 months (July to May) after the purchase, the gain, like any other gain from a holding period that does not exceed 12 months, is short term. LO 8.f

duration: the lower the coupon rate, the ________the duration

longer

at expiration of an options contract, the buyer ______________ the premium and the seller _______ the premium

loses; gains

to open this type of account the customer must meet certain minimal suitability requirements

margin account

Combined account

margin account with both long and short positions in different securities

financial leverage

the ability to use long-term debt to increase its return on equity -higher the leverage (debt/equity)ratio, the higher they are leverage

Interpositioning

The placing of a third party between a broker-dealer and its customer when filling a trade. This practice is prohibited unless it results in a better execution for the customer Usually results in a worse price for the customer

Rehypothecation

The pledging of a client's securities As collateral for a bank loan. Brokerage firms may rehypothecate up to 140% of the value of their customers' securities to finance margin loans to customers. Related item(s): hypothecation.

Workable Indication

The price at which a municipal Securities dealer is willing to purchase securities from another municipal dealer. The price may be revised if market conditions change.

Customer Identification Program (CIP)

The processes used by a financial institution to verify and validate the stated identity of a customer. It is normally part of the anti-money laundering processes of a financial institution. -OFAC (terrorist) -verifies identity

Adjusted Gross Income (AGI)

the amount of income remaining after subtracting all allowable adjustments to income from gross income

Which of the following statements regarding corporate zero coupon bonds are true? Interest is paid semiannually. The discount is in lieu of periodic interest payments. The discount must be accreted and is taxed annually. The discount must be accreted annually with taxation deferred until maturity.

The discount is in lieu of periodic interest payments. The discount must be accreted and is taxed annually. The investor in a corporate zero coupon bond receives the return in the form of growth of the principal amount over the bond's life. The bond is purchased at a deep discount and redeemed at par at maturity. That discount from par represents the interest that will be earned at maturity date. However, the discount is accreted annually, and the investor pays taxes yearly on the imputed interest. LO 5.a

In a functional allocation oil and gas program, which of the following statements are true? The general partner picks up all tangible drilling costs. The general partner picks up all intangible drilling costs. The limited partners pick up all tangible drilling costs. The limited partners pick up all intangible drilling costs.

The general partner picks up all tangible drilling costs. The limited partners pick up all intangible drilling costs. In a functional allocation program, the general partner picks up all tangible drilling costs, while the limited partners pick up all intangible drilling costs. As intangible drilling costs are deductible as they are incurred, this type of program benefits the limited partners. Tangible drilling costs, however, are deductible pro rata over the estimated life of the well. LO 11.f

SMA is equal to

The greater of the EE or the SMA already in the account

FINRA Rule 2330, which deals with members' responsibilities regarding variable annuities, applies under which of the following circumstances? The initial purchase of a deferred variable annuity. The initial purchase of an immediate variable annuity. The initial subaccount allocations. The initial subaccount reallocations.

The initial purchase of a deferred variable annuity. The initial subaccount allocations. This rule applies to recommended purchases and exchanges of deferred (not immediate) variable annuities and recommended initial subaccount allocations (there is no such thing as initial reallocations). On the other hand, this rule does not apply to reallocations among subaccounts made or to funds paid after the initial purchase or exchange of a deferred variable annuity. LO 9.e

political risk

The likelihood that political forces will cause drastic changes in a country's business environment that will adversely affect the profit and other goals of a particular business enterprise. -nonsystematic risk

Which of the following statements regarding limited partnerships are true? The maximum commission in selling partnership offerings is 5%. The maximum commission in selling partnership offerings is 10%. Commissions taken are deducted from the original investment to determine beginning cost basis. Commissions taken are not deducted from the original investment to determine beginning cost basis.

The maximum commission in selling partnership offerings is 10%. Commissions taken are not deducted from the original investment to determine beginning cost basis.

Which of the following statements regarding variable annuities are true? The number of accumulation units is always fixed throughout the accumulation period. The number of accumulation units can rise during the accumulation period. The number of annuity units is fixed at the time of annuitization. The number of annuity units rises once annuitization begins.

The number of accumulation units can rise during the accumulation period. The number of annuity units is fixed at the time of annuitization. The number of variable annuity accumulation units can rise during the accumulation period when additional units are being purchased. When a variable annuity contract is annuitized, the number of annuity units is fixed. LO 9.c

outstanding stock

The number of shares held by investors; excludes treasury shares

Which of the following procedures are required to open and maintain an options account? The registered representative must document that the client has received a current Options Clearing Corporation (OCC) disclosure document. Which of the following procedures are required to open and maintain an options account? The registered representative must document that the client has received a current Options Clearing Corporation (OCC) disclosure document. The client must verify and return his background and financial information within 15 days. If there is a material change in the client's financial status, amendment of the options agreement is required. Any recommendations made must consider the financial needs and situation of the client.

The registered representative must document that the client has received a current Options Clearing Corporation (OCC) disclosure document. The client must verify and return his background and financial information within 15 days. If there is a material change in the client's financial status, amendment of the options agreement is required. Any recommendations made must consider the financial needs and situation of the client. The client must have a current OCC disclosure document. An understanding of the client's financial situation is required to make any recommendations. Changes in the client's status must be updated as soon as possible, and the option agreement form must be returned within 15 days of account approval. LO 10.j

Legislative Risk

The risk that new laws reduce the value of a security, such as a change in the tax laws increasing tax rates on interest received from debt investments. -nonsystematic risk

Inflation Risk (Purchasing Power Risk)

The risk that the return from an investment will not cover the loss in purchasing power caused by inflation. -systematic risk

If a customer writes 2 ABC Feb 90 puts at 8 and buys 2 ABC Feb 80 puts at 2, which of the following statements are true? The spread is bullish. The spread is bearish. The breakeven point is 84. The breakeven point is 86.

The spread is bullish. The breakeven point is 84. This is a credit put spread (the net credit being 6 points per share) in which the breakeven point is calculated by subtracting the net premium (debit or credit) from the higher strike price (90 - 6 = 84). A credit put spread is like the net sale of a put, and buying the lower strike price in any spread (put or call) is bullish. LO 10.h

Market Capitalization

The total value of a company's issued shares

Odd Lot Theory

Theory that the ODD LOT investor who are small traders usually sell and buy at the wrong times and that profits can be made by acting contrary to odd-lot trading patterns.

Build America Bonds (BABs)

These bonds were created under the Economic Recovery and Reinvestment Act of 2009 to assist in reducing costs to issuing municipalities and stimulating the economy. While bonds to fund municipal projects have traditionally been sold in the tax-exempt arena, a BABs are taxable obligations. Bondholders pay tax on interest received from BABs, but tax credits are provided in lieu of the tax-exempt status usually afforded the interest on municipal securities. These bonds attracted investors who would normally not buy tax-exempt municipal bonds and expanded the pool of investors to include those in lower income tax brackets, investors funding retirement accounts where tax-free securities would normally not be suitable, public pension funds, and foreign investors. *There are 2 types of Build America Bonds issued: Tax credit BABs and direct payment BABs.*

Which of the following statements regarding U.S. government agency obligations are true? They are direct obligations of the U.S. government. They generally have higher yields than direct U.S. obligations. The Federal National Mortgage Association (FNMA) is a publicly traded corporation. Securities issued by the Government National Mortgage Association (GNMA) trade on the NYSE floor.

They generally have higher yields than direct U.S. obligations. The Federal National Mortgage Association (FNMA) is a publicly traded corporation U.S. government agency debt is an obligation of the issuing agency. This obligation causes agency debt to trade at slightly higher yields that reflect this greater risk. FNMA securities and GNMA pass-through certificates trade over the counter. GNMA is the only agency whose securities are direct U.S. government obligations. LO 7.c

Debt Statement (GOs)

This is also used in the analysis of GO debt and it includes the estimated full valuation of taxable property, the estimated assessed value of property, and the assessment percentage. To evaluate the municipality's debt structure, an analyst calculates "total debt", the sum of all bonds issued by the municipality, and subtracts "self-supporting debt" from this figure. Although revenue bond debt is included in total debt, it is backed by revenues from the facility it financed and is not a burden on the municipality's taxpayers. The result is the municipality's "net direct debt", which includes GOs and short-term notes issued in anticipation of taxes or for interim financing. To net direct debt, the analyst adds "overlapping debt".

a firm can impose a maintenance level higher above FINRA minimum maintenance rule levels

This is house minimum

Which publication provides the most up to date information relevant to the secondary municipal bond market?

Thomas muni market monitor

Footnotes to Financial Statements

Tie up loose ends of financial statement; accounting methods and disclosure *found at the bottom of the financial statement and can be several pages long

Technical- resistance level

Top of the trading range

Treasury Inflation-Protected Securities (TIPS)

U.S. Treasury securities that protect investors against inflation -issued with a fixed interest rate -principal amount is adjusted semiannually by an amount equal to the change in Consumer Price Index (CPI) -interest payment is given every 6mo -in times of inflation, interest payments increase -in times of deflation, interest payments decrease -exempt from state and local income taxes -subject to federal taxation -when the principal is adjusted for inflation it is considered reportable income

*the investment company act of 1940 does not include variable annuities in its definition of investment companies however, the separate account is most often a

UIT

*when interest rates rise, the value of a call option goes

UP -opposite of what we learned earlier

*Eurodollars are paid in

USD

ADR dividends are in

USD

Form 1099-DIV

Used by banks and other financial institutions to report dividends and other distributions to taxpayers and to the IRS

Share Identification

Used to identify the specific per share cost basis when shares are sold

Depletion allowances in oil and gas programs apply to the

Using up of natural resources like oil and gas

Debit Register (DR)

the amount of money borrowed by the customer

Oil and gas taxation-intangible drilling costs

Write off for the expenses of drilling are usually 100% deductible in the first year of operation -such as wages, supplies, fuel costs, insurance -no salvage value

Which of the following registers the securities and packages the program for a limited partnership? A) Syndicator B) Property manager C) General partner D) Limited partners

a A syndicator handles the registration of the limited partnership units. LO 11.c

A customer is considering adding a real estate investment trust (REIT) to her portfolio. She lists all of the following as advantages. You correct your customer and point out that one of them is not an advantage of investing in REITs. Which of the following is not an advantage of investing in REITs? A) Dividend treatment B) Using real estate as a potential hedge against the movement of other equity securities the customer owns C) Being able to divest of the shares easily D) Having a professionally managed portfolio of commercial real estate assets

a Of those listed, only dividend treatment can be identified as not being an advantage. While the expectation of receiving dividends is inherently good, dividends paid by REITs to their shareholders are not recognized as qualified and are therefore taxable to the investor at their full ordinary income tax rate. The shares are traded on exchanges or over the counter and are considered liquid, and having professionally managed assets should be a plus. While real estate valuation and price movements are subject to many forces, historically, real estate has provided some hedge against the movements of other equity securities. LO 11.b

The IRS will generally consider a direct participation program to be an abusive tax shelter unless the program can show a profit motive. A popular method of measuring the economic viability of a DPP is A) cash flow analysis. B) income to debt analysis. C) the ratio of gains to losses. D) passive loss analysis.

a On the exam, there are two accepted measures of the economic viability of a DPP. Those are cash flow analysis and internal rate of return (IRR). Cash flow analysis compares the income to the expenses, not the debt. LO 11.g

One way in which open-end investment companies differ from closed-end investment companies is that an open-end investment company's shares A) outstanding will vary in number at any point in time. B) may be priced at a premium or discount relative to its net asset value. C) are purchased and redeemed based on supply and demand. D) are traded in the secondary markets rather than on an exchange.

a Open-end investment companies are capitalized with a continuous offering of new shares. As a result, the number of shares outstanding is constantly changing. It is the closed-end company, traded in the secondary markets, whose share prices are based on supply and demand, which causes them to be bought or sold at a premium or discount to the NAV. LO 8.b

An investor in an equipment-leasing direct participation program (DPP) using straight-line depreciation would probably not be concerned about A) the likelihood of recapture. B) legislative risk. C) the quality of the management. D) liquidity risk.

a Recapture of deductions is a concern when accelerated, but not when straight-line depreciation is used. In any business, there is always concern about the quality of the management. By and large, DPPs are not liquid investments, so an investor needing a quick sale may have problems. The nature of DPPs tends to make them more sensitive to legislative risk than most other securities. LO 11.g

A stock mutual fund wishes to advertise itself as diversified. To be able to do so, the fund must invest its total assets, such that A) at least 75% of its assets meet stated diversification requirements. B) it holds no more than 10% of the voting stock of any one company. C) its portfolio consists of at least 15 different stock holdings D) no more than 5% of its assets in any one company.

a The Investment Company Act of 1940 requires that a minimum of 75% of the assets of the diversified company meet stated requirements. Those requirements include the following: At least 75% of the fund's total assets must be invested in cash and securities that are not issued by the fund or any of its affiliates. Within that 75%, no more than 5% of the fund's total assets can be in a single stock. Within that 75%, no holding can represent more than 10% of the voting control of a single company. Please note that there are no restrictions on the nondiversified 25%. It can all be in a single stock. That means 30% of the fund's total assets can be in one company. Likewise, that 25% can be used to purchase a controlling interest in companies. LO 8.a

A customer, currently finding the income offered from a money market fund quite low, asks if there might be any debt instruments providing income that one could expect to at least keep pace with inflation as well as offer some tax relief. What suitable recommendation could be made that meets the investor's investment objectives? A) TIPS B) Participating preferred C) GNMAs D) U.S. T-bills

a The investor has requested a debt security that can meet three criteria: provide income, keep pace with inflation, and offer tax relief. Treasury Inflation Protection Securities (TIPS) are specifically designed to provide income that keeps pace with inflation. In addition, the interest is tax exempt at the state and local level, providing some tax relief. GMNAs will provide income, but they are fully taxable on a state and federal level and offer no inflation protection. T-bills provide income that is probably lower than the money market fund the investor was unhappy with, and participating preferred stock is not a debt security. LO 7.a

it is required that every registered investment company keeps its assets with

a custodian (usually a bank)

*trade blotters are

a daily record of all activity including cash received and disburse, securities received and delivered, and identification of securities bought and sold that day -does NOT include client information or settlement days -blotters are to be posted no later than the 1st business day following the activity

Tennessee Valley Authority (TVA)

a federal corporation established to construct dams and power plants in the Tennessee Valley region to generate electricity as well as to prevent floods

12b-1 fee

a fee that an investment company charges to help pay for marketing and advertising a mutual fund

amortizing loan

a loan on which the borrower makes monthly payments that include interest on the loan plus some part of the loan balance

*nonrecourse loan

a loan that carries neither a penalty nor further obligation to repay if not paid back -non recourse loans add to the investor's basis

holding a quote

a municipal securities dealer may quote a bond price that is firm for a certain time (1hr usually). -usually done so a dealer who requested a quote has time to search for a better quote

records of associated persons Rule G-7

a municipal securities dealer must obtain and keep on file specific information of associated persons -U4 & U5 needs updating within 30 days of changes -must be kept on file for 3 years after termination

lease-rental bonds

a municipality issues bonds to finance office construction for itself or its state or community

Balanced Fund

a mutual fund that invests in both stocks and bonds -aka hybrid funds

preferred stock

a nonvoting share of ownership in a corporation that pays a fixed dividend -purchased for income

if the bond has a YTC lower than its CY, it is trading at

a premium

cumulative voting

a system in which a shareholder can accumulate all of his or her votes and vote them all for one candidate or split them among several candidates

Expense Ratio

the annual cost of management fees and operating expenses as a % of fund's net assets

What might happen if a limited partner begins making business decisions for the partnership? A) He may be removed from the partnership completely. B) He might jeopardize his limited liability status. C) There would be no effect because of the partnership democracy. D) He ascends to general partnership status as a reward for his decision-making contribution.

b

ETF risk-index risk

mimics the composition of the specified index -if one or more of the securities in that index underperforms, the stuck with it until the index removes the security

If an investor expects to have a large amount of passive income over the next two years, which of the following programs listed will most likely lead to the largest amount of shelter? A) Equipment leasing B) Oil and gas drilling C) Undeveloped land purchasing D) Real estate income

b Passive income can only be sheltered by passive loss, so the real estate income program will only add to the income. Oil and gas drilling programs allocate the majority of investment dollars to drilling. These are intangible drilling costs (IDCs), which are 100% deductible when drilling occurs. Undeveloped land has very little in the way of losses, and equipment leasing programs usually generate income shortly after starting. LO 11.f

GNMA: if interest rates fall, homeowners

pay off their mortgages early

GNMA: if interest rates rise, homeowners

pay off their mortgages slowly

the buyer of an option ________the prem

pays

defined benefit plan

pension plan that guarantees a specified level of retirement income

buy stop order are entered

above the current market

proxy

absentee ballot -canceled if s/o goes to the meeting, dies or sends an additional one in

Margin Account

account at a brokerage firm that requires a substantial deposit of cash or securities and permits the purchase of other securities using credit granted by the brokerage firm

Money Market Funds (MMFs)

accounts that pool money from individuals and invest in securities that have a short-term maturity, such as one year or less -check writing privileges -nav is at $1

unless a bond is trading flat, the bond cost to the buyer and the proceeds to the seller include

accrued interest -accrued interest increases the bond cost to the buyer and proceeds to the seller

Investors placing zero-coupon bonds in their portfolios are most likely to be looking to provide accumulation of capital. current income. protection against reinvestment risk. tax deferral.

accumulation of capital. protection against reinvestment risk. Zero-coupon bonds are always purchased at a discount because they pay no interest. At maturity, the bondholders receive the maturity value. That represents the initial investment plus interest. Therefore, the investors are receiving more capital than invested (capital accumulation). Zero-coupon securities avoid reinvestment risk because there are no periodic interest payments to be reinvested. When you purchase one of these securities, the quoted yield to maturity is exactly what you will earn if you hold it to the end. With no interest payments, there is no current income. There is no tax deferral with a zero. In fact, unless it is a zero coupon municipal bond, there is phantom income; income not currently received but currently taxable. LO 5.a

Non-Qualified

after tax -employer sponsored plan where there are no tax advantages other than that the pay isn't received until sometime later when the individual should be in a lower tax bracket

nonqualified plans are funded by

after tax $

ERISA - nondiscrimination

all employees must be treated impartially through a uniformly applied formula

for checks made payable to TIC or JTWROS they have to be made payable/endorsed by

all owners

the 5% markup policy applies to

all transactions in nonexempt listed or unlisted securities traded on an exchange or OTC -DOESN'T APPLY TO MUNICIPAL SECURITIES, MUTUAL FUNDS, VARIABLE ANNUITY CONTRACTS, OR SECURITIES SOLD IN PUBLIC OFFERINGS

Prepaid Tuition Plans

allow contributors to prepay college tuition and other fees for a designated beneficiary for a set number of academic periods or course units while locking in current tuition costs

Payroll Deduction Plan

allows employees to authorize their employer to deduct specified amounts for retirement savings from their paychecks; the money is deducted after taxes are paid and may be invested in any number of retirement vehicles as the employee's option.

rights offering

allows stockholders to purchase common stock below the current market price -rights are NOT ownership

workout quote

an approximate figure used to provide the buyer or seller with an indication of price, not a firm quote Special handling will be required to accommodate a particular trade: market is too thin it is too big

sweep account

an asset management account that sweeps any unused balance in the brokerage account into a money market investment at the end of each business day

the reports that a IC must report annually include of

an audited balance sheet and income statement

Ballon Maturity

an issuer pays part of a bond's maturity before the final maturity date, but the largest portion is paid off at maturity

American style option

an option that can be exercised at any time until the expiration -nearly all equity options are American

Market-on-close order

an order to buy or sell during the close On NYSE, orders must be entered before 3:40pm ET

limit order

an order to either buy a security at a specified or lower price or to sell a security at or above a specified price

Real Estate Investment Trust (REIT)

an organization that purchases, owns, and manages real estate for its investors

the number of annuity units is calculated when

an owner annuitizes the contract

12b-1 is expressed as a _____________ amount but is charges ___________

annual amount; quarterly

the death benefit payable under a variable life insurance policy is adjusted on an

annual basis and can increase or decrease based on the performance of the separate account compared with the assumed interest rate (AIR)

*best feature of using a broker's broker is

anonymity. your firm doesn't disclose the identity of your customer and the broker's broker does not disclose the identity of the buyer of your client's bonds

quotation

any bid or offer of municipal securities

MSRB Rule G-21 advertising

any material for use in public media is considered advertising -principal of the dealer must approve all advertising before use, and a copy of each advertisement must be kept on file for 4 years

listed security

any security that is bought or sold on an exchange

Quantitative Suitability

requires a series of recommendations to be appropriate

MSRB Rule G-15

requires members to provide customer with written confirmation of transactions

penny stock rule: disclosure of compensation

requires members to provide penny stock purchasers with information on the compensation to be earned by both the member and the RR as the result of the transaction -prevents excessive markups

penny stock rules: frequency of customer statements

requires members to provide purchasers with monthly statements showing the market value of each stock purchased

A blind pool offering A) is one in which the properties are purchased on a lottery basis. B) is connected with oil and gas leases. C) is one in which 25% or more of the properties are not specified. D) generates nonallocated income.

c Many times, large real estate or oil and gas programs are offered in the form of a blind pool. In a blind pool, 25% or more of the specific properties (in real estate) or sites (in oil and gas) have not been identified at the time of the offering. When investing in a blind pool, the participants are relying on the expertise of the program sponsor to select locations that will prove profitable. LO 11.g

penny stock rule: risk disclosure document

before their initial investment, individuals must be given a risk disclosure document and the firm must receive it back signed and dated -once sent after 2 days the firm can initiate first trade

sell stop orders are entered

below the current market

treasury securities are available in

book entry form

when the issuer of an insured municipal bond defaults, what does the insurance company do?

both principal and interest are returned over the remaining term of the bond

MSRB rule g-30

broker dealers cannot charge a commission and a markup or markdown on the same transaction

when are debt securities redeemed?

by the issuer on a specific maturity date

All of the following statements regarding the Federal National Mortgage Association (FNMA) are true except A) FNMA pass-through certificates are not guaranteed by the U.S. government. B) interest on FNMA certificates is taxable at all levels. C) FNMA is owned by the U.S. government. D) FNMA is a publicly held corporation.

c FNMA is a publicly held corporation; it is not owned by the federal government. The interest income on all mortgage-backed securities is fully taxable on the local, state, and federal levels. Though FNMA is a government agency, FNMA pass-through certificates are not guaranteed by the U.S. government. The only tested U.S. agency whose securities are considered direct obligations of the U.S. government is the Government National Mortgage Association (GNMA). LO 7.c

High-tax-bracket investors are likely to receive the most favorable tax treatment from investing in A) GNMA pass-through securities. B) preferred stock. C) municipal bonds issued by a political subdivision of their state. D) bonds issued by the U.S. Treasury.

c Municipal bonds issued by a political subdivision of the investor's state are free of federal, state, and local income taxes. Treasury securities are taxed on the federal level, but not the state and local levels. GNMAs are taxed at every level. Qualifying dividends on preferred stock are taxed on all levels, although generally at a rate not exceeding 20%. LO 7.e

When analyzing a company's balance sheet, you notice that it is using the first in, first out accounting method to value its inventory. This information is most likely shown A) on the cover of the balance sheet or at the top of the first page. B) next to the inventory listing in the current assets portion of the balance sheet. C) in a footnote to the balance sheet. D) at the end of the balance sheet in a summary statement required by the SEC.

c Notations regarding accounting methods used, such as those for valuing inventory, would generally be found in the footnotes of the balance sheet. LO 8.d

Limited partnership programs are categorized as direct participation programs. The term direct participation refers to A) the ability, through the partnership democracy, for each partner to have her vote flow through to the general partner. B) the ability of any partner, limited or general, to participate in the running of the partnership. C) the flow-through of profits and losses of the partnership to the individual limited partners. D) the general partners directly participating in the day-to-day management of the partnership.

c Understanding the flow-through concept is critical with DPPs. Only DPPs allow flow-through of losses. LO 11.f

When discussing mutual funds with a customer, which of the following statements is not prohibited? A) The income yield of the fund consists of both dividends and capital gains. B) Get a few friends to join with you to form an investment club, and you may qualify for a breakpoint. C) Buy shares of different funds in the same fund family, and you may qualify for a breakpoint on the total purchase. D) Buy the shares on record date to receive the dividend.

c ost funds provide a combination privilege, allowing investors to aggregate purchases made in different funds in the same family to qualify for a breakpoint. The income yield of a mutual fund includes dividends only. A group of friends is not eligible for a breakpoint. (Investment clubs are not eligible.) Selling dividends is a prohibited practice because of the immediate tax liability incurred with the dividend and the share price adjustment that results after the dividend distribution. LO 8.d

which of the following currency options are not traded in the US options exchanges? a. British pound b. Swiss Franc c. USD d. Canadian dollar

c they are called FCOs since they are only available in the US on foreign currencies

if a mistake was made on the order ticket, no charge can be made on the order without

the approval of the principal or the branch manager

*REITS offer investors ____________ and _____________ but do not offer _____________________________________

dividends; gains; flow through losses like DPP

*current yield of stock is calculated by

dividing the current dividend by the current stock price

Eurodollar bonds

dollar-denominated bonds sold outside the United States and the issuer's country but the interest and principal are paid in USD ex: Indian bank held denominated bonds issued by a Korean company

*the expense risk fee covers the risk that

the costs of administering and issuing the policy may be greater than assumed

investors can use ____ to protect a short stock position

calls

European style option

can be exercised only on the expiration date -nearly all yield and index-based are European

buy a put and it expires results in a

capital loss

when an investor sells a Treasury security at a price lower than the purchase price it is considered a

capital loss

Coverdell's and ESA's can only be opened as a _____ account

cash

a UGMA/UTMA account can only be opened as a ______ acct.

cash

401ks can only be opened as a ____ acct.

cash acct

IRAs can only be opened as a ____ acct.

cash acct

Current assets include

cash, accounts receivable, inventory and prepaid expenses

if a put is exercised, ____ will be delivered within _____ day(s)

cash; 2

a benefit of fee based account reduce the likelihood of this abusive sales practice:

churning

laws and regulations as investment contraints

client can't make the purchase due to lack of accredited status & security isn't for sale in their state

which has a higher growth opportunity preferred or common stock?

common stock does -preferred stock is subject to inflation risk

who has the opportunity to benefit from an increase in a price of shares?

common stock holders

which will have a lower coupon rate a nonconvertible or convertible bond?

convertible bond because of the conversion feature -conversion price is higher than the price of common stock

which preferred stock would be suitable for an investor who likes a fixed dividend but would also like the opportunity to take advantage of capital appreciation?

convertible preferred stock

investors use this to reduce risk on short calls

credit call spread

investors use this to reduce risk on short puts

credit put spread

ADR investors are subject to _________ risk

currency and political

Long Margin Account

customers purchase securities and pay interest on the money borrowed until the loan is repaid

if a customer wants to buy a security that the rep thinks this investment is unsuitable the rep must,

tell the client that she/he feels that it is unsuitable if the client insists on the purchase it is coded as unsolicited

The rights and liabilities of general and limited partners are listed in A) the certificate of partnership. B) the partnership title. C) the Uniform Limited Partnership Act. D) the partnership agreement.

d

A municipal securities dealer informed XYZ municipal bond fund that it was the leading retailer of XYZ shares and that, in return, XYZ should employ the dealer in effecting more transactions for the fund's portfolio. Which of the following statements regarding the request is true? A) It is not permissible because municipal securities dealers are not allowed to execute trades for the portfolios they underwrite. B) It is permissible because it suggests a more reciprocal arrangement between the two parties. C) It is permissible because MSRB rules do not cover municipal bond issuers or funds. D) It is not permissible because it violates the MSRB anti-reciprocal rule.

d An investment company must select a dealer to execute its portfolio transactions based on services provided. It is a violation of the anti-reciprocal rule (Municipal Securities Rulemaking Board Rule G-31) for an investment company to choose a firm to trade its portfolio based solely on sales of units or shares of the fund. LO 8.j

A fund aims for consistent total returns. The management is empowered to shift assets among stocks, bonds, and short-term fixed-income securities in accordance with its projections of future market conditions. Because of its ability to diversify among many investment instruments, the fund has the potential to provide maximum returns while reducing volatility. This information describes which of the following mutual funds? A) ABC Stock Index Fund B) ATF Capital Appreciation Fund C) KPL Government Income Fund D) KPL Asset Allocation Fund

d Asset allocation funds shift assets among stocks, bonds, and short-term fixed-income securities in accordance with projections of future market conditions. LO 8.g

You are reviewing a company's financial statements to assist a customer. What kind of information is most likely to be found in the footnotes to those financial statements? A) The name, address, and contact information for the firm's chief executive officer B) The names of the company's most formidable competitors C) The length of time each director has served on the board of directors D) Accounting methods used

d Footnotes to a company's financial statements will contain important financial information such as accounting methods used, as well as important management philosophy that may impact the company's overall financial health and performance. LO 8.d

The STU Corporation has issued common stock, preferred stock, promissory notes, and mortgage bonds. Should STU enter bankruptcy proceedings, the order of payment against claims would be A) the promissory notes, the mortgage bonds, the preferred stock, and the common stock. B) the preferred stock, the common stock, the mortgage bonds, and the promissory notes. C) the mortgage bonds, the preferred stock, the common stock, and the promissory notes. D) the mortgage bonds, the promissory notes, the preferred stock, and the common stock.

d In a bankruptcy, secured creditors, such as those with a mortgage against real property, have the first priority. They are followed by unsecured creditors, such as holders of promissory notes, with stock holders coming last. Preferred stock is "preferred" over common stock in both liquidation priority and payment of dividends. LO 5.b

ADR registered owner is

the US bank that is responsible for them

Investment Company Act of 1940 BOD

in management investment companies (open&closed) cannot have a BOD that consists of more than 60% of persons who meet the definition of interested persons of the investment company -40%+ must be non-interested

financial risk

inability to meet debt obligations which could lead to bankruptcy -relates primarily to those companies that use debt financing (leverage)

mutual fund dividend

income paid to investors out of profits earned by the mutual fund from its investments -defined as qual or non-qual -paid quarterly -requires written statement to accompany dividend payment

if the separate account performance for the year is greater than the AIR, the death benefit will

increase

in times of inflation for TIPS, interest payments

increase

*CMO taxation

interest taxed at federal, state and local levels

equity REIT

invest in and operate commercial properties -receives rental income & possible capital gains upon a future sale of projects

global funds

invest in foreign and US securities -currency and political risk -diversity

Option Income Funds

invest in securities on which options can be written and earn premium income from writing options

value funds

invest in stocks considered to be selling below their true value -underperforming

if a party in a JTWROS dies, the account cannot be transferred until

death certificate is received

FNMA issues

debentures, short term discount notes, and MBS

investors use this to reduce cost on long calls

debit call spread

*when analyzing revenue bonds, one should look at

debt coverage ratio=available revenues/debt service requirements -the higher the better

*in a gross revenue pledge, what is paid first & second and from where?

debt service is paid first from gross revenues -second is operations and maintenance are paid after debt service

If the Federal Reserve increases the interest rate dramatically, the market price of all bonds will

decline regardless of credit quality

if the separate account performance for the year is less than the AIR, the death benefit will

decrease

*durable POA

designed to provide the appointed power of the acct even upon the grantor's incapacitation *-can be due to physical or mental cause -death of individual=terminates the dPOA

Credit Put maximum loss

difference between strike prices - net premium

Debit Call Spread maximum gain

difference between the strike prices - net debit -ML + MG =SPREAD

Debit Put Spread maximum gain

difference between the strike prices - net debit -ML + MG =SPREAD

If a bond has a YTM greater than its coupon, the bond is trading at

discount

if a bond has a YTM less than its YTC, the bond is trading at

discount

*if a 5% coupon bond is currently yielding 6% is it selling at a discount, par, or a premium?

discount current yield is higher than the coupon rate

Money Market Instruments are issued at a

discount to their face value with the difference paid at maturity representing the interest

most money markets are issued at a

discount, they do not pay interest

ETF benefits-taxation

investor ONLY realizes a capital gain or loss upon the sale -As a RIC, at least 90% of the NII is distributed

Dark Pools

electronic trading networks where participants can anonymously buy or sell large blocks of securities -not open to the public -does not affect public quotes or prices

Deferred Compensation Plan risk

employee covered by the plan has no right if business fails

section 457 plans can be used by

employees of the state, political subdivision of a state, and any agency or instrumentality of a state

in addition to interest payments for CMOs investors must recieve their ____________

entire principal before the nest tranche begins to receive principal payments

the seller of the money market is the

entity borrowing the money

mutual fund benefits-exchanges within a family of funds

exchange privileges or conversion privilege within the family of funds at NAV with no additional sales charge

T or F: A registered IC must use a bank as a custodian with FDIC coverage

false, does not require FDIC coverage

municipal bonds are exempt from

federal income tax

interest paid on farm credit system, and federal home loan bank securities is taxable at the

federal level -exempt from the state and local levels usually

bond trustee is usually a

financial institution, such as a trust company -trustees represent the investors -makes sure the borrower lives up to the terms of the loan

quantitative analysis-GO

focuses on objective information regarding a municipality's population, property values, and per capita income

qualitative analysis-GO

focuses on subjective factors that affect a municipality's securities. the community's attitude toward debt, taxation, population trends, plans being undertaken in the area

confirmation delivery

for each transaction, the customer must be sent or given a written confirmation of the trade at or before the completion of the transaction

*Eurobonds are paid in

foreign currency

an ADR represents a foreign/domestic security trading in a foreign market/domestic market

foreign security trading in the US/domestic

options on indexes allows investors to profit

from the movements of markets and hedge against these market swings

ERISA funding

funds contributed in the plan must be segregated from the corporations assets and be in the best interest of the participant

If a bond is trading at a discount, we can determine that the YTM is >/</= the coupon

greater than

for a fixed annuity, the rate of return is

guarenteed

thinly traded security

have a large spread between buy and sell

the lower strike price on a call the ____________the premium

higher

the higher the risk, the

higher return

Debit Put Spread breakeven

higher strike price - net debit

Dealer's Inventory Costs

if a customer's buy order is filled from the broker-dealer's inventory, the net price to the customer is based on the prevailing inside market -the price at which the BD acquired the stock being sold to the customer has no bearing on the net price to the customer must be reasonably related to the current market.

death benefit fee

if an investor died during the accumulation period, the bene will receive the greater of the current value of the account of amount invested

UIT benefits-income

if the UIT is composed of debt securities, it will generally provide higher income than a mutual fund with a similar portfolio -no management fee since they don't have a portfolio manager

call buyers want to be ______ the money

in

put buyers want to be ______ the money

in

FINRA rule-non cash compensation

in IC, mutual fund distributors may pay commissions and concessions to member only in the form of cash -gifts are an exception cannot exceed more than $100

Reverse stock split

investor's shares are split in half for the same total amount of money REVERSE DOWN 1:2 1:3

marketability-municipal bond-insurance

municipal bond issuers can issuer their securities' principal and interest payments by buying insurance from a number of insurers -insured bonds are generally issued with lower coupon rates because investors will accept lower rates of returns for the added safety insurance affords -interest and principal will be paid as scheduled

BD as FA-financial advisory relationship exists when a

municipal dealer provides financial advisory or consulting services to an issuer with respect to a new issue for a fee or other compensation -relationship must be in writing before, upon or promptly after inception and describes compensation

A prospectus for a variable annuity contract must provide full and fair disclosure. is required by the Securities Act of 1933. must be filed with FINRA. must precede every sales presentation.

must provide full and fair disclosure. is required by the Securities Act of 1933. A variable annuity is a security and must be registered with the SEC, not FINRA. As part of the registration requirements, a prospectus must be filed and distributed to prospective investors. The time of distribution of the prospectus can be before the sales presentation or at the same time as the presentation. It is incorrect to state that it must precede every sales presentation. LO 9.a

*options: credit spread=_____________=__________________

narrow=expire

specified adult

natural person who is 65 years old + or 18+ who the RR believes to have a mental or physical impairment to which they are unable to protect their own assets

OTC is a ______ market

negotiated -market maker chooses to deal & buy stocks

Debit Call Spread maximum loss

net debit -ML + MG =SPREAD

Debit Put Spread maximum loss

net debit -ML + MG =SPREAD

cash flow

net income or loss + depreciation (or non cash charges)

Credit Call Spread maximum gain

net premium

Credit Put maximum gain

net premium

Accredited Investor

net worth over $1M (excludes residence) OR individual with an income of $200k+ OR couple with $300k+ of income & has expectations of meeting the same salary

customer pays 43.25 net means

net= no commission

fixed annuity, monthly payments

never falls below guaranteed minimum

are fee based accounts wrap-fee accounts?

no

do parents have legal control over a UTMA/UGMA?

no

do section 457 have a 10 federal penalty if w/d before 59.5? a

no

roth IRA distribution age

no age requirement to start distribution

is tax-exempt income from municipal securities shown in AGI?

no but is on on the 1040

UIT risk-market risk

no ongoing management risk and the portfolio is fixed -like a sinking ship, if a few portfolios are doing bad there's no way to stop it

*can students participate in an education's TSA?

no, because the plan is only available to employees

*is a customer's signature required on a new account form?

no, but a partner, officer or manager signifying that the acct. has been accepted in accordance w/ the member's policies and procedures is

coupon rate is also know as

nominal yield

*payroll deduction plans are ________ plan

non-qualified

section 457 plans-nonqual or qual?

non-qualified

**are REITs investment companies?

nope

*are REIT considered DPP?

nope

*are hedge funds an investment company?

nope

*does investment companies include holding companies?

nope

*does net direct debt or net total debt include self supporting debt & sinking fund accumulation?

nope

*family balance sheet does it show dividends, salary, interest or paid expenses?

nope

are warrants a good option for someone who is seeking income?

nope

do revenue bonds require voter approval?

nope

is time or price included in discretion?

nope

ETFs are they actively managed or not actively managed?

not actively managed

interest issues of US territories is taxed on

nothing

interest earned on any security issued by the US treasury is taxable ___

on the federal level -exempt from the state and local level

Issued stock - Treasury stock = ____________________

outstanding stock

when a call option is covered it means that the investor

owns the number of shares covered by the option contract

partnership accounts need to have a _______to open the account stating who can make the transactions

partnership agreement

An investor in a limited partnership generating passive losses can offset these against passive income from other partnerships. rental income from direct investments in real estate. dividends received from listed securities. capital gains from the sale of unlisted securities.

passive income from other partnerships. rental income from direct investments in real estate. Passive losses can be deducted from passive income and income from certain real estate investments; it cannot be deducted from active or portfolio (investment) income. LO 11.f

Hypothecation Agreement

permits the broker-dealer to pledge the client's margin securities as collateral for a loan that the BD takes out. In simple terms, there are two loans taking place: The loan from the BD to the client with the client's securities used as collateral. That is covered in the credit agreement The loan from a bank to the BD with the client's securities used as collateral for the BD's loan. The authorization for the BD to use those securities is found in the hypothecation agreement.

Annuitant

person who pays the premium in one lump sum or in periodic payments

Revenue Bonds are rated according to a facility's

potential to generate sufficient money to cover operating expenses and principal and interest payments -not subject to statutory debt limits

Variable Life Insurance-premiums

premiums are split, part of the premium is placed in the general assets of the insurance company & the other part goes into the separate account and represents the cash value of the policy -cash value not guarenteed

SIMPLE plans have contributions that are ____ tax

pretax

Pretax Income

pretax income=taxable income - interest payment expenses

stop orders are designed to

prevent a loss or protect a profit

secondary market

previously issued securities are traded among investors

firm quote

price at which a market maker stands ready to buy or sell at least one trading unit-100 shares of stock or 5 BONDS

close end fund benefits-pricing

price of CEF is based on supply and demand -can result in the fund selling at a premium or discount from its NAV -mutual fund can never be purchased at a discount from its NAV

CMO are paid by

principal and interest from the mortgage pool monthly HOWEVER it repays principal to only one tranche at a time

how are revenue bonds paid?

principal and interest on revenue bonds are paid exclusively from money generated by the facility the issuer finances

Classes of CMOs

principal only interest only planned amortization class targeted amortization class

TOD accounts avoid

probate court

REITs offer

professional management and diversification -publicly traded -source of long term financing for real estate projects

corporate charter

proof that the corporation exists

bonds issued by cities, towns, and countries are backed by

property (ad valorem) taxes, license fees, fines, and other sources of revenue to the municipality

community property

property acquired during marriage and owned equally by both spouses

a mutual fund is offered by

prospectus through the SEC

the benefit of having a prime brokerage account

provides the client with the ability to trade w/ multiple brokerage houses while keeping a central main account with their cash & securities

Currency or exchange rate risk

purchasers of foreign securities face the uncertainty that the value of either foreign currency or the domestic currency will flucuate

variable annuity protects against this risk

purchasing power risk

investors can use ____ to protect a long stock position

puts

nondiscrimination rule apply to _____ plans

qualified

employer can deduct contributions made into a

qualified account

a legal opinion is either issued as

qualified or unqualified

members are required to send statements to customers at least

quarterly -monthly for penny stocks

Quick Asset Ratio

quick assets (cash and debtors) /current liabilities (share premium account)

issuing nonvoting stock allows a company to

raise additional capital while maintaining management control and continuity without diluting voting power

when the OCC is notified by a BD that one of its customers wants to exercise the OCC

randomly selects a firm with a short position in that option to which it assigns the exercise

The holder of a yield-based call option would be more likely to profit if rates rise. rates fall. debt prices rise. debt prices fall.

rates rise. debt prices fall. Holders of yield-based call options profit if rates rise. Prices of debt securities fall if rates rise. LO 10.g

the seller of an option __________the prem

receives

sovereign debt

refers to debt that is accrued by the sovereign, that is, the government

UIT benefit-rolling over proceeds

regardless of a debt UIT or equity UIT, there is a date when the trust terminates -investors have the option of investing the proceeds into a new trust without charge

Close-ended investment company number of shares

registers the fixed, number of shares with the SEC & offers them through a limited time

The Securities Exchange Act of 1934 created the SEC, required the

registration of broker-dealers, empowered the Federal Reserve to control the extension of credit on securities transactions, and created rules dealing with secondary market trading.

when the RR checks the execution report alongside with the order ticket, if everything is correct the RR

reports the execution to the customer

penny stock rules: customer suitability determination

requires members who are soliciting members to make a suitability determination -income, net worth, objectives risk tolerance -suitability statement is produced with this and is sent to the client to sign and is sent back to the firm, once returned trading can begin

penny stock rule: disclosure of quotations

requires to provide purchases with current bid and ask quote on the stock to prevent the practice of quoting prices that are away from the current market

Technical Analysis

research on recurrent and predictable stock price patterns and on proxies for buy or sell pressure in the market

Tax Reform Act of 1986

restricted the federal income tax exemption of interest for municipal bonds to public purpose bonds, which are bonds issued to finance projects that benefit citizens in general rather than particular private interests

buyerss of calls want the market price of the underlying stock to

rise

interest rates fall, preferred stock prices

rises

Sovereign Risk

risk associated with a government defaulting on its debt obligations -nonsystematic risk

systematic risk

risk in the return of an investment that is associated with the macroeconomics factors that affect all risky assets -CANNOT BE DIVERSIFIED AWAY

the lower the debt limit for bonds, the less

risk of excessive borrowing and default by the municipality -debt limits can also make a bond safer for investors

Common Stock Limited Liability

s/o cannot lose more than what they have invested -if company goes bankrupt, collectors cannot come after s/o

FINRA rule for sales charges for variable annuities

sales charges must be reasonable

mutual fund risk-expense risk

sales charges, 12b-1 feed, redemption fees -management fees -tax efficiency, s/o has no control over the manager's timing of purchases and sales

same series of options

same class, exercise price & expiration month

same class of options

same underlying security

modern portfolio theory

scientific approach to measure risk and choosing investments -minimizing risk by combining volatile and price stable investments in a single portfolio -seeks to reduce risk while simultaneously increasing expected returns -wants negative correlation

municipal securities are considered ____________in safety of principal

second in safety of principal only behind US government and US government securities

if an investor thinks the value of the currency is going to fall, they should

sell calls or buy puts on the currency

investors in a close end IC close their position by

selling them in the secondary market

*bonds are paid

semiannually -coupon rate divided by 2 equals the semiannual payment

bond interest are generally paid

semiannually based on a stated coupon rate

risk of investing in a funds of hedge fund

shares are NOT traded in the secondary market

if a stock's holding period is 12 months or less before the purchase of the put, the gain will be classified as

short term

Treasury Bills (T-Bills)

short-term debt obligations the U.S. government sells to raise money -less than 1 year -issued at a discount from par -considered money market instrument

Repurchase Agreements

short-term sales of government securities with an agreement to repurchase the securities at a higher price

*duration maturity rule

shortest time, lower duration

Nominal quote

someone's assessment of where a stock might trade in an active market Must be clearly labeled

Option Clearing Corporation (OCC)

standardizes and guarantees the performance of and issue contracts

interest on municipal securities is taxable on

state and local levels

if the separate account performance for the year is equal than the AIR, the death benefit will

stay the same

cost basis of inherited property

stepped up or down to its fair market value (FMV) at the date of the decedent's death -sale of inherited shares are subjected to a more favorable long-term tax rate no matter how long they have been held by the bene

Blend/Core Funds

stock funds with a portfolio comprised of a number of different classes of stock allows investors to diversify their investment via management and secs in a single fund

triple taxation may be avoided if the mutual fund qualified under

subchapter M of IRA -if a mutual fund acts as a conduit (pipeline) for the distribution of NII the fund may qualify as a regulated investment company -90% of NII must be distributed to s/o -the undistributed amount is responsible for taxes

tender offer

takeover attempt in which outsiders directly offer to buy the stock of the firm's shareholders

*401(k) benefit

takes advantage of dollar cost averaging resulting in a lower cost per share in a fluctuating market

taxes for annuities are

tax deferred

1035 exchange

tax free exchange between contracts for annuities -IRS allows annuity and life insurance policyholders to exchange their policies w/o current liability

distributions are _______to s/o whether the distributions are received in cash or reinvested

taxable


Related study sets

Bio 131 Regulation of Gene Expression Weeks 10/11 Lecture 1

View Set

NUR 108 Test 2 Perry & Potter After Chapter Quiz ?'s

View Set

Chapter 2: Solids, Liquids, and Gases

View Set

AC Exam #16 - Project Delivery Methods

View Set

Avoiding Group Harms - International Research Perspectives

View Set

The Fall of the Bastille --- 14 July 1789

View Set

Management Chapter 2 Study Guide

View Set